Construa o gráfico da função 1 e da função 2 usando um único plano cartesiano disponível a seguir


917   

Suponha que o número de carteiros necessários para distribuir, em cada dia, as correspondências entre as residências de um bairro seja dado pela função $f(x)=\frac{22x}{500+2x}$, em que $x$ é o número de residências e $f(x)$ é o número de carteiros. Se foram necessários 6 carteiros para distribuir, em um dia, estas correspondências, qual o número de residências desse bairro, que as receberam?



Substituindo $f(x) = 6$ na expressão da função:

$6 = \dfrac{22 x}{500+2x}$
$\Rightarrow 6(500+2x) = 22x$
$\Rightarrow 3000 + 12x = 22x$
$\Rightarrow 10x = 3000$
$\Rightarrow x = 300$ residências.


621   

Um funil de volume especificado deve ter a forma de um cone circular reto. Encontre a razão da altura pelo raio da base para que a quantidade de material empregado em sua fabricaçao seja a menor possível.


1667   

Calcule a integral a seguir:

$\int{\frac{9r^2\ dr}{\sqrt{1-r^3}}}$


622   

Se um corpo de peso $P$ é arrastado ao longo de um piso horizontal  por meio de uma força de grandeza $F$ e orientada segundo um  ângulo $\theta$ radianos com o plano do piso, então $F=\frac{kP}{k\sin \theta +\cos \theta}$, onde $k$ é  uma constante. Encontre $\cos \theta$, quando $F$ for mínimo.


619   

Determine o domínio da seguinte função:

$f\left( x\right) =\sqrt{x-\sqrt{x}}$.


$\left\{ x\geq 1\right\} $.


1524   

De acordo com a teoria da relatividade, o comprimento de um objeto parece, a um observador, depender da velocidade relativa entre este e o objeto. Se o observador estabelecer o comprimento do objeto, em repouso, como $L_0$, então o comprimento, a uma velocidade $v$, parecerá:
$L=L_0\sqrt{1-\frac{v^2}{c^2}}$.
Esta equação é chamada Fórmula de Contração de Lorentz, sendo que $c$ é a velocidade da luz no vácuo, em torno de $3\times10^8m/s$. Qual o comportamento de $L$ conforme $v$ aumenta?
Determine $\lim\limits_{v\rightarrow c^- }L$. Por que o limite lateral à esquerda foi necessário, e como esta necessidade se relaciona com as Leis da Física?



747   

Calcule o seguinte limite:
$\lim\limits_{x\rightarrow 0}\left( 1+2x\right) ^{\dfrac{1}{x}}$.


$e^2$.


596   

Mostre que $x\neq y\Longrightarrow x^{2}+2xy<2x^{2}+2y^{2}$.



Note que $(x-y)^2+y^2>0$ sempre que $x\neq y$. Daí, $x^2-2xy+y^2+y^2>0$, que é equivalente a $2x^2+2y^2-x^2-2xy>0$, que, por sua vez, é equivalente a x^{2}+2xy<2x^{2}+2y^{2}$.


1905   

Seja $y=f(x)$ uma curva suave em $\left[a,b\right]$. Prove que se houver números não-negativos $m$ e $M$, tais que $m \leq f'(x) \leq M$ para todo $x$ em $\left[a,b\right]$, então o comprimento de arco $L$ de $y=f(x)$ satisfaz a desigualdade $(b-a)\sqrt{1+m^2} \leq L \leq (b-a) \sqrt{1+M^2}$.


1637   

Na lei logística de crescimento admite-se que, no instante $t$, a taxa de crescimento $f'(t)$ de uma quantidade $f(t)$ seja dada por $f'(t)=Af(t)(B-f(t))$, com $A$ e $B$ constantes. Se $f(0)=C$, mostre que $f(t)=\dfrac{BC}{C+(B-C)e^{-ABt}}$.


1903   

Uma hipociclóide de quatro cúspides (também chamada astróide) é a curva dada paramétricamente pelas equações $x=a\cos^3 \theta$ e $y=a \sin^3 \theta$.

  1. Use um recurso gráfico para gerar o gráfico de uma astróide usando $a=1$.
  2. Ache o comprimento exato de uma astróide.


214   

Avalie os seguintes limites para a função definida por partes
$ f(x) = \left\{\begin{array}{ccc}
x+1, & &  \text{ se } x<1 \\
1,  & & \text{ se }  x=1\\
x-1, & &  \text{ se } x>1
\end{array}
\right.$

  1. $ \lim\limits_{x\to 1^-} f(x)$

  2. $ \lim\limits_{x\to 1^+} f(x)$

  3. $ \lim\limits_{x\to 1} f(x)$

  4. $f(1)$


  1. 2
  2. 0
  3. Não existe
  4. 1


1296   

A região no plano $xy$ limitada pela curva $y=x^2+1$ e pela reta $y=-x+3$ gira em torno do eixo $x$ gerando um sólido $S.$ Calcule o volume de $S.$


1668   

Calcule a integral a seguir:

$\int{\cos 2x\ dx}$


$sin(x)cos(x)+C$


903   

Resolva a equação $\sqrt{9x+4} + \sqrt{3x-4} = 2 \sqrt{3x}$.


102   

Mostre que a função \begin{align*} f\left( x\right) =\left\{ \begin{array}{cc} \dfrac{x^{3}-4x}{x^{2}-4}, & \text{se } x\neq \pm 2 \\ 2, & \text{se } x=2 \\ -3, & \text{se } x=-2 \end{array} \right. \end{align*} é contínua em todos os pontos, com exceção do ponto $x=-2$.


652   

Esboce o gráfico de cada uma das funções abaixo.

  1. $y=\frac{2|x+1|}{3}$

  2. $y=\sqrt{5-x^{2}}$


1571   

Seja $f(x)=\left\{\begin{array}{ll}
-x+3, & \text{se } x<3 \\
x-3, & \text{se } x \geq 3  
\end{array}\right.$

  1. $f$ é contínua em $3$. Por quê?
  2. $f$ é derivável em $3$. Por quê?


623   

Uma página impressa deve ter $24cm^2$ de área reservada à parte escrita, uma margem de $1,5 cm$ nas partes superior e inferior e uma margem de $1cm$ nos lados. Quais as dimensões da página de menor área que preenche essas condições?


1341   

Obtenha as assíntotas verticais de $f(x)=\frac{x^2+1}{(x-1)^2}$.



As assíntotas verticais são os pontos $x$ tais que o limite é infinito.

Para $f(x)=\frac{x^2+1}{(x-1)^2}$ temos que:

$\lim \limits_{x \to 1} \frac{x^2+1}{(x-1)^2} = \infty$,

Logo $x=1$ é uma assíntota vertical de $f$. Como não há mais pontos no domínio de $f$ que podem levar a um limite infinito, esta é a única assíntota.


1103   

Avalie a seguinte integral indefinida:
  $\int 5e^\theta\  d\theta$


  $5e^\theta+C$


1196   

Calcule a derivada da seguinte função:
 $f\left(  x\right)  =3^{2x}\ln\left(  x^{2}\right)  .$


$2\ 3^{2 x} \log (3) \log \left(x^2\right)+\frac{2\ 3^{2 x}}{x}$


68   

Suponha que para todo $x$, $\left| f\left( x\right) \right| \leq x^{4}$. Calcule $\lim\limits_{x\rightarrow 0}\dfrac{f\left( x\right) }{x}.$


41   

Calcule os seguintes limites:

  1. $\lim\limits_{x\rightarrow \infty }\left( 1+\dfrac{1}{x}\right)  ^{x+2}$

  2. $\lim\limits_{x\rightarrow \infty }\left( 1+\dfrac{1}{2x}\right) ^{x}  $

  3. $\lim\limits_{x\rightarrow \infty }\left( \dfrac{x+2}{x+1}\right)  ^{x}$


1530   

A resposta do corpo humano a uma dose de um medicamento pode ser representada pela equação:
$$R=M^2\left(\dfrac{C}{2}-\dfrac{M}{3}\right),$$
onde $C$ é uma constante positiva e $M$ a quantidade de medicamento absorvida pelo sangue. Se $R$ for uma variação da pressão sanguínea, é medida em milímetros de mercúrio; se for variação de temperatura, é medida em graus. Determine a sensibilidade do organismo ao medicamento,  $dR/dM$.


731   

Calcule o seguinte limite:

$\lim\limits_{x\rightarrow \infty }\ln \dfrac{x}{x+1}$.


$0$


566   

Estude a função $f\left( x\right) =\dfrac{x^{3}}{x^{2}-1}$ com relação à concavidade, pontos de inflexão, máximos e mínimos, e esboce o seu gráfico.


608   

Use a desigualdade triangular $\left| a+b\right| \leq \left| a\right|+\left| b\right| $\emph{ }para mostrar que $\left| x-y\right| \geq \left|x\right| -\left| y\right| $ para todo $x,y\in \mathbb{R}$. Em particular, conclua que $\left| x-y^{2}\right| \geq \left| x\right| -y^{2}.$


937   

Determine a função inversa de:

  1. $f(x) = x^2$
  2. $f(x) = x^3 + 2.$


1131   

Aproxime numericamente o seguinte limite

  $ f(x)=\frac{x^2-11 x+30}{x^3-4 x^2-3 x+18}$


  1.   \begin{array}{cc}
      x & f(x) \\ \hline
      2.9 & 132.857 \\
       2.99 & 12124.4 \\
        \end{array}
       A tabela parece indicar que $\lim\limits_{x\to3^-}f(x) =\infty$.
  2.     \begin{array}{cc}
      x & f(x) \\ \hline
       3.1 & 108.039 \\
       3.01 & 11876.4 \\
        \end{array}
        A tabela parece indicar que $\lim\limits_{x\to3^+}f(x) =\infty$.
  3.   As tabelas parecem indicar que $\lim\limits_{x\to3}f(x) =\infty$.


1106   

O que é um problema de valor inicial?


1261   

Se $ F(x)= { \int_1^xf(t)dt}$ e $ f(t)={ \int_{x^2}^1\frac{\sqrt{1+u^4}}{u}du}$, determine $F''(2)$.


1821   

A figura abaixo mostra o gráfico do polinômio $\displaystyle y=\dfrac{1}{10}x^5(x-1)$ gerado no software Mathematica$^\textrm{TM}$ usando uma janela de $[-2;2,5]\times[-1;5]$.

Construa o gráfico da função 1 e da função 2 usando um único plano cartesiano disponível a seguir

Mostre que a escolha da escala vertical faz com que o computador perca importantes aspectos do gráfico. Descreva quais são os aspectos perdidos e faça o seu próprio esboço indicando-os.


1513   

Dê os domínios e esboce os gráficos de $f+g$ e $\dfrac{g}{f}$ nos seguintes casos:

  1. $f(x)=x$ e $g(x)=x^2-1$.
  2. $f(x)=x$ e $g(x)=\dfrac{1}{\sqrt{x}}$.


851   

Determine as derivadas das seguintes funções:

  1. $f\left( x\right) =e^{\cos \left( x^{2}\right) }$.

  2. $f\left( x\right) =\left( \sin x+\cos x\right)^{3}$.

  3. $f\left( x\right) =x^{3}e^{-3x}.$


671   

Calcule a integral $\int x^{2}e^{x^{3}}dx$.


$\dfrac{e^{x^3}}{3}+C$.


677   

Mostre que a equação $\sin x +\cos x =0$ tem exatamente duas raízes reais.


1572   

Prove que se $f$ for derivável em $p$, então $f$ será contínua em $p$. 



Veja Guidorizzi, volume $1$, página $152$.


74   

Identifique as assíntotas verticais e horizontais, caso existam, da função

  $f(x)=\frac{x^2+x-12}{7 x^3-14 x^2-21 x}$.


 Assíntota horizontal em $y=0$; assíntotas verticais em $x=-1$ e $x=0$.


609   

Sejam $x$ e $y$ dois números reais positivos. Demonstre que $\sqrt{xy}\leq \dfrac{x+y}{2}.$



Elevando ambos os membros da expressão $\sqrt{xy}\leq \dfrac{x+y}{2}$ ao quadrado obtemos $xy\leq \dfrac{x^2+2xy+y^2}{4}$. Simplificando chegamos a $x^2-2xy+y^2=(x-y)^2 \geq0$. Como a última expressão obtida é verdadeira e todas as expressões são equivalentes entre si, segue o resultado. Esse resultado diz que a média geométrica entre dois dois números reais positivos é sempre menor que, ou igual, à média aritmética entre eles.


1126   

Seja:

  • $\int_0^3{s(t)dt} = 10$
  • $\int_3^5{s(t)dt} = 8$
  • $\int_3^5{r(t)dt} = -1$ e
  • $\int_0^5{r(t)dt} = 11$

A partir destes valores, calcule as seguintes integrais:

  1. $\int_0^3 \big(s(t) + r(t)\big)\ dt$
  2. $\int_5^0 \big(s(t) - r(t)\big)\ dt$
  3. $\int_3^3 \big(\pi s(t) - 7r(t)\big)\ dt$
  4. Encontre valores para $a$ e $b$ tal que:
    $\int_0^5 \big(ar(t)+bs(t)\big) \ dt=0$


  1. $22$
  2. $-7$
  3. $0$
  4. $b=-\frac{11}{18}a,\quad a\in\mathbb{R}$


800   

Resolva os itens.

  1. Considere a parábola $y=x^{2}$ e faça a seguinte construção: para cada $a\neq 0$ trace a reta normal à parábola no ponto $\left( a,a^{2}\right) $ e seja $P$ o ponto onde essa normal encontra o eixo $y$. Calcule o limite do ponto $P$ quando $a$ tende a zero.

  2. Calcule o mesmo limite fazendo a mesma construção para a curva quártica $y=x^{4}$ em lugar da parábola.


791   

Determine a equação da reta tangente em $\left( p,f\left(p\right) \right)$:

$f\left( x\right) =x^{2},\;p=2$.


$y=4x-4$.


171   

Seja $h$ uma função definida em $[-1,1]$, sendo que $h(-1) = -10$ e $h(1) = 10$. Existe um valor $-1<c<1$ tal que $h(c) = 0$? Por quê?


Não é possível dizer: O Teorema do Valor Intermediário só se aplica para funções contínuas, e nada foi afirmado sobre a continuidade de $h$.


892   

Resolva a equação modular $||x-2|-|x-1|+1| =2$.


802   

Calcule $f^{\prime }\left( x\right)$:

$f\left( x\right) =x^{2}e^{x}$.


$f'(x)=e^x(x^2+2x)$.



Usando a regra da derivada do produto, temos que

\[f^\prime(x) = (x^2  e^x)^\prime = (x^2)' \cdot e^x + x^2 \cdot (e^x)^\prime.\]

Como $(x^2)^\prime = 2x$ e $(e^x)^\prime = e^x$, então

\[(x^2)' \cdot e^x + x^2 \cdot (e^x)^\prime = 2x e^x + x^2 e^x.\]

Colocando o fator comum $e^x$ em evidência, concluímos que

\[f^\prime (x) = e^x (x^2 + 2x).\]


961   

Calcule e justifique os seguintes limites, quando existirem, ou justifique a inexistência:

$\lim\limits_{x\rightarrow 0}\dfrac{\tan x}{x}$
$\lim\limits_{x\rightarrow 0}\dfrac{x^{3}}{\sin x}$


1581   

Dados $f(x) = 2x^2+4x-3$ e $x_0 = -0,9$, escolha um valor inteiro próximo a $x_0$ tal que $f(x_0)$ e $f'(x_0)$ sejam fáceis de calcular, e calcule uma linearização da função neste ponto.


1246   

Esboce o gráfico da funçao $f\left( x\right) =\frac{x^{2}}{x-1}$, indicando domínio, limites laterais e no infinito, assíntotas verticais e inclinadas, intervalos de crescimento e decrescimento e estudo da concavidade.


536   

Ache os pontos de máximo e de mínimo absolutos da função $f(x)=x+3x^{2/3}$.


616   

Esboce o gráfico da função abaixo e resolva a inequação:

 $f\left( x\right) =\left\vert 2x^{2}-1\right\vert <1$.


1829   

Seja $f(x)=(x^3+1)/x$. Mostre que o gráfico de $y=f(x)$ tende à curva $y=x^2$ "assintotamente" no sentido de que $$ \lim_{x\to\pm\infty}\left[f(x)-x^2\right] = 0. $$ Esboce o gráfico de $y=f(x)$ mostrando o seu comportamento assintótico.


335   

Seja $f(x)=x^n+5x^{n-1}+3$, onde $n>1$ é um inteiro. Prove que $f(x)$ não pode ser expressa como um produto de polinômios não constantes com coeficientes inteiros.


1127   

Um objeto é lançado para cima com uma velocidade, em pés por segundo, dada por $v(t) = -32t+96$; de uma altura de $64$ pés.

  1. Qual a velocidade inicial do objeto?
  2. Em que momento o objeto tem deslocamento nulo?
  3. Quanto tempo leva para o objeto retornar a sua posição inicial?
  4. Quando o objeto alcançará uma altura de $210$ pés?


  1. $96ft/s$.
  2. $6s$.
  3. $6s$.
  4. Nunca, a altura máxima é $208ft$.


739   

Encontre o volume do sólido obtido pela rotação da região limitada pela curva dada em torno do eixo especificado. Esboce a região e o sólido.
  $y=x^{2},0\leq x\leq 2,y=4,x-0$ ao redor do eixo $y.$


1122   

Com base no gráfico, avalie as seguintes integrais:

Construa o gráfico da função 1 e da função 2 usando um único plano cartesiano disponível a seguir

  1. $\int_0^2 f(x)\ dx$
  2. $\int_2^4 f(x)\ dx$
  3. $\int_2^4 2f(x)\ dx$
  4. $\int_0^1 4x\ dx$
  5. $\int_2^3 (2x-4)\ dx$
  6. $\int_2^3 (4x-8)\ dx$


  1. $4$
  2. $2$
  3. $4$
  4. 2
  5. $1$
  6. 2


1651   

Um objeto é solto de um helicóptero. O objeto cai cada vez mais rápido, mas sua aceleração diminui com o passar do tempo devido à resistência do ar. A aceleração foi medida nos primeiros cinco segundos, quando ele atingiu o chão, e o resultado está na tabela a seguir:


$
\begin{array}{ccccccc} \hline
t & 0    & 1    & 2    & 3    & 4    & 5    \\\hline
a & 9,81 & 5,95 & 3,61 & 2,19 & 1,33 & 0,81 \\\hline
\end{array}
$

  1. Faça uma estimativa superior para o módulo da velocidade quando $t=5$.

  2. Faça uma estimativa superior para o módulo da posição quando $t=5$.

  3. Faça uma estimativa superior para a altura da queda.


803   

Calcule $f^{\prime }\left( x\right)$:

$f\left( x\right) =e^{x}\cos x$.


$f'(x) = e^x(\cos x - \sin x)$.



Usando a regra da derivada do produto, temos que

\[f^\prime(x) = (e^x \cos x)^\prime = (e^x)^\prime \cdot \cos(x) + e^x \cdot (\cos x)^\prime.\]

Como $(e^x)^\prime = e^x$ e $(\cos x)^\prime = -\sin x$, então

\[(e^x)^\prime \cdot \cos(x) + e^x \cdot (\cos x)^\prime = e^x \cos x + e^x (-\sin x) .\]

Colocando o fator comum $e^x$ em evidência, concluímos que

\[f^\prime (x) = e^x (\cos x- \sin x).\]


1656   

Demonstre que não é possível que o valor de $\int_0^1\sin(x^2)\ dx$ seja $2$. Depois, utilizando a desigualdade $\sin x \leq x$, válida para $x \geq 0$, determine um limitante superior para esta integral.



Sabemos que $\sin x \leq 1,\,\forall\,x\in\mathbb{R}$. Assim, como $\int_a^b f(x)dx \leq max\_{a \leq x \leq b} f(x) (b-a)$, podemos dizer que $\int_0^1\sin(x^2)\ dx \leq 1$.

Utilizando a desigualdade $\sin x \leq x$, podemos determinar de maneira ainda mais precisa um limitante superior para a integral.

 $\int_0^1\sin(x^2)\ dx \leq \int_0^1x^2\ dx = \frac{1}{3}x^3 \vert^1_0=\frac{1}{3}$


885   

Mostre que a equação $|ax-b|=r$, com $r\geq 0$ e $a\neq 0$, tem como soluções os elementos do conjunto $\left\lbrace \frac{b+r}{a},\frac{b-r} {a}\right\rbrace$.



Temos duas possibilidades: $ax-b=r$ ou $ax-b=-r$. Da primeira equação obtemos $x=\dfrac{b+r}{a}$ e da segunda$x=\dfrac{b-r}{a}$. 


1089   

Use suas próprias palavras para definir o significado de $\int{f(x)}\ dx$.


O símbolo $\int{f(x)}\ dx$ é chamado integral indefinida de $f$ e corresponde ao conjunto de todas as antiderivadas da função $f$.


827   

Calcule $f^{\prime }\left( x\right)$:

  $f\left( x\right) =\pi ^{x}$.


$f'(x)=ln(\pi)\pi^x$.


1306   

Calcule o limite $\lim_{x\rightarrow 1} \frac{\sqrt{x+1}-\sqrt{2}}{\sqrt{x^2+3}-2}$ ou prove que não existe.



Racionalizando e aplicando diferença de quadrados temos:
\begin{equation*}
\frac{\sqrt{x+1}-\sqrt{2}}{\sqrt{x^2+3}-2} = \frac{\sqrt{x+1}-\sqrt{2}}{\sqrt{x^2+3}-2}\cdot \frac{\sqrt{x+1}+\sqrt{2}}{\sqrt{x+1}+\sqrt{2}}\cdot \frac{\sqrt{x^2+3}+2}{\sqrt{x^2+3}+2} =
 \frac{x-1}{x^2-1}\cdot\frac{\sqrt{x^2+3}+2}{\sqrt{x+1}+\sqrt{2}}.
\end{equation*}
Logo,
$\lim_{x\rightarrow 1} \frac{\sqrt{x+1}-\sqrt{2}}{\sqrt{x^2+3}-2}=\lim_{x\rightarrow 1}\frac{x-1}{x^2-1}\cdot\frac{\sqrt{x^2+3}+2}{\sqrt{x+1}+\sqrt{2}}=\lim_{x\rightarrow 1}\frac{1}{x+1}\cdot\frac{\sqrt{x^2+3}+2}{\sqrt{x+1}+\sqrt{2}}= \frac{1}{\sqrt{2}}.$


520   

Uma escada de $10$ metros de comprimento está apoiada em uma parede vertical. Se a base da escada começa a escorregar horizontalmente a uma taxa constante de $0,6 m/s$, com que velocidade o topo da escada percorre a parede quando ele está a $6 m$ do solo?


1584   

Dados $f(x) = \dfrac{x}{x+1}$ e $x_0 = 1,3$, escolha um valor inteiro próximo a $x_0$ tal que $f(x_0)$ e $f'(x_0)$ sejam fáceis de calcular, e calcule uma linearização da função neste ponto.


1895   

Mostre que se $f$ é contínua e côncava para cima em $\left[a,b\right]$, então $f_{med}>f\left(\dfrac{a-b}{2}\right)$, onde $f_{med}$ é o valor médio da função $f$ no intervalo $\left[a,b\right]$.


530   

Suponha que uma gota de neblina seja uma esfera perfeita e que, por condensação, capte umidade a uma taxa proporcional à área de sua superfície. Mostre que nessas circunstâncias o raio da gota cresce a uma taxa constante.


1600   

Um caminhoneiro estava em uma estrada cujo limite de velocidade era de $100km/h$. Ao passar no segundo pedágio, distante $120km$ do primeiro, o caminhoneiro recebeu uma multa, pois levou $30$ minutos para ir do primeiro ao segundo pedágio. Ele tentou contestar a multa, mas não obteve sucesso. Por que a multa foi justa?


1187   

Calcule a derivada da seguinte função:
 $f\left(  x\right)  =\frac{e^{x}-e^{-x}}{2}.$


 $f'\left(  x\right)  =\frac{e^{x}+e^{-x}}{2}.$


92   

Calcule o limite $\lim\limits_{x\to 3} \frac{x^2-2x-3}{x^2-4x+3}$.


$2$


624   

Entre todos o cilindros retos que tem uma área total dada, ache o que tem volume máximo.


595   

Resolva a equação $\left| {\frac{3x+8}{2x-3}}\right| =4$.



Temos duas possibilidades: $\frac{3x+8}{2x-3}=4$ ou $\frac{3x+8}{2x-3}=-4$. Da primeira equação obtemos $3x+8=8x-12$, i. e., $x=4$. Da segunda equação obtemos $3x+8=-8x+12$, que fornece $x=4/11$.


1523   

Suponha $g(x) \neq 0$, para todo $x \in Dom(g)$, $L \neq 0$ e $\lim\limits_{x \to p}g(x)=L$. Prove que $\lim\limits_{x \to p}\dfrac{1}{g(x)}=\dfrac{1}{L}$.



 Veja Guidorizzi, volume $1$, página $87$.


120   

Seja $f$ uma função contínua e decrescente em $\left[a,b\right]$. Mostre que $f$ tem uma inversa decrescente em $\left[f(b),f(a)\right]$.


1299   

Utilize a fórmula
  \[
  S=\int_{a}^{b}2\pi f\left(  x\right)  \sqrt{1+\left(  f^{\prime}\left(
  x\right)  \right)  ^{2}}dx
  \]
  para mostrar que a superfície de uma esfera de raio $R$ é $4\pi
  R^{2}$.



Uma esfera de raio $R$ centrada na origem pode ser obtida pela rotação ao redor do eixo $x$ do semicírculo $x^{2}+y^{2}=R^{2}$ com $y\geq0$. Este semicírculo pode ser visto como o gráfico da função $f\left(  x\right)  =\sqrt{R^{2}-x^{2}}$, com $t\in\left[-R,R\right]  $.
  Considerando $f\left(  x\right)  =\sqrt{R^{2}-x^{2}}$ temos que:
  \[
  f^{\prime}\left(  x\right)  =-\frac{t}{\sqrt{R^{2}-x^{2}}}\text{.}%
  \]
  Usando a fórmula acima temos que:
  \begin{align*}
  S  & =\int_{-R}^{R}2\pi\sqrt{R^{2}-x^{2}}\sqrt{1+\left(  f^{\prime}\left(
  x\right)  \right)  ^{2}}dx\\
  & =\int_{-R}^{R}2\pi\sqrt{R^{2}-x^{2}}\sqrt{1+\frac{x^{2}}{R^{2}-x^{2}}}dx\\
  & =\int_{-R}^{R}2\pi\sqrt{R^{2}-x^{2}}\sqrt{\frac{\left(  R^{2}-x^{2}\right)
  +x^{2}}{R^{2}-x^{2}}}dx\\
  & =\int_{-R}^{R}2\pi\sqrt{R^{2}-x^{2}}\sqrt{\frac{R^{2}}{R^{2}-x^{2}}}dx\\
  & =\int_{-R}^{R}2\pi R\frac{\sqrt{R^{2}-x^{2}}}{\sqrt{R^{2}-x^{2}}}dx\\
  & =\int_{-R}^{R}2\pi Rdx
  \end{align*}
  Temos então que:
  \begin{align*}  S  & =\int_{-R}^{R}2\pi Rdx\\  & =\left.  2\pi Rx\right\vert _{-R}^{R}\\  & =4\pi R^{2}  \end{align*}


50   

Avalie os seguintes limites de acordo com o gráfico da função:

  $f(x) = x^2\sin (\pi x)$

Construa o gráfico da função 1 e da função 2 usando um único plano cartesiano disponível a seguir

  1. $\lim\limits_{x\to -\infty} f(x)$

  2. $\lim\limits_{x\to \infty} f(x)$


519   

Enche-se um balão esférico a uma taxa de $4,5$ decímetros cúbicos por minuto. Calcule a taxa de variação do raio quando este medir $2$ decímetros.


1817   

Calcule $F'(x)$ sendo $F(x)$ igual a:

  1. $x^2e^x\cos{x}$
  2. $e^x \sinh{x} \cos^2{x}$


1295   

A região limitada pelo triângulo de vértices $(1,0),$ $(2,1)$ e $(1,1)$ gira em torno do eixo $y$ gerando um sólido $S.$ Calcule o volume de $S.$


1184   

Determine a derivada da seguinte função:
  $f\left( x\right) =\ln \left( 3\cos ^{5}\left( 4x\right)\right) .$


$f'(x) = -20\tan(4x)$.


63   

Calcule, quando existirem, os seguintes limites:

  1. $\lim\limits_{x\rightarrow 2}\dfrac{x^{2}+x-6}{  \left( x-2\right) ^{3}}$

  2.  $\lim\limits_{x\rightarrow 2}\dfrac{\sqrt{6-x}-2}{\sqrt{  3-x}-1}$

  3. $\lim\limits_{x\rightarrow \infty }\sqrt{3x+4}-\sqrt{3x}.$


1595   

Se uma função par $f(x)$ possui um valor máximo local em $x=c$, pode-se dizer algo sobre o valor de $f$ quando $x=-c$?


Ela também terá um máximo local em $x=-c$. É uma questão de simetria de seu gráfico em relação ao eixo das ordenadas.


1690   

Existe uma curva continuamente derivável $y=f(x)$ cujo comprimento ao longo do intervalo $0\leq x\leq a$ seja sempre $\sqrt{2}a$?


1613   

Calcule o limite $\lim\limits_{x \to \infty}\dfrac{x^n}{e^x}$.


$0$.


140   

Para a função a seguir, dê os intervalos nos quais ela é contínua:

$ g(t) = \frac{1}{\sqrt{1-t^2}}$.


  $(-1,1)$


1166   

Encontre, se existirem, o valor máximo absoluto e o valor mínimo absoluto da função $f(x)= \sqrt[3]{x^3-x^2},$ no intervalo $[0,1].$


148   

Mostre que a equação

  \begin{equation*}
  x^{26}+x^{2}-320=0
  \end{equation*}

  possui ao menos uma raiz real positiva e também uma raiz real negativa.


896   

Esboce o gráfico da função $f(x)=|(x-1)^2-3|$.


172   

Calcule o limite a seguir:

  $\lim\limits_{x \to -\infty } e^x \sin(x)$



Observe que $-1 \leq \sin(x) \leq 1$ e, portanto, como $e^x \geq 0$, $-e^x \leq e^x \sin(x) \leq e^x$.

  Como $\lim\limits_{x \to -\infty} e^x = 0$ e $\lim\limits_{x \to -\infty} -e^x = 0$, então, pelo Teorema do Confronto temos $\lim\limits_{x \to -\infty} e^x \sin(x) = 0$


222   

Estime numericamente os seguintes limites para a função $f(x)= \frac{x^2+5 x-36}{x^3-5 x^2+3 x+9}$:

  1. $\lim\limits_{x \to 3^-} f(x)$

  2. $\lim\limits_{x \to 3^+} f(x)$

  3. $\lim\limits_{x \to 3} f(x)$



  1. \begin{tabular}{cc}

    $x$ & $f(x)$ \\ \hline

    $2.9$ & $-335.64$ \\

    $2.99$ & $-30350.6$ \\

    \end{tabular}

    A tabela parece indicar que $\lim\limits_{x\to3^-}f(x) =-\infty$.

  2. \begin{tabular}{cc}

    $x$ & $f(x)$ \\ \hline

    $ 3.1$ & $-265.61$ \\

    $3.01$ & $-29650.6$ \\

    \end{tabular}

    A tabela parece indicar que $\lim\limits_{x\to3^+}f(x) =-\infty$.

  3. Ao analisar as duas tabelas, parece que $\lim\limits_{x\to3}f(x) =-\infty$.


    662   

    Derive a função $f\left( x\right)  = \int_{x^{2}}^{e^{3x}}\cos t\sin tdt$.


    83   

    Calcule os limites:

    1. $\lim\limits_{x\to\pi} \frac{3x+1}{1-x}$

    2. $\lim\limits_{x\to\pi} \frac{x^2+3x+5}{5x^2-2x-3}$

    3. $\lim\limits_{x\to\pi} \left(\frac{x-3}{x-5}\right)^7$


    1115   

    Encontre $f(x)$ que satisfaça o seguinte problema de valor inicial:
      $f''(x) = 24x^2+2^x-\cos x$ e $f'(0)= 5$, $f(0) = 0$


    $\frac{2 x^4 \ln ^2(2)+2^x+x \ln 2) (\ln 32-1)+\ln
         ^2(2) \cos (x)-1-\ln ^2(2)}{\ln ^2(2)}$


    1528   

    Conforme $x$ aumenta, tanto $1/x$ quanto $1/(ln\ x)$ tendem a zero. Dada a função: $f(x)=\left(\frac{1}{x}\right)^{1/(ln\ x)}$ avalie $f(x)$ para valores cada vez maiores de $x$. Qual o padrão observado? Com o auxílio de recursos computacionais, observe o gráfico de $f(x)$ para valores grandes de $x$.

    Sugestão: Procure, no site, o exercício 1527. Compare os resultados obtidos.


    828   

    Calcule $f^{\prime }\left( x\right)$:

    $f\left( x\right) =\log _{a}x,\;a>0$ e $a\neq 1$.


    $f'(x)=\dfrac{1}{xln(a)}


    1526   

    É verdade que, ao se esticar um elástico puxando-o por suas extremidades em direções opostas, algum ponto do elástico permanecerá em sua posição inicial? Justifique sua resposta.


    1278   

    Esboce o gráfico da função $f\left(  x\right)  =\frac{2x^{2}}{3x^{2}-3}$ . Para fazê-lo, determine:

    1. Domínio da função

    2. Zeros e inteceptos

    3. Simetrias

    4. Assíntotas horizontais e verticais

    5. Intervalos de crescimento e decrescimento

    6. Pontos de máximo e mínimo

    7. Concavidade

    8. Pontos de inflexão



    1. Dom$f=\left\{  x\in\mathbb{R}|x\neq\pm1\right\}  $

    2. $f\left(  x\right)  =0$ se, e somente se, $x=0$

    3. A função é par: $f\left(  -x\right)  =f\left(  x\right)  $

    4. Usando L'Hopital ou colocando-se $x^{2}$ em evidêncai no numerador e
        denominador, obtemos que
        \[
        \lim_{x\rightarrow\infty}f\left(  x\right)  =\lim_{x\rightarrow-\infty
        }f\left(  x\right)  =2/3
        \]
        \begin{align*}
        \lim_{x\rightarrow-1^{+}}f\left(  x\right)    & =-\infty\\
        \lim_{x\rightarrow-1^{-}}f\left(  x\right)    & =\infty\\
        \lim_{x\rightarrow1^{+}}f\left(  x\right)    & =\infty\\
        \lim_{x\rightarrow1^{-}}f\left(  x\right)    & =-\infty
        \end{align*}

    5. \begin{align*}
        f^{\prime}\left(  x\right)    & =\frac{4x\left(  3x^{2}-3\right)
        -2x^{2}\left(  6x\right)  }{\left(  3x^{2}-3\right)  ^{2}}\\
        & =\frac{-12x}{\left(  3x^{2}-3\right)  ^{2}}
        \end{align*}
        logo a derivada é positiva se $x<0$ e negativa se $x>0$, ou seja $f$ é crescente para $x<0$ e decrescente para $x>0$

    6. $x=0$ é ponto de máximo da função.

    7. A função não tem pontos de inflexão pois $\pm1\notin
        Dom\left(  f\right)  $
        \begin{align*}
        f"\left(  x\right)    & =\frac{-12\left(  3x^{2}-3\right)  ^{2}+12x2\left(
        3x^{2}-3\right)  6x}{\left(  3x^{2}-3\right)  ^{4}}\\
        & =\frac{-12\left(  3x^{2}-3\right)  +12x2\cdot6x}{\left(  3x^{2}-3\right)
        ^{3}}\\
        & =\frac{-36x^{2}+36+12^{2}x^{2}}{\left(  3x^{2}-3\right)  ^{3}}\\
        & =\frac{-12x^{2}+12+48x^{2}}{\left(  x^{2}-1\right)  ^{3}}\\
        & =12\frac{3x^{2}+1}{\left(  x^{2}-1\right)  ^{3}}
        \end{align*}
        Observando que $3x^{2}+1>0,\forall x$, temos que $f"\left(  x\right)  >0$ se, e somente se,
        $x^{2}-1>0$ se, e somente se, $x>1$ ou $x<-1$ logo $f$ tem concavidade para cima se
        $x\in(-\infty,-1)\cup\left(  1,\infty\right)  $ e concavidade par baixo se
        $x\in\left(  -1,1\right)  $.

    8. Esboço do Gráfico:

      Construa o gráfico da função 1 e da função 2 usando um único plano cartesiano disponível a seguir


    1305   

    Resolva os itens:

    1. Mostre que $\lim\limits_{x\rightarrow 0^{+}}\left( x\ln x\right) =0$;
    2. Utilize o item anterior para avaliar $\lim\limits_{x\rightarrow 0^{+}}x^{x}.$


    610   

    Mostre que $\sqrt{xy}\leq {\frac{x+y}{2}}$$\forall x,y\geq 0$.



    Elevando ambos os membros da expressão $\sqrt{xy}\leq \dfrac{x+y}{2}$ ao quadrado obtemos $xy\leq \dfrac{x^2+2xy+y^2}{4}$. Simplificando chegamos a $x^2-2xy+y^2=(x-y)^2 \geq0$. Como a última expressão obtida é verdadeira e todas as expressões são equivalentes entre si, segue o resultado. Esse resultado diz que a média geométrica entre dois dois números reais positivos é sempre menor que, ou igual, à média aritmética entre eles.


    931   

    Esboce o gráfico das funções $f(x) = \log_2 x $ e $ f(x) = \log_\frac{1}{2} x$ num mesmo sistema cartesiano. Qual relação você observa entre os gráficos? Explique.


    527   

    A base $x$ e a altura $y$ de um retângulo estão variando com o tempo. Em um dado instante, $x$ mede $3 cm$  e cresce a uma taxa de $2 cm/s$, enquanto $y$ mede $4 cm$ e decresce a uma taxa de $1 cm/s$. Determine, nesse instante, a taxa de variação da área $A$ do retângulo em relação ao tempo.


    1249   

    Seja $f\left( x\right) =\dfrac{x^{3}}{x^{2}-1}$.

    1.   Encontre o domínio de $f$, os pontos de intersecção do gráfico de $f$ com os eixos, o sinal de $f$ e analise a simetria de $f$.

    2.   Caso existam, determine as assíntotas horizontais, verticais e oblíquas de $f$.

    3.   Determine os intervalos de crescimento e decrescimento de $f$, seus pontos de máximo e mínimo locais.

    4.   Determine os intervalos onde $f$ tem concavidade para cima e para baixo e os pontos de inflexão.

    5.   Esboce o gráfico de $f$ usando as informações obtidas nos itens anteriores.


      Considere $\frac{d}{dx}\left( \frac{x^{3}}{x^{2}-1}\right) =x^{2}\frac{x^{2}-3}{\left( x^{2}-1\right) ^{2}}$ e $\frac{d^{2}}{dx^{2}}\left(\frac{x^{3}}{x^{2}-1}\right) =2x\frac{x^{2}+3}{\left( x^{2}-1\right) ^{3}}$


    1315   

    Um retângulo tem sua base no eixo $x$ e seus dois vértices superiores na parábola $y=-x^2$. Qual é a maior área que esse retângulo pode ter? Quais são suas dimensões?


    957   

    Calcule o limite, caso exista:

    $\displaystyle \lim_{x\rightarrow \infty}\left( x-\sqrt{x^2 + 4x} \right) $


    1698   

    Determine a área da superfície gerada pela rotação da curva a seguir em torno do eixo indicado.

    $y=\sqrt{2x-x^2}$,  $0,5\leq x \leq 1,5$, eixo $x$


    741   

    Encontre o volume do sólido obtido pela rotação da região limitada pela curva dada em torno do eixo especificado. Esboce a região e o sólido.
      $y=x^{2}-6x+10,y=-x^{2}+6x-6$, ao redor do eixo $y.$


    592   

    Encontre os valores de $x$ para os quais cada o número $\sqrt{7x+9}$ é real.



    Este número será real se o valor dentro da raiz for maior ou igual a zero.
    $\begin{array}{rcl} 7x+9 &\geq& 0 \\ 7x &\geq& -9 \\ x &\geq& -\dfrac{9}{7}. \end{array}$
    Portanto o conjunto dos valores de $x$ tais que $\sqrt{7x+9}$ é real é $\{x \in \mathbb{R} ; x \geq -9/7\}$.


    1628   

    Prove que $\displaystyle\int x^me^xdx=x^me^x-m \displaystyle\int x^{m-1}e^xdx$, para $m$ inteiro positivo.


    149   

    Seja $f: \mathbb{R} \to \mathbb{R}$ uma função contínua no intervalo $\left[2,6 \right]$ com $f(2)=3$ e $f(6)=5$. Use o Teorema de Weierstrass e o Teorema do Valor Intermediário pra mostrar que a imagem de $f$ é um intervalo fechado.


    1294   

    Encontre a área da região limitada pela elipse $x^2+\frac{y^2}{4}=1.$



    Primeiramente, escreve-se a equação da elipse na forma $y=\pm f(x)$: 

    $y=\pm 2\sqrt{1-x^2}$

    Observação: Nesta forma, é possível ver mais facilmente que a elipse não apresenta nenhum ponto com $\left\vert x\right\vert >1$.

    Se denotarmos $f_1(x)= 2\sqrt{1-x^2}$ e $f_2(x)=- 2\sqrt{1-x^2} $, a área da região limitada pela elipse é portanto 

    $\int_{-1}^{1}f_1(x)-f_2(x)\,dx=2\int_{-1}^{1}f_1(x)\,dx=2 \left(\sqrt{1-x^2} x+\sin ^{-1}(x)\right)=2\pi$


    1102   

    Avalie a seguinte integral indefinida:
      $\int (t^2+3)(t^3-2t)\  dt$


    $t^6/6+t^4/4-3t^2+C$


    123   

    Classifique a veracidade das afirmações a seguir

    1. Se $f$ é contínua em $[0,1)$ e $[1,2)$, então $f$ é contínua em $[0,2)$.
    2. A soma de funções contínuas também é contínua
    3. Se $f$ é contínua em $[a,b]$, então $\lim_{x\to a^-}f(x) = f(a)$.


    1. Falso
    2. Verdadeiro
    3. Falso


    785   

    Mostre que qualquer reta tangente ao gráfico da hipérbole $xy=a^2$ determina com as assíntotas um triângulo de área igual a $2a^2$.


    1292   

    Encontre a área da região no primeiro quadrante limitada pelos eixos coordenados e pela curva $y=\frac{\sqrt{9-x^2}}{3}.$


    932   

    Seja $a>0$. Esboce o gráfico das funções $f(x) = \log_a x $ e $ f(x) = \log_\frac{1}{a} x$ num mesmo sistema cartesiano. Qual relação você observa entre os gráficos? Explique.


    177   

    Considere o número inteiro $P = 100 \cdot 101 \cdot 102 \cdot \ldots \cdot 200$, produto de $101$ números inteiros sucessivos. Ao escrever-se $P$ como um produto de fatores primos, qual o número de vezes que o fator $7$ aparece?


    1522   

    Seja $f$ uma função definida num intervalo aberto $I$ e $p \in I$. Suponha que $f(x) \leq f(p)$ para todo $x \in I$. Prove que $\lim\limits_{x \to p}\dfrac{f(x)-f(p)}{x-p}=0$, desde que o limite exista.


    1916   

    Em $1635$, Bonaventura Cavalieri, um aluno de Galileu, estabeleceu o seguinte resultado, chamado Princípio de Cavalieiri: se dois sólidos tiverem a mesma altura, e se as áreas de suas seções transversais tomadas paralelas e a iguais distâncias de suas bases forem sempre iguais, então os sólidos têm o mesmo volume. Use esse resultado para achar o volume do cilindro oblíquo da figura.


    1827   

    As curvas de crescimento logístico modelam a taxa de crescimento de uma certa população em função dos fatores ambientais. Em um período prolongado de tempo, a população tende a um valor limite que representa o máximo número de indivíduos que o espaço ou alimento pode sustentar. Estas curvas são da forma $$ y(t)=\dfrac{L}{1+Ae^{-kt}}, $$ onde $y$ é a população no momento $t$ ($t\geq 0$) e $A$, $k$ e $L$ são parâmetros positivos. Suponha que uma população $y$ cresce de acordo com o modelo logístico acima.

    1.  Qual é a taxa de crescimento de $y$ em $t=0$?

    2.  Descreva como a taxa de crescimento de $y$ varia com o tempo.

    3.  Em que momento a população cresce mais rapidamente?


    743   

    Encontre o volume do sólido obtido pela rotação da região limitada por $y=x-x^{2}$ e $y=0$ ao redor da reta $x=2$.


    145   

    Para a função a seguir, dê os intervalos nos quais ela é contínua:

     $ f(k) = \sqrt{1-e^k}$.


    $(-\infty,0]$


    189   

    Determine qual o último número $N$, escrito na sucessão dos números naturais $12345678910111213...N$, sabendo que foram escritos $3849$ algarismos.


    1030   

    Utilizando o gráfico, avalie os seguintes limites para a  função

    $ f(x) = \frac{1}{(x-3)(x-5)^2}$.

    Construa o gráfico da função 1 e da função 2 usando um único plano cartesiano disponível a seguir

    1. $ \lim\limits_{x\to 3^-} f(x)$

    2. $ \lim\limits_{x\to 3^+} f(x)$

    3. $ \lim\limits_{x\to 3} f(x)$

    4. $ \lim\limits_{x\to 5^-} f(x)$

    5. $ \lim\limits_{x\to 5^+} f(x)$

    6. $ \lim\limits_{x\to 5} f(x)$


    1. $-\infty$
    2. $\infty$
    3. O limite não existe
    4. $\infty$
    5. $\infty$
    6. $\infty$


    884   

    Para quaisquer $x,y\in \mathbb{R},$ mostre que vale $|xy|=|x||y|.$


    1264   

    Calcule a seguinte integral:
       $\int_{0}^{\pi }x^{2}senx dx$.


    $\pi^2-4$


    1343   

    Obtenha as assíntotas verticais de $f(x)=\frac{x^2+1}{x}$.


    $x=0$.


    580   

    Esboce neste mesmo gráfico a reta $y=2x+3$. Indique a região delimitada por esta reta e pelo gráfico de $f\left(x\right) $, para $2\leq x\leq 3$. Calcule a área desta região.


    1748   

    1. Mostre que se $f''(a)$ existe, então $f''(a) = \displaystyle \lim_{h \to 0} \dfrac{f(a+h) - 2f(a) + f(a-h)}{h^2}.$ (Sugestão: use o polinômio de Taylor $P_{2,a}(x)$ com $x=a+h$ e com $x=a-h$).

    2. Conclua que $\dfrac{f(a+h) - 2f(a) + f(a-h)}{h^2}$ é uma boa aproximação para $f''(a)$, para $h$ pequeno.

    3. Sabendo que a posição de uma partícula em função do tempo $x(t)$ é tal que $x(0)=2$, $x(1)=4$ e $x(2)=5$, utilizando a fórmula acima obtenha uma aproximação para a aceleração da partícula entre os tempos $t=0$ e $t=2$. (Escolha apropriadamente os valores de $a$ e $h$).


    930   

    Calcule, apresentando todos os cálculos e/ou justificativas.

    1. $log_6 (36) +log_3 (6^4)$
    2. $8^{\frac {2} {3}}+\sqrt{log_2 (16)}+2^{2^3}+(2^2)^{3}$


    1718   

    1. Dê um exemplo de função contínua em seu domínio mas que não é diferenciável em algum(ns) ponto(s).

    2. Qual a relação entre a continuidade e a diferenciabilidade de uma função? Demonstre.


    670   

    Calcule a integral $\int e^{x}\sin xdx$.


    $\dfrac{1}{2}e^x(sinx-cosx)+C$.


    649   

    Verifique se as funções abaixo são pares, ímpares ou nenhuma das duas coisas.

    1. $f(x)=x^{3}+x$

    2. $f(x)=x^{4}+2x^{3}+x^{2}$



    1. $f(-x)=(-x)^{3}+(-x) = -x^3-x = -(x^3+x) = -f(x)$, logo a função é ímpar.
    1. $f(-x)=(-x)^{4}+2(-x)^{3}+(-x)^{2} = x^4-2x^3+x^2$, que não é igual a $f(x)$ nem $-f(x)$, logo a função não é par nem ímpar.


    835   

    Determine a derivada de $f\left( t\right) =t^{3}e^{-3t}$.


    $-3 e^{-3t} (t-1) t^2$.


    1525   

    Resolva os itens:

    1. Prove que existe $r>0$ tal que $\cos{x}-1<\dfrac{\sin{x}}{x}-1<0$ para $0<|x|<r$.
    2. Calcule $\lim\limits_{x \to 0}\dfrac{x-\sin{x}}{x^2}$.


    333   

    Encontre todas as funções polinomiais $f$ com coeficientes reais tais que $(x-27)f(3x)=27(x-1)f(x)$ para todo número real $x$.


    722   

    Lembrando que o comprimento do traçado de um gráfico de uma função $f(x)$ no intervalo $[a,b]$ é dado por $\int_a^b \sqrt{1+(f'(x))^2} dx$, calcule o comprimento da circunferência de raio $r=1$.


    210   

    Utilizando o gráfico a seguir, avalie os seguintes limites

    Construa o gráfico da função 1 e da função 2 usando um único plano cartesiano disponível a seguir

    1. $ \lim\limits_{x\to 1^-} f(x)$
    2. $ \lim\limits_{x\to 1^+} f(x)$
    3. $ \lim\limits_{x\to 1} f(x)$
    4. $f(1)$
    5. $ \lim\limits_{x\to 0^-} f(x)$
    6. $ \lim\limits_{x\to 0^+} f(x)$


    1. $2$
    2. $2$
    3. $2$
    4. $1$
    5. Como $f$ não é definida para $x<0$, esse limite é indefinido.
    6. $1$


    1626   

    A atmosfera da Terra absorve aproximadamente $32\%$ da radiação proveniente do Sol. A Terra também emite radiação (a maior parte em forma de calor) e a atmosfera absorve aproximadamente $93\%$ dessa radiação. A diferença entre a radiação que entra na Terra e a que sai é chamada efeito-estufa. Modificações nesse equilíbrio podem afetar o clima da Terra. Seja $I_0$ a intensidade da radiação do Sol e $I$ a intensidade depois de percorrer uma distância $x$ na atmosfera. Se $p(h)$ é a densidade da atmosfera na altitude $h$, então a espessura ótica é $f(x)=k \displaystyle\int_0^x p(h) dh$, onde $k$ é uma constante de absorção e $I$ é dada por $I=I_0e^{-f(x)}$. Mostre que $dI/dx=-kp(x)I$.


    843   

    Calcule a derivada da função:

    $y=\dfrac{x\tan 3x}{x^{2}+4}$.


    $y' =  -(2 x^2 \tan(3 x))/(x^2 + 4)^2 + (\tan(3 x))/(x^2 + 4) + (3 x \sec^2(3 x))/(x^2 + 4)$.


    1029   

    Utilizando o gráfico, avalie os seguintes limites para a  função

    Construa o gráfico da função 1 e da função 2 usando um único plano cartesiano disponível a seguir

    $f(x) = \frac{1}{(x+1)^2}$

    1. $ \lim\limits_{x\to -1^-} f(x)$

    2. $ \lim\limits_{x\to -1^+} f(x)$


    1. $\infty$
    2.  $\infty$


    1105   

    Avalie a seguinte integral indefinida:
      $\int e^\pi\  dx$


    $e^\pi x+C$


    692   

    Calcule o seguinte limite

    $\lim\limits_{x\rightarrow \infty }\left( \dfrac{x+2}{x+1}\right)^{x}$.


    $e$.


    100   

    Calcule e justifique os seguintes limites, quando existirem, ou justifique a inexistência:

    1. $\lim\limits_{x\rightarrow 1}\dfrac{f\left( x\right) -f\left(1\right) }{x-1}$, onde $f\left( x\right) =\left\{ \begin{array}{cc} x^{2} & \text{se }x\leq 1 \\ 2x-1 & \text{se }x>1 \end{array} \right. $
    2. $\lim\limits_{x\rightarrow 2}\dfrac{f\left( x\right) -f\left(2\right) }{x-2}$, onde $f\left( x\right) =\left\{ \begin{array}{cc} x & \text{se }x\geq 2 \\ x^{2}/2 & \text{se }x<2 \end{array} \right. $
    3. $\lim\limits_{h\rightarrow 0}\dfrac{f\left( x+h\right) -f\left( x\right) }{h}$, com $f\left( x\right) =x^{2}-3x$ e $f\left( x\right) =1/x$


    1909   

    A cápsula cônica de reentrada de um veículo espacial é desenhada de tal forma que uma secção transversal tomada $x$ pés da ponta e perpendicular ao eixo de simetria é um círculo de raio $\dfrac{1}{4}x^2$ pés. Ache o volume do cone sabendo que o seu comporimento é de $20$ pés.


    1790   

    Seja $f$ uma função definida em $\mathbb{R}$ e suponha que exista $M>0$ tal que $|f(x)-f(p)|\leq M|x-p|$ para todo $x$. Prove que $f$ é contínua em $p$.


    675   

    Calcule a integral $\int \dfrac{\sin x}{\cos ^{3}x}dx.$


    188   

    Encontre a fração geratriz das dízimas seguintes:

    1. 2,001111...     
    2. 2,1010101010...   
    3. 1,23333333...



    1. $\begin{array}{rcl} 2,001111... &=& 2 + 0,00111... \\ &=& 2 + \dfrac{0,111...}{100} \\ &=& 2 +   \dfrac{1}{100} \dfrac{1}{9} \\&=& 2 + \dfrac{1}{900} \\  &=& \dfrac{1800+1}{900} \\  &=& \dfrac{1801}{900}. \end{array} $
    2. $\begin{array}{rcl} 2,101010... &=& 2 + 0,101010... \\ &=& 2 + \dfrac{10}{99} \\ &=& \dfrac{198+10}{99} \\  &=& \dfrac{208}{99}. \end{array} $
    3. $\begin{array}{rcl} 1,2333... &=& 1,2 + 0,0333... \\ &=& \dfrac{12}{10} + \dfrac{1}{10}\dfrac{3}{9} \\ &=& \dfrac{12}{10} + \dfrac{3}{90} \\&=& \dfrac{108+3}{90} \\  &=& \dfrac{111}{90}. \end{array} $


    1920   

    Mostre que a a área lateral $S$ de um cone circular reto de altura $h$ e raio da base $r$ é $S=\pi r \sqrt{r^2+h^2}$.


    114   

    Dê um exemplo de uma função que seja contínua em todos os pontos da reta, exceto nos pontos da forma $k \pi$, $k \in \mathbb{Z}$.


    $f(x)=1$, se $x=k \pi$, $k \in \mathbb{Z}$; $f(x)=0$, caso contrário.


    1636   

    Calcule a integral $\displaystyle\int \dfrac{1}{ax^n+bx}dx$.


    1759   

    Sendo $n$ um número positivo, mostre que

    $$\displaystyle \int_{-1}^1 x^n \, dx = \left. \dfrac{x^{n+1}}{n+1} \right |_{-1}^1.$$

    Se $n$ for um número negativo diferente de $-1$, esta expressão continua válida?


    1274   

    Calcule a integral $\int{\frac{\sin 10x}{4+\cos 10x}dx}.$


    $\dfrac{1}{10}ln(cos(10x)+4)+C$


    573   

    Estude a função $f\left( x\right) =\sin x+\cos x$ com relação à concavidade, pontos de inflexão, máximos e mínimos, e esboce o seu gráfico.


    1094   

     \item Avalie a seguinte integral indefinida:
      $\int  dt$


      $t+C$


    126   

    Seja $f(x)= \left\{\begin{array}{ccc} x^2-5, & &\text{se } x<5 \\ 5x, & &\text{se } x \geq 5 \end{array}\right.$.
    Calcule:

    1. $ \lim\limits_{x\to 5^-} f(x)$
    2. $ \lim\limits_{x\to 5^+} f(x)$
    3. $ \lim\limits_{x\to 5} f(x)$
    4. $f(5)$
    5. $f$ é contínua em $x=5$?


    1. $20$.

    2. $25$.

    3. Não existe.

    4. $25$

    5. Não.


    945   

    Mostre, usando a definição de limite, que $\displaystyle \lim_{x\to 5} 3-x = -2$



    Seja $\epsilon >0$ dado. Queremos encontrar $\delta >0$ tal que, quando$|x-5|<\delta$, $|f(x)-(-2)|<\epsilon$.
    Considerando $|f(x)-(-2)|<\epsilon$:
    \begin{gather*}
    |f(x) + 2 | < \epsilon \\
    |(3-x) + 2 |<\epsilon \\
    | 5-x | < \epsilon \\
    -\epsilon < 5-x < \epsilon \\
    -\epsilon < x-5 < \epsilon. \\
    \end{gather*}
    Isso implica que podemos estabelecer $\delta =\epsilon$. Portanto:
    \begin{gather*}
    |x-5|<\delta \\
    -\delta < x-5 < \delta\\
    -\epsilon < x-5 < \epsilon\\
    -\epsilon < (x-3)-2 < \epsilon \\
    -\epsilon < (-x+3)-(-2) < \epsilon \\
    |3-x - (-2)| < \epsilon,
    \end{gather*}

    que é o que buscávamos provar.


    1197   

    Calcule a derivada da seguinte função:
     $f\left(  x\right)  =\frac{\cos^{2}\left(  x\right)  +\sin^{2}\left(x\right)  }{\sqrt{x^{3}+1}}.$


    139   

    Para a função a seguir, dê os intervalos nos quais ela é contínua:

     $ f(t) = \sqrt{5t^2-30}$.


     $(-\infty,-\sqrt{6}]\cup [\sqrt{6},\infty)$


    1592   

    A altura de um corpo em movimento vertical é dada por
    $s = -\frac{1}{2}gt^2+v_0t+s_0,\quad g>0$
    com $s$ em metros e $t$ em segundos. Determine a altura máxima do corpo em função da velocidade inicial $v_0$, da aceleração da gravidade $g$ e da posição inicial $s_0$.


    587   

    Mostre que $x^{2}-xy+y^{2}\geq 0$, $\forall x,y\in \mathbb{R}^+$ e que vale a  igualdade se e somente se $x=y=0$.



    Note que $x^{2}-xy+y^{2}=x^2-2xy+y^2+xy=(x-y)^2+xy \geq 0$, pois $(x-y)^2 \geq 0$ e $xy \geq 0$. O único modo de ocorrer a igualdade é quando as duas parcelas forem iguais a zero, o que ocorre se, e somente se, $x=y=0$.


    1783   

    Prove a seguinte generalização do Teorema do Valor Médio: Se $f$ é contínua e diferenciável sobre o intervalo $(a,b)$ e os limites $\displaystyle \lim_{y\to a^+}f(y)$ e $\displaystyle \lim_{y\to b^-}f(y)$ existem, então existe $x\in (a,b)$ tal que $$f'(x)=\dfrac{\displaystyle \lim_{y\to b^-}f(y)-\lim_{y\to a^+}f(y)}{b-a}.$$ (Sua prova deve começar mais ou menos assim: "Esta é uma conseqüência do Teorema do Valor Médio porque ...".)


    1612   

    Calcule o limite $\lim\limits_{x \to \infty}\dfrac{e^x}{x^n}$.


    $\infty$.


    733   

    Calcule a área no plano entre os gráficos de $f\left( x\right) =x^{3}-x$ e $g\left( x\right) =sen\left( \pi x\right) $ no intervalo $[0,1]$.



    Sabemos que, no intervalo $[0,1]$, $g(x)=\sin(\pi x)>0$. Uma análise das raízes de $f(x)=x^3-x$ nos mostra que no intervalo referido, $f(x)<0$. Assim,como não há mudança de sinal de $f(x)-g(x)$, o cálculo da área entre as curvas se resumo ao cálculo da integral definida

    $\int_0^1 \left(g(x)-f(x)\right)\,dx= \left.\left(-\frac{x^4}{4}+\frac{x^2}{2}-\frac{\cos (\pi  x)}{\pi }\right) \right\vert_0^1=\frac{1}{4}+\frac{2}{\pi }$


    901   

    Resolva a equação $x^4-13x^2 + 36 = 0$.



    Chamando $x^2=y$, transformamos a equação para:
    $y^2 -13y + 36=0$. Resolvendo esta equação:
    $\Delta = 13^2-4.1.36 = 25.$
    $y = \dfrac{13 \pm \sqrt{25}}{2}.$ 
    Assim as soluções são: $y = 9$ ou $y = 4$.
    Substituindo em $y=x^2$:
    $x^2 = 9 \Rightarrow x = \pm 3$.
    $x^2 = 4 \Rightarrow x = \pm 2$.
    Portanto as soluções são $x=-2$, $x=2$, $x=-3$ e $x=3$.


    1180   

    Para cada uma das afirmações abaixo, demonstre se verdadeiro
    ou dê contra-exemplo se for falso.

    1. $|x-y|\leq |x|+|y|,\forall x,y\in \mathbb{R}$.
    2. $x<y\Longrightarrow x^{2}<y^{2}$.
    3. $x<y\Longleftrightarrow 1/y<1/x$.


    840   

    Derive a função abaixo e avalie a derivada no ponto indicado:

    $f\left( x\right) =\dfrac{\ln \left( x^{2}\right) +5x^{3}}{1+\cos^{2}x};$ avaliar em $f\,^{\prime }\left( \pi /2\right) .$.


    $f'(x) = (15 x^2 + 2/x)/(\cos^2 x + 1) + (2 (5 x^3 + \log(x^2)) \sin x \cos x )/(\cos^2 x + 1)^2$.

    $f'(\pi/2) = \dfrac{4}{\pi} + \dfrac{15 \pi^2}{4}$.


    205   

    Utilizando o gráfico a seguir, avalie os seguintes limites

    Construa o gráfico da função 1 e da função 2 usando um único plano cartesiano disponível a seguir

    1. $ \lim\limits_{x\to 0^-} f(x)$
    2. $ \lim\limits_{x\to 0^+} f(x)$
    3. $ \lim\limits_{x\to 0} f(x)$
    4. $f(0)$


    1. $4$
    2. $-4$
    3. Não existe.
    4. $0$


    110   

    Mostre que a função $f\left( x\right) =\dfrac{1}{x}$ é contínua em seu domínio.


    1114   

    Encontre $f(\theta)$ que satisfaça o seguinte problema de valor inicial:
      $f''(\theta) = \sin \theta$ e $f'(\pi)= 2$, $f(\pi) = 4$


      $\theta-\sin (\theta)-\pi +4$


    1301   

    Esboce o gráfico da função $f\left(  x\right)  =\cos x-\sin x$ . Para fazê-lo, determine:

    1. Domínio da função

    2. Zeros e inteceptos

    3. Simetrias

    4. Assíntotas horizontais e verticais

    5. Intervalos de crescimento e decrescimento

    6. Pontos de máximo e mínimo

    7. Concavidade

    8. Pontos de inflexão



    1. Dom$\left(  f\right)  =\mathbb{R}$
    2. $f\left(  0\right)  =1$ e $f\left(  x\right)  =0$ se, e somente se, $\cos x=\sin x$ se, e somente se,
        $x=\frac{\pi}{4}+k\pi$ com $k\in\mathbb{Z}$
    3. $f$ é periódica, com período $2\pi$
    4. A função não possui assíntotas verticais (pois é contínua na reta) e tampouco horizontais (pois é periódica)
    5. \begin{align*}
        f^{\prime}\left(  x\right)    & =-\sin x-\cos x=0\text{ se, e somente se,}\\
        \cos x  & =-\sin x\text{ se, e somente se, }x=\frac{3\pi}{4}+k\pi\text{ com }k\in
        \mathbb{Z}\text{.}
        \end{align*}
        Considerando no período $\left[  0,2\pi\right]  $ temos que
        \begin{align*}
        f^{\prime}\left(  x\right)    & >0\text{ se }x\in\left(  \frac{3\pi}{4}
        ,\frac{7\pi}{4}\right)  \text{ (intervalo de crescimento)}\\
        f^{\prime}\left(  x\right)    & <0\text{ se }x\in\lbrack0,\frac{3\pi}{4}
        )\cup(\frac{7\pi}{4},2\pi]\text{ (intervalo de crescimento)}
        \end{align*}
    6. Novamente considerando no período $\left[  0,2\pi\right]  $ temos que $\frac{3\pi}{4}$ é ponto de mínimo e $\frac{7\pi}{4}$ é ponto de máximo.
    7. \begin{align*}
        f"\left(  x\right)    & =-\cos x+\sin x=0\text{ se, e somente se,}\\
        \cos x  & =\sin x\text{ se, e somente se, }x=\frac{\pi}{4}+k\pi\text{ com }k\in
        \mathbb{Z}\text{.}
        \end{align*}
        Considerando no período $\left[  0,2\pi\right]  $ temos que
        \begin{align*}
        f"\left(  x\right)    & >0\text{ se }x\in\left(  \frac{\pi}{4},\frac{5\pi}
        {4}\right)  \text{ (concavidade para cima)}\\
        f"\left(  x\right)    & <0\text{ se }x\in\lbrack0,\frac{\pi}{4})\cup
        (\frac{5\pi}{4},2\pi]\text{ (concavidade para baixo)}
        \end{align*}
    8. Esboço do Gráfico:
      Construa o gráfico da função 1 e da função 2 usando um único plano cartesiano disponível a seguir


    1753   

    1. Utilizando somas superiores, mostre que a área sob o gráfico de $y=x^2$ no intervalo $[0,b]$ é $b^3/3$.

    2. Mostre o mesmo resultado utilizando somas inferiores.


    1286   

    Seja $f:I \rightarrow \mathbb{R}$, contínua, onde I é um intervalo fechado qualquer. Prove que a imagem de $f$ é um intervalo fechado.


    213   

    Avalie os seguintes limites para a função definida por partes
    $ f(x) = \left\{\begin{array}{ccc}
    x^2, & &  \text{ se } x<2 \\
    x+1,  & &  \text{ se } x=2\\
    -x^2+2x+4, & & \text{ se }  x>2
    \end{array}
    \right.$

    1. $ \lim\limits_{x\to 2^-} f(x)$

    2. $ \lim\limits_{x\to 2^+} f(x)$

    3. $ \lim\limits_{x\to 2} f(x)$

    4. $f(2)$


    1. 4
    2. 4
    3. 4
    4. 3


    190   

    Quatro números inteiros positivos e distintos, $m, n, p$ e $q$, satisfazem a equação $(7-m)(7-n)(7-p)(7-q)=4$.

    Calcule a soma $m+n+p+q$.


    A única maneira de escrevermos $4$ como produto de inteiros positivos, a menos de ordem dos fatores,  é $4=1 \cdot 1\cdot 2 \cdot 2$. Assim, uma possibilidade é $m=n=6, p=q=5$. Há várias outras possibilidades, mas que não alterarão a soma $m+n+p+q=22$. De fato, se mudássemos os valores de $m,n,p$ e $q$, eles continuaríam sendo $1,1,2$ e $2$ em alguma ordem e a soma não mudaria, já que a adição é comutativa e associativa.


    1742   

    Escreva o número $\sin 2$ como uma soma (com a notação $\Sigma$), com um erro menor que $10^{-12}$.


    1128   

    Um objeto é lançado para cima com uma velocidade, em pés por segundo, dada por $v(t) = -32t+64$, de uma altura de $48$ pés.

    1. Qual a velocidade máxima do objeto?
    2. Qual o deslocamento máximo do objeto?
    3. Em que momento ocorre o maior deslocamento do objeto?
    4. Em que momento o objeto alcança a altura de $0$ pés?

     Dica: encontre o momento no qual o deslocamento é $-48ft$


    1. $64ft/s$
    2. $64ft$
    3. $t=2$
    4. $t=2+\sqrt{7}\approx 4.65s$.


    888   

    Determine o conjunto solução da equação $|x|^2-5|x|+6=0$.


    1716   

    Dizemos que duas famílias de curvas são trajetórias ortogonais uma da outra se cada curva de uma família for ortogonal a cada curva da outra. Faça um esboço de gráfico da família de curvas $x^2+(y-c)^2=c^2$ e da família $(x-k)^2+y^2=k^2$ no mesmo plano cartesiano, para alguns valores de $c$ e $k$ reais (se necessário, utilize algum recurso computacional). Mostre que estas famílias (de círculos) são ortogonais uma da outra. (Sugestão: retas tangentes são perpendiculares em um ponto de interseção se as suas inclinações são recíprocas negativas uma da outra.)


    77   

    Seja $ f(x)=\left\{\begin{array}{ll} \sqrt{x-4} &\text{ se } x>4\\8-2x&\text{ if } x<4\end{array}\right.$.

    Decida se $\lim\limits_{x\rightarrow 4}f(x)$ existe. Se o limite não existe, explique.


    804   

    Calcule $f^{\prime }\left( x\right)$:
    $f\left( x\right) =\dfrac{1+e^{x}}{1-e^{x}}$.


    $f'(x) = \dfrac{2 e^x}{(1-e^x)^2}$.



    Queremos calcular a derivada da divisão da função $1+e^x$ pela função $1-e^x$. Usando a regra da derivada do quociente, obtemos:

    \[\left( \dfrac{1+e^x}{1-e^x} \right)^\prime = \dfrac{(1+e^x)^\prime \cdot (1-e^x) - (1+e^x)\cdot (1-e^x)^\prime}{(1-e^x)^2}.\]

    Como

    \[(1+e^x)^\prime = (1)^\prime + (e^x)^\prime = 0 + e^x = e^x\]

    e, analogamente,

    \[(1-e^x)^\prime = -e^x,\]

    temos então que

    $\dfrac{(1+e^x)^\prime \cdot (1-e^x) - (1+e^x)\cdot (1-e^x)^\prime}{(1-e^x)^2} = \dfrac{e^x (1-e^x)-(1+e^x)(-e^x)}{(1-e^x)^2} = \dfrac{e^x(1-e^x)+e^x(1+e^x)}{(1-e^x)^2}$.

    Para simplificar o numerador, colocamos o fator comum $e^x$ em evidência: $e^x(1-e^x+1+e^x) = 2e^x$. Portanto, concluímos que

    \[f'(x) = \dfrac{2 e^x}{(1-e^x)^2}.\]


    1805   

    Dependendo da função e limites de integração, é possível transformar uma integral imprópria em uma integral ``própria'' com mesmo valor, por meio de uma substituição apropriada.

    1. Ilustre esse processo calculando a integral $\displaystyle \int_0^1 \sqrt{\dfrac{1+x}{1-x}} \, dx$ por meio da substituição $u=\sqrt{1-x}$.

    2. Tente calcular diretamente a integral (utilize algum recurso computacional se a integral estiver muito difícil). Compare os resultados obtidos.


    774   

    Determine uma reta que seja tangente à elipse $x^{2}+2y^{2}=9$ e que intecepte o eixo das ordenadas no ponto de ordenada $9/4$.


    338   

    Seja $P(x)$ um polinômio de coeficientes inteiros com grau $d>0$. Seja $n$ o número de inteiros distintos $k$ tais que $(P(k))^2=1$. Prove que $n\leq d+2.$


    783   

    Encontre a equação da reta tangente à curva $y=2x^2+3$ que seja paralela à reta $8x-y+3=0$.


    $y=8x+3$.


    71   

    Construa uma função com uma assíntota vertical em $x=5$ e uma assíntota horizontal em $y=5$.


    156   

     Seja $f:[a,b] \to [a,b]$ uma função contínua. Prove que $f$ possui um ponto fixo, ou seja, algum valor de $x$ tal que $f(x)=x$.


    1799   

    Um $n$-ágono regular é um polígono de $n$ lados que possui todos os lados iguais e todos os ângulos de mesma medida.

    1. Encontre o perímetro de um $n$-ágono regular inscrito num círculo de raio $r$.
    2. O perímetro do $n$-ágono se aproxima de algum valor quando $n$ cresce?
    3. Deduza a fórmula do comprimento de uma circunferência de raio $r$.


    1120   

    Com base no gráfico, avalie as seguintes integrais:

    Construa o gráfico da função 1 e da função 2 usando um único plano cartesiano disponível a seguir

    1. $\int_0^1 (-2x+4)\ dx$
    2. $\int_0^2 (-2x+4)\ dx$
    3. $\int_0^3 (-2x+4)\ dx$
    4.  $\int_1^3 (-2x+4)\ dx$
    5. $\int_2^4 (-2x+4)\ dx$
    6. $\int_0^1 (-6x+12)\ dx$


    1. 3
    2. 4
    3. 3
    4. 0
    5. $-4$
    6. 9


    1562   

    Seja $g(x)=log_a{x}$, em que $a>0$ e $a \neq 1$ é um real dado. Mostre que $g'(x)=\dfrac{1}{x \ln{a}}$.


    182   

    Se duas torneiras, de igual vazão, enchem uma piscina em $5$ horas,  quanto tempo três torneiras, de mesma vazão que as primeiras, encherão a piscina?



    Como duas torneiras de igual vazão enchem a piscina em $5$ horas,  uma única torneira encheria em $10$ horas. Ora, $3$ torneiras de igual vazão trabalhando juntas reduziriam esse tempo de $10$ horas dividindo-o por $3$. A resposta é $10/3$ horas.


    751   

    Calcule o seguinte limite:
    $\lim\limits_{x\rightarrow 1}\ln \dfrac{x^{2}-1}{x-1}$.


    $ln2$.


    1090   

    É mais correto se referir a uma antiderivada de $f(x)$ ou a antiderivada de $f(x)$?


    O correto é uma antiderivada, já que existem infinitas antiderivadas para uma dada função.


    1302   

    Determine os valores de $\lambda$ que tornam contínua a função $f:\mathbb{R\rightarrow R},$ da por:
      \[
      f\left( x\right) =\left\{
      \begin{array}{c}
      x^{2}+\lambda x\mbox{ se }x\leq 1 \\
      \left( \lambda x\right) ^{2}-1=\lambda ^{2}x^{2}-1\mbox{ se }x>1
      \end{array}
      \right. \mbox{.}
      \]


    1385   

    Uma cultura de bactérias cresce na taxa de $3e^{0,2t}$ por hora com $t$ em horas e $o\leq t\leq 20$.

    1. Quantas bactérias novas estarão na cultura depois de cinco horas?
    2. Quantas bactérias são introduzidas da sexta a décima quarta horas?
    3. Para que valor aproximado de $t$ a cultura conterá 150 bactérias novas?


    756   

    Dê um exemplo de uma função tal que $\lim\limits_{x\rightarrow p}\left| f\left( x\right) \right| $ exista mas $\lim\limits_{x\rightarrow p}f\left( x\right) $ não exista.


    599   

    Para cada uma das afirmações abaixo, demonstre-a, se verdadeira, ou dê um contra-exemplo, se for falsa.

    1. $|x-y|\leq |x|+|y|,\forall x,y\in \mathbb{R}$.

    2. $x<y\Longrightarrow x^{2}<y^{2}$.


    905   

    Resolva a equação modular $|x-1|-2|x-2| =-3$.


    607   

    Encontre todos os números reais que satisfazem cada uma das desigualdades abaixo. Dê o intervalo solução e ilustre a solução sobre a reta real.

    1. ${\frac{3}{x}}+{\frac{x-3}{x-1}}<{\frac{2}{x-1}}$

    2. ${\frac{1}{x}}+{\frac{3}{2x}}\geq 5$


    1666   

    Calcule a integral a seguir:

    $\int{\frac{\left(1+\sqrt{x}\right)^{1/3}}{\sqrt{x}}dx}$


    586   

    Encontre todos os números reais que satisfazem a cada uma das desigualdades abaixo. Dê o intervalo solução e ilustre a solução sobre a reta real.

    1. $7+|x|<{\frac{1}{x+2}}$

    2. $\left| {\frac{2x-3}{x+1}}\right| \leq {\frac{1}{2}}$


    1745   

    Escreva o número $e^2$ como uma soma (com a notação $\Sigma$), com um erro menor que $10^{-5}$.


    916   

    Determine o conjunto solução da equação $|x|^2+|x|-6=0$.


    1193   

    Calcule a derivada da seguinte função:
     $f\left(  x\right)  =5^{x\cos\left(  x^{2}\right)  }.$


    169   

    Seja $f$ uma função contínua em $[1,5]$, sendo que $f(1) = -2$ e $f(5) = -10$. Existe um valor $1<c<5$ tal que $f(c) = -9$? Por quê?


     Sim, pelo Teorema do Valor Intermediário.


    39   

    Calcule o limite $\lim\limits_{x\rightarrow -\infty }\left( 2^{x}+2^{-x}\right)$.


    $\infty$.


    81   

    Suponha que você tenha as seguintes informações sobre duas funções $f$ e $g$:

    1. $\lim\limits_{x\to 1} f(x) = 0$

    2. $\lim\limits_{x\to 1} g(x) = 0$

    3. $\lim\limits_{x\to 1} f(x)/g(x) = 2$

    O que você pode dizer sobre o valor de $\left|\frac{f(x)}{g(x)}\right|$ quando $x \approx 1$?


    1544   

    Determine a equação da reta tangente ao gráfico de $f(x)=tg{x}$ no ponto de abscissa $0$.


    $y=x$


    1797   

    Calcule a integral $\displaystyle \int \dfrac{10x^2+9x+1}{2x^3+3x^2+x} \, dx$.


    543   

    Seja $f\left( x\right) =x^{3}+3x.$

    1. Estude o sinal de $f^{\prime }(x).$

    2. Calcule $\lim\limits_{x\rightarrow +\infty }f\left(x\right) $ e $\lim\limits_{x\rightarrow -\infty }f\left( x\right) .$

    3. Utilizando as informações acima esboce o gráfico de $f\left( x\right) .$


    73   

    Identifique as assíntotas verticais e horizontais, caso existam, da função  $f(x)=\frac{-3 x^2-9 x-6}{5 x^2-10 x-15}$.


     Assíntota horizontal em $y=-3/5$; assíntota vertical em $x=3$.


    1588   

    Mostre que a linearização de $f(x)=(1+x)^k$ em $x=0$ é $L(x)=1+kx$.


    1252   

    Calcule a derivada de ordem $1000$ da função $f(x)=\sin{kx}, k \in R$.


    $f^{1000}(x)=k^{1000}\sin{kx}$


    793   

    Determine a equação da reta tangente em $\left( p,f\left(p\right) \right)$:

    $f\left( x\right) =\sqrt{x},\;p=9$.


    $y=\dfrac{x+9}{6}$.


    183   

    Existem, para doação a escolas, $2000$ ingressos de um espetáculo e $1575$ de outro. Cada escola deve receber ingressos para somente um dos espetáculos e todas as escolas devem receber a mesma quantidade de ingressos. Distribuindo-se todos os ingressos, qual o número mínimo de escolas que poderão ser contempladas nessa doação?


    669   

    Calcule a integral $\int_0^{1} xe^x dx$.


    1


    65   

     Sendo $f(x) = \left\{\begin{array}{cl} x^2-x+1 & x\leq 3 \\ 2x+1 & x>3 \end{array}\right.$, calcule $\lim\limits_{x\to 3} f(x)$.


    7


    1580   

    Dados $f(x) = x^{-1}$ e $x_0 = 0,9$, escolha um valor inteiro próximo a $x_0$ tal que $f(x_0)$ e $f'(x_0)$ sejam fáceis de calcular, e calcule uma linearização da função neste ponto.



    A linearização da função $f(x)$ em torno de um ponto $x_0$ nada mais é do que assumir que ela se comporta como uma reta que passa pelo ponto $(x_0,f(x_0))$ com inclinação $f'(x_0)$.

    Neste caso temos $f(x)=x^{-1}$ e $f'(x)=-x^{-2}$. Linearizando a função em torno de $1$, temos $\frac{y-f(1)}{x-1}=f'(1)=\frac{y-1}{x-1}= -1$ portanto temos

    $y=2-x$


    1129   

    Aproxime numericamente o seguinte limite
      $ f(x)=\frac{x^2-9 x+18}{x^2-x-6}$


    1.  \begin{array}{cc}
        x & f(x) \\ \hline
        2.9 & -0.632 \\
        2.99 & -0.6032 \\
        2.999 & -0.60032 \\
        \end{array}
      A tabela parece indicar que $\lim\limits_{x\to3^-}f(x) =-0.6$.
    2.  \begin{array}{cc}
        x & f(x) \\ \hline
         3.1 & -0.5686 \\
        3.01 & -0.5968 \\
        3.001 & -0.59968 \\
          \end{array}
      A tabela parece indicar que   $\lim\limits_{x\to3^+}f(x) =-0.6$.
    3. As tabelas parecem indicar que   $\lim\limits_{x\to3}f(x) =-0.6$.


    811   

    Calcule $f^{\prime }\left( x\right)$:

    $\dfrac{x+\sqrt[4]{x}}{x^{2}+3}$.


    $f'(x) = \dfrac{3-7x^2}{4 x^{3/4}(x^2+3)^2}$.


    1258   

    Calcule a seguinte integral:

       $\int \dfrac{x^{3}+x}{x-1}dx.$


    631   

    Determine $f\left(x\right)$ sabendo que: \begin{equation*} f^{\prime \prime }\left( x\right)  = \dfrac{1}{x^{2}}+8e^{2x}+2,\;f^{\prime }\left( 2\right) =4e^{4}\text{ e }f\left( 1\right) =2e^{2}\text{.} \end{equation*}


    1794   

    A área de um setor circular com raio $r$ e ângulo central $\theta$ é $A=\frac{1}{2} r^2 \theta$. Demonstre esta fórmula. (Dica: assuma um ângulo $0<\theta<\pi/2$ e considere o círculo centrado na origem, de forma que tenha equação $x^2+y^2=r^2$. Então $A$ é a soma da área de um triângulo e a porção restante do setor cirular. Faça um esboço do gráfico para facilitar.)


    345   

    Encontre as raízes do polinômio $x^4-6x^3+13x^2-12x+4.$
    Sugestão: Utilize o teste das raízes racionais


    1681   

    Calcule a seguinte integral:

    $\int_{0}^{\infty}{\frac{dx}{\sqrt{4-x}}}$


    Não converge.


    806   

    Calcule $f^{\prime }\left( x\right)$:

    $f\left( x\right) =e^{x}\sin x\cos x$.


    $f'(x) = \dfrac{1}{2} e^x ( \sin (2x) + 2 \cos (2x))$.


    943   

    Calcule, através da definição, o limite $ \lim_{x\to 2} 5 = 5$



    Seja $\epsilon >0$ dado. Queremos encontrar $\delta >0$ tal que, quando $|x-2|<\delta$, $|f(x)-5|<\epsilon$. Entretanto, como $f(x)=5$ é uma função constante, a segunda inequação é simplesmente $|5-5|<\epsilon$, o que é sempre verdade. Assim, pode-se escolher um $\delta$ qualquer; arbitrariamente, escolhe-se $\delta =\epsilon$.


    850   

    Calcule a derivada da função:

    $y=\dfrac{1}{2}\cot ^{2}5x+\ln \sin x.$


    $y'=\cot(x) - 5 \cot(5 x) \csc^2(5 x)$.


    1492   

    Dê o domínio e esboce o gráfico das seguintes funções:

    1.  $f(x)=1+1/x$
    2.  $f(x)=\dfrac{2}{x+1}$


    555   

    Determine os intervalos de decrescimento e crescimento e esboce o gráfico da seguinte função  $f\left( x\right) =x+\dfrac{1}{x^{2}}$.


    582   

    Encontre todos os números reais que satisfazem a cada uma das desigualdades abaixo. Dê o intervalo solução e ilustre a solução sobre a reta real.

    1. $|2x+1|\leq 1$

    2. $\left| {\frac{x}{x^{2}+1}}\right| \leq 1$


    749   

    Calcule o seguinte limite:
    $\lim\limits_{x\rightarrow 0}\dfrac{e^{x^{2}}-1}{x}$.


    $0$.


    112   

    Suponha que $\left| f\left( x\right) -f\left( 1\right) \right| \leq \left( x-1\right) ^{2}$. Demonstre que $f\left( x\right) $ é contínua em $1$.


    1321   

    Mostre que a funçao $y(x)$ com $y(0)=0$ que é definida implicitamente pela equaçao $y-x^{2}+y^{3}+xy^{2}+x^{2}y=0$ tem um extremo relativo no ponto $x=0$. Identifique esse extremo.


    179   

    Considere os números inteiros ``$abc$'' e ``$bac$'', em que $a$, $b$ e $c$ são algarismos distintos e diferentes de zero e $a>b$. A diferença $abc-bac$ é sempre um múltiplo de determinado número. Que número é esse?



    Note que "$abc$"$=100a+10b+c$ e que "$bac$"$=100b+10a+c$. Assim, "$abc$"-"$bac$"$=90a-90b=90(a-b)$, que é um número sempre múltiplo de $90$ e de todos os divisores de $90$.


    1325   

    Dentre todos os retângulos inscritos numa circunferência de raio $R$, quais as dimensões daquele que tem a maior área?


    627   

    Uma lata cilíndrica, sem tampa (mas com fundo), é feita para receber um volume de $900ml$ . Encontre as dimensoes que minimizarão o custo do metal para fazer a lata.


    134   

    Para a função a seguir, responda se a mesma é contínua nos pontos abaixo (e, caso não o seja, justifique)

      $ f(x) = \left\{\begin{array}{ccc}
    \frac{x^2+5x+4}{x^2+3x+2}, & &  \text{se } x\neq -1\\
    3, & &\text{se }  x=-1
    \end{array}\right.$

    1. $x=-1$
    2. $x=10$


    1. Sim.
    2. Sim.


    1727   

    Uma partícula se move na circunferência $x^2 + y^2 = a^2$ de tal modo que a componente $x$ de sua velocidade é $\dfrac{dx}{dt}=-y$. Encontre $\dfrac{dy}{dt}$ e determine se o sentido do movimento é horário ou anti-horário.


    1244   

    Calcule a derivada da seguinte função:
        $f\left(  x\right)  =\sin\left(  \arccos\left(  x\right)  \right)  .$


    1611   

    Calcule o limite $\lim\limits_{x \to \infty}\dfrac{\ln{x}}{cotg{x}}$.


    1574   

    Determine $f'$, $f''$ e $f'''$ sendo $f(x)=1/x$.


    $f'(x)=-\dfrac{1}{x^2}$, $f''(x)=\dfrac{2}{x^3}$ e $f'''(x)=-\dfrac{6}{x^4}$.


    651   

    Sejam $f(x)=\frac{x^{2}-25}{x^{2}-1}$ e $g(x)=\sqrt{x}$. Dê o domínio de cada uma das funções $f$, $g$, $f\circ g$ e $g\circ f$.


    1721   

    Imagine uma estrada em que o limite de velocidade é especificado a cada ponto dela. Isto é, existe uma certa função $L$ tal que o limite de velocidade no quilômetro $x$ da estrada é $L(x)$. Dois carros, $A$ e $B$, estão viajando nesta estrada; o carro $A$ com posição $a(t)$ e o $B$ com posição $b(t)$.

    1. Escreva uma equação para o fato de que o carro $A$ sempre anda no limite de velocidade. (A resposta não é $a'(t)=L(t)$.)

    2. Suponha que $A$ sempre ande no limite de velocidade, e que a posição de $B$ no tempo $t$ é a posição de $A$ no tempo $t-1$. Mostre que $B$ também anda no limite da velocidade em todo o tempo.

    3. Suponha agora que $B$ anda sempre a uma distância fixa atrás de $A$. Sobre quais condições $B$ sempre irá andar no limite de velocidade?


    1282   

    Calcule a integral $\int{ \frac {1}{x^2+3x-10} dx}.$


    1739   

    Escreva o polinômio $p(x)=x^2-4x-9$ em $x$ como um polinômio em $(x-3)$. (Só é necessário calcular o polinômio de Taylor em $3$, do mesmo grau do polinômio original. Por quê?)


    1312   

    Avalie o limite $\lim\limits_{x\rightarrow p}\ln\left(\dfrac{x^{n}-p^{n}}{x-p}\right)$, onde $n$ é qualquer número natural.


    1498   

    Esboce juntas as curvas dadas no plano cartesiano e identifique cada uma com sua equação:
    $y=2^x$, $y=4^x$,$y=3^{-x}$, e $y=\left( 1/2 \right)^{x}$.


    1508   

    Uma droga é administrada por via intravenosa para combater a dor. A função
    $f(t)=90-52\ ln(1+t), \quad 0 \leq t\leq4$ 
    fornece o número de unidades da droga que permanecem no corpo após $t$ horas.

    1. Qual foi o número inicial de unidades administradas?
    2. Quanto estará presente após $2$ horas?
    3. Esboce o gráfico de $f(t)$


    1619   

    Calcule o limite $\lim\limits_{x \to \infty}\dfrac{x+\cosh(x)}{x^2+1}$.


    $\infty$.


    539   

    Prove que a equação $x^3-4x+2=0$ tem exatamente três raízes reais distintas.



    Temos, primeiramente:

    $f'(x)=3x^2-4$

    $f''(x)=6x-4$

    É possível ver portanto que $f(x)$ tem dois pontos críticos: $x=\pm \frac{2}{\sqrt{3}}$. Como $f''(x)>0$ para $x>0$ e $f''(x)<0$ para $x<0$, $f(x)$ tem uma concavidade para baixo em $x=-\frac{2}{\sqrt{3}}$ e uma concavidade para cima em $x=\frac{2}{\sqrt{3}}$. 

    Temos que $f(0)=2$. Como na concavidade em $x=\frac{2}{\sqrt{3}}$ temos $f(x)<0$, sabemos que a primeira raiz está entre $0$ e $\frac{2}{\sqrt{3}}$. É fácil observar que $\lim\limits_{x \to \infty}f(x)=\infty$, o que nos mostra uma segunda raiz. Finalmente, como $\lim\limits_{x \to -\infty}f(x)=-\infty$, temos uma terceira raiz para algum valor $x<0$, provando então que a função em questão tem três raízes distintas.


    1182   

    Demonstre para todos números reais $a,b$ que $\max(a,b)=\frac{1}{2}(a+b)+\frac{1}{2}|b-a|,$ onde $\max(a,b)=a$ se $a\geq b$ e $\max(a,b)=b$ se $a<b$.


    1275   

    Calcule a integral $\int{\sin2\theta e^{\sin^2\theta}}d\theta.$


    $e^{\sin^2\theta}+C$


    1814   

    Use a derivada dada para encontrar as coordenadas $x$ de todos os pontos críticos de $f$ e classifique-os em máximo relativo, mínimo relativo ou nenhum dos dois.

    1.  $\displaystyle f'(x)=x^2(2x+1)(x-1)$;

    2.  $\displaystyle f'(x)=\dfrac{9-4x^2}{\sqrt[3]{x+1}}$.


    638   

    Seja $f\left( x\right) =\left| x\right| -x$. Mostre que $f\left( x\right) =0$ para $x\geq 0$ e $f\left( x\right) =-2x$ para $x<0$. Faça o gráfico dessa função.


    844   

    Calcule a derivada da função:

    $y=\ln \sqrt{\dfrac{1+\sin x}{1-\sin x}}$.


    $y' = \sec x$.


    1898   

    Prove que se $f$ é integrável em $\left[a,b\right]$ e $m \leq f(x) <M$ para todo $x$ em $\left[a,b\right]$, então $\displaystyle\int_{a}^{b}f(x)dx=(b-a)\mu$ para algum $\mu$ tal que $m < \mu <M$.


    88   

    Calcule o limite $\lim\limits_{x\to p}\frac{x^{4}-p^{4}}{x-p}$



    $\begin{array}{rcl} \lim\limits_{x\to p}\dfrac{x^{4}-p^{4}}{x-p} &=& \lim\limits_{x\to p} \dfrac{(x^2+p^2)(x^2-p^2)}{x-p} \\ &=& \lim\limits_{x\to p} \dfrac{(x^2+p^2)(x+p)(x-p)}{x-p} \\ &=& \lim\limits_{x\to p} (x^2+p^2)(x+p) \\ &=& (p^2 + p^2)(p+p) \\ &=& 4p^3. \end{array}$


    206   

    Utilizando o gráfico a seguir, avalie os seguintes limites

    Construa o gráfico da função 1 e da função 2 usando um único plano cartesiano disponível a seguir

    1. $ \lim\limits_{x\to 1^-} f(x)$
    2. $ \lim\limits_{x\to 1^+} f(x)$
    3. $ \lim\limits_{x\to 1} f(x)$
    4. $f(1)$


    1. $2$
    2. $2$
    3. $2$
    4. $2$


    340   

    Sejam $a_1,a_2,\ldots,a_{100}$, $b_1,b_2,\ldots,b_{100}$ números reais distintos. Uma tabela de dimensões $100\times 100$ é preenchida com esses números tal que o número $a_i+b_j$ é inserido na célula situada exatamente abaixo da interseção da $i$-ésima linha com a $j$-ésima coluna. Dado que em cada coluna o produto de todos os números é igual a $1$, prove que em cada linha o produto de todos os números é $-1$.


    1200   

    Uma partícula tem sua posição variando com o tempo de acordo com a relação $s(t)=-2\sin(t)+3\cos(t)$ , onde $s$ é dado em metros e $t$ em segundos.

    1. Encontre a velocidade da partícula no instante $t$.
    2. Encontre a velocidade da partícula no instante $t=3$ segundos.


    1. $v(t)=-3\sin(t)-2\cos(t)$.

    2. $v(3)=-3\sin(3)-2\cos(3)$.


    1617   

    Calcule o limite $\lim\limits_{x \to \infty}\dfrac{x \sin^{-1}(x)}{x- \sin(x)}$.


    $-\infty$.


    1663   

    Calcule a seguinte integral:

    $\int{e^{\sqrt{3x+9}}dx}$.


    1761   

    Prove que $\cosh^2(x)-\sinh^2(x)=1$.



    $\begin{array}{rcl} \cosh^2x - \sinh^2 x &=& \left(\dfrac{e^{-x} + e^x}{2}\right)^2 - \left(\dfrac{e^{x} - e^-x}{2}\right)^2 \\ &=& \dfrac{1}{4} (e^{-2x} + 2 e^{-x}e^x + e^{2x}) - \dfrac{1}{4} (e^{2x} - 2 e^xe^{-x} + e^{-2x}) \\ &=& \dfrac{1}{2} + \dfrac{1}{2} \\ &=& 1.\end{array}$


    59   

    Explique, usando propriedades de limites, porque $\lim\limits_{x\rightarrow 2}\frac{x^2-4}{x-2}\not = \frac{\lim\limits_{x\rightarrow 2} (x^2-4)}{\lim\limits_{x\rightarrow 2}(x-2)}$.

    Note que somente podemos usar as propriedades de limite quando um limite existe e é finito. Além disso, lembre-se que limites que recaem na expressão indeterminada "$\frac{0}{0}$", podem existir ou não. Calcule os seguintes limites.

    1. $\lim\limits_{x\rightarrow 2}\frac{x^2-4}{x-2}$

    2. $\lim\limits_{h\rightarrow 0}\frac{(2+h)^2 - 4}{h}$

    3. $\lim\limits_{x\rightarrow 2}\left(\frac{4}{x^2-2x}-\frac{x}{x-2}\right)$

    4.  $\lim\limits_{t\rightarrow 0}\frac{\sqrt{t^2+4}-2}{t^2}$


    898   

    Qual o conjunto solução da equação $|x-2|-|x-1|+|x+3|=0$?


    688   

    Calcule o limite justificando as passagens.

    $\lim\limits_{x\rightarrow 3^{+}}\dfrac{5}{x-3}$.


    46   

    Sabendo que $\lim\limits_{x\to2} f(x) = 3$ e $\lim\limits_{x\to2} g(x) = -1$, calcule os seguintes limites:

    1. $\lim\limits_{x\to2}(f+g)(x)$

    2. $\lim\limits_{x\to2}(fg)(x)$

    3. $\lim\limits_{x\to2}(f/g)(x)$

    4. $\lim\limits_{x\to2}f(x)^{g(x)}$


    1757   

    Mostre que $\displaystyle \int_0^x \dfrac{\sin t}{t+1} \, dt > 0$

    para todo $x>0$.


    212   

    Avalie os seguintes limites para a função definida por partes
    $ f(x) = \left\{\begin{array}{ccc}
    a(x-b)^2+c, & & \text{ se }  x<b \\
    a(x-b)+c, & & x \text{ se } \geq b
    \end{array}
    \right.,$
    sendo que $a$, $b$ e $c$ são números reais.

    1. $ \lim\limits_{x\to b^-} f(x)$
    2. $ \lim\limits_{x\to b^+} f(x)$
    3. $ \lim\limits_{x\to b} f(x)$
    4. $f(b)$


    1. $c$
    2. $c$
    3. $c$
    4. $c$


    1918   

    Use camadas cilíndricas para encontrar o volume do sólido resultante quando se faz girar a área entre as curvas $y=2x-1$, $y=-2x+3$ e $x=2$ em torno do eixo $y$.


    933   

    Calcule:

    1. $log_2 (8)$
    1. $log_3 (27)$



    1. $\log_2(8) = x$
      $2^x = 8$
      $2^x = 2^3$
      $x = 3$.
    2. $\log_3(27) = x$
      $3^x = 27$
      $3^x = 3^3$
      $x = 3$.


    1795   

    Considere um tanque de formato cilíndrico que é utilizado para armazenamento de produtos químicos líquidos, com diâmetro de $10$m. Sua posição é tal que suas seções transversais circulares são verticais. Se o produto químico ocupa o cilindro até $7$m de profundidade, qual porcentagem da capacidade total que está sendo utilizada?


    1701   

    1. Prove que $\lim_{x\to a}f(x)=l$ se, e somente se, $\lim_{x\to a}[f(x)-l]=0$. Sugestão: Primeiro, compreenda por qual razão a afirmação anterior é óbvia; então dê uma prova formal.

    2. Prove que $\lim_{x\to 0}f(x)=\lim_{x\to a}f(x-a)$.

    3. Prove que $\lim_{x\to 0}f(x)=\lim_{x\to 0}f(x^3)$.

    4. Dê um exemplo em que $\lim_{x\to 0}f(x^2)$ existe, mas $\lim_{x\to 0}f(x)$ não existe.


    221   

    Estime numericamente os seguintes limites para a função $f(x)=\frac{x^2-1}{x^2-x-6}$:

    1. $\lim\limits_{x \to 3^-} f(x)$

    2. $\lim\limits_{x \to 3^+} f(x)$

    3. $\lim\limits_{x \to 3} f(x)$



    1. \begin{tabular}{cc}

      $x$ & $f(x)$ \\ \hline

      $2.9$ & $-15.1224$ \\

      $2.99$ & $-159.12$ \\

      $2.999$ & $-1599.12$

      \end{tabular}

      A tabela parece indicar que $\lim\limits_{x\to3^-}f(x) =-\infty$.

    2. \begin{tabular}{cc}

      $x$ & $f(x)$ \\ \hline

      $ 3.1$ & $16.8824$ \\

      $3.01$ & $160.88$ \\

      $3.001$ & $1600.88$

      \end{tabular}

      A tabela parece indicar que $\lim\limits_{x\to3^+}f(x) =\infty$.

    3. Ao analisar as duas tabelas, parece que  $\lim\limits_{x\to3}f(x)$ não existe.


    545   

    Esboce o gráfico de $f(x)=x^3-6x^2 +9x+1$, indicando campo de definição, intervalos de crescimento e de decrescimento, assíntotas horizontais, verticiais e inclinadas (se houver), limites no infinito, extremos relativos, estudo da concavidade, pontos de inflexão e reta tangente à curva nos pontos de inflexão.


    589   

    Se $0<x<y$ prove que $\sqrt[3]{y-x}>\sqrt[3]{y}-\sqrt[3]{x}$.


    674   

    Calcule a integral $ \int_{4}^{9}{\sqrt{x}}{\sqrt{x}-1}dx$.


    829   

    Calcule $f^{\prime }\left( x\right)$:

    $f\left( x\right) =\dfrac{\ln x}{x}$.


    $f'(x)=1-ln(x)$.


    594   

    Prove que $\sqrt{6}$ é irracional.


    1263   

    Se $f(8)=12$, $f'(x)$ é contínua e ${ \int_1^8 f'(x)dx=30}$, determine o valor de $f(1)$.


    913   

    Determine o conjunto de todos os números reais para os quais a expressão $\frac{\sqrt{4-x^2}}{\sqrt[3]{x-1}}$ está definida.


    938   

    Calcule $f^{-1}$ para a função $f(x)=1+3x.$



    Seja $y = f(x)$. Então:
    $y = 1 + 3 x$.
    Isolando $x$:
    $3 x = y - 1$
    $x = \dfrac{y-1}{3}$.
    Logo:
    $f^{-1}(x) = \dfrac{x-1}{3}$.


    612   

    Esboce o gráfico da função abaixo e resolva a inequação:

    $f\left( x\right) =\left( 2x-3\right) \left( x^{2}+1\right) <0$.


    1168   

    Considere a função $f$ cuja derivada é $f'(x)=(x-1)^2(x+2)$.

    1. Quais são os pontos críticos de $f$?

    2. Em quais intervalos $f$ é crescente ou decrescente?

    3. Em quais pontos $f$ assume valores máximos e mínimos locais?


    962   

    Calcule e justifique os seguintes limites, quando existirem, ou justifique a inexistência:

    1. $\lim\limits_{x\rightarrow 0}x\sin \left( \dfrac{1}{x}\right)$
    2. $\lim\limits_{x\rightarrow p}\dfrac{\sin \left(x^{2}-p^{2}\right) }{x-p}$


    664   

    Derive a função $h\left( x\right)  = \int_{\cos 5x}^{x^{7/3}}e^{r}\left( r^{2}+1\right) dr$.


    150   

     Mostre que $f(x) = \cos x - \frac{x}{10}$ tem pelo menos dois zeros em $[0, 2\pi]$.


    1678   

    Em uma reação química de dois reagentes, a velocidade da reação depende, em geral, da concentração destes. Seja $a$ a quantidade do reagente $A$ e $b$ a quantidade do reagente $B$ em $t=0$, sendo $x$ a quantidade do produto no instante $t$, a velocidade de formação de $x$ pode ser dada pela equação diferencial

    $\frac{dx}{dt} = k(a-x)(b-x)$,
    sendo que $k$ é uma constante para a reação. Encontre $x(t)$ se:

    1. $a=b$

    2. $a \neq b$

    Em ambos os casos, considere $x(t=0)=0$.


    1271   

    Calcule a seguinte integral:
       $ \int_0^{2\sqrt{3}}\frac{x^3}{\sqrt{16-x^2}}dx$.


    529   

    Um balão está subindo verticalmente acima de uma estrada a uma velocidade constante de $1$ pé por segundo. Quando ele está a $65$ pés acima do solo, uma bicicleta que se desloca a uma velocidade constante de $17$ pés por segundo  passa por baixo dele. A que taxa a distância $s(t)$ entre a bicicleta e o balão aumentará três segundos depois?


    1863   

    Mostre que a função $y=f(x)$ definida por $f(x)=\left\{\begin{array}{ll}\sqrt{x-r}, & \text{se} x \geq r \\ -\sqrt{r-x}, & \text{se} x<r \end{array}\right.$ tem a propriedade que para todo número real $a$, se $x_1=r+a$,  então $x_2=r+a$ e, por outro lado, $x_1=r-a$, então $x_2=r+a$. 


    941   

    Calcule, por meio da definição, o limite $\lim_{x\to 2} x^3-1 = 7$.



    Considere $\epsilon >0$ arbitrário. Queremos encontrar $\delta >0$ tal que quando $|x-2|<\delta$, $|f(x)-7|<\epsilon$.
    Considere $|f(x)-7|<\epsilon$, lembrando que o objetivo é afirmar algo sobre $|x-2|$:
    \begin{gather*}
    |f(x) -7 | < \epsilon \\
    |x^3-1 -7 |<\epsilon \\
    | x^3-8 | < \epsilon \\
    | x-2 |\cdot|x^2+2x+4| < \epsilon \\
    | x-3 | < \epsilon/|x^2+2x+4| \\
    \end{gather*}
    Como $x$ está próximo de $2$, podemos considerar $1<x<3$. Portanto
    \begin{gather*}
    1^2+2\cdot1+4<x^2+2x+4<3^2+2\cdot3+4 \\
    7 < x^2+2x+4 < 19 \\
    \frac{1}{19} < \frac{1}{x^2+2x+4} < \frac{1}{7} \\
    \frac{\epsilon}{19} < \frac{\epsilon}{x^2+2x+4} < \frac{\epsilon}{7} \\
    \end{gather*}
    Seja $\delta =\frac{\epsilon}{19}$. Então:
    \begin{gather*}
    |x-2|<\delta \\
    |x-2| < \frac{\epsilon}{19}\\
    |x-2| < \frac{\epsilon}{x^2+2x+4}\\
    |x-2|\cdot|x^2+2x+4| < \frac{\epsilon}{x^2+2x+4}\cdot|x^2+2x+4|\\
    \end{gather*}
    Assumindo $x$ próximo de $2$, $x^2+2x+4$ é positivo e podemos eliminar o módulo do lado direito da equação.
    \begin{gather*}
    |x-2|\cdot|x^2+2x+4| < \frac{\epsilon}{x^2+2x+4}\cdot(x^2+2x+4)\\
    |x^3-8| < \epsilon\\
    |(x^3-1) - 7| < \epsilon,
    \end{gather*}

    que é o que desejávamos provar.


    613   

    Esboce o gráfico da função abaixo e resolva a inequação:

    $f\left( x\right) =\dfrac{\left( x-3\right) }{x^{2}+1}<0$.


    625   

    Uma página impressa deve ter $24$ $cm^{2}$ de área reservada à parte escrita, uma margem de $1,5 cm$ nas partes superior e inferior e uma margem de $1 cm$ nos lados. Discuta a existência das dimensões (e calcule quando existir) daquelas que tem área total máxima e área total mínima.


    526   

    Uma escada de $8 m$ está encostada em uma parede. Se a extremidade inferior da escada for afastada do pé da parede a uma velocidade constante de $2 m/s$, com que velocidade a extremidade superior estará descendo no instante em que a inferior estiver a $3 m$ da parede?


    1342   

    Obtenha as assíntotas verticais de $f(x)=\frac{x^2+1}{x-1}$.


    $x=1$.


    174   

    Três números naturais e múltiplos consecutivos de 5 são tais que o triplo do menor é igual ao dobro do maior. Dentre esses números, qual o maior?



    Note que todo múltiplo de $5$ pode ser escrito na forma $5n$, onde $n$ é algum número natural. Com essa ideia, podemos representar três múltiplos consecutivos de $5$ por: $5(n-1)$, $5n$ e $5(n+1)$. Como o triplo do menor é igual ao dobro do maior obtemos a equação $15(n-1)=10(n+1)$. Resolvendo essa equação encontramos $n=5$ e o maior número dentre os três é $5 \cdot 6=30$.


    106   

    Mostre, usando a definição, que a função $f\left( x\right) =ax+b$ é contínua em seu domínio.


    164   

    Uma importante aplicação do Teorema do Valor Intermediário é o Método da Bissecção.

    Suponha que estamos interessados em encontrar as raízes de uma função contínua $f(x)$. O Método da Bissecção é uma alternativa que pode resultar em boas aproximações para as raízes, após sucessivas aplicações do método.

    Para iniciar o método, precisamos encontrar dois valores $a$ e $b$ tais que $f(a) \cdot f(b) < 0$.

    Sem perda de generalidade, vamos assumir $f(a) < 0$, $f(b) > 0$ e $a<b$. O Teorema do Valor Intermediário afirma que existe um valor $c$ no intervalo $[a,b]$ tal que $f(c) = 0$. O teorema não afirma nada a respeito da localização de $c$ dentro do intervalo, apenas que ele existe.

    O Método da Bissecção é, portanto, uma maneira sistemática de obter este valor $c$. Seja $d=\frac{a+b}{2}$ o meio do intervalo. Existem três possibilidades:

    1. $f(d) = 0 $ - Por sorte, encontramos a raiz e não é necessário prosseguir com o método.
    2. $f(d) < 0$ - Como $f(b)>0$, sabemos que há uma raiz no intervalo $[d,b]$. Este intervalo tem metade do tamanho do intervalo original, então estamos mais próximos de obter uma boa aproximação para a raiz.
    3. $f(d) > 0$ - Como $f(a)<0$, sabemos que há uma raiz no intervalo $[a,d]$. Novamente, este intervalo tem metade do tamanho do intervalo original, então estamos mais próximos de obter uma boa aproximação para a raiz.

    O Método da Bissecção é a aplicação sucessiva dos passos descritos até que se esteja próximo o suficiente da raiz de $f(x)$ para a aplicação desejada. Nota-se que para o caso em que $f(a)>0$ e $f(b)<0$ o método ainda funciona, mas no caso 2 o intervalo escolhido seria $[a,d]$ e no caso e $[d,b]$ (por quê?).

    Utilize o Método da Bissecção para encontrar as raízes de $f(x) = x^2+2x-4$ no intervalo $[1,1.5]$.


    A raiz aproximada é $x=1.23$.

      Os intervalos utilizados são:

      $[1,1.5] \quad [1,1.25] \quad [1.125,1.25]$

      $[1.1875,1.25]\quad [1.21875,1.25]\quad [1.234375,1.25]$

      $[1.234375,1.2421875]\quad [1.234375,1.2382813]$.


    617   

    Determine o domínio da seguinte função:

    $f\left( x\right) =\sqrt[4]{\dfrac{x}{x+4}}$.


    $\left\{ x\geq 0\right\} \cup \left\{ x<-4\right\} $.


    1183   

    Prove que para todo $x>0$ vale $x+\frac{1}{x}\geq 2$. Para quais números $x>0$ vale a igualdade?


    160   

    Seja $f:\mathbb{R} \to \mathbb{R}$ uma função contínua tal que, para todo real x, tenhamos $f(f(f(x))) = x^2 + 1$. Prove que $f$  é par.


    1664   

    A velocidade de uma partícula que se move de um lado para o outro sobre uma reta é $v=ds/dt = 6\sin2t\ m/s$ para qualquer $t$. Se $s=0$ quando $t=0$, determine o valor de $s$ para $t=\pi/2\ s$.


    1569   

    Seja $f(x)=\left\{\begin{array}{ll}
    x+1, & \text{se } x<2 \\
    1, & \text{se } x \geq 2
    \end{array}\right.$

    1. $f$ é contínua em $2$. Por quê?
    2. $f$ é derivável em $2$. Por quê?


    1. Não.
    2. Não


    1322   

    Entre todos o cilindros retos que tem uma área total dada, ache o que tem volume máximo.


    1138   

    Escreva a taxa de crescimento de $y$ em termos das taxas de crescimento das variáveis $k$, $l$ e $m$ para os seguintes casos. Assuma $\beta$ como uma dada constante.

    1.    $y=(klm)^{\beta }$  
    2.    $y=(kl)^{\beta }(1/m)^{1-\beta }$


    759   

    Use o Teorema do Confronto para calcular $\lim\limits_{x\rightarrow0^{+}}\sqrt{x} \,e^{\sin\left(  \pi/x\right)  }\text{.}$
      Lembre-se de justificar porque o Teorema do Confronto pode ser útil.


    1901   

    A ciclóide é um caminho traçado por um ponto na borda de uma roda que gira ao longo de uma reta. Use as equações paramétricas de uma ciclóide para mostrar que o comprimento $L$ de um arco de uma ciclóide é dado pela integral $L=\displaystyle\int_{0}^{2\pi} \sqrt{2(1-\cos\theta)}d \theta$

    Construa o gráfico da função 1 e da função 2 usando um único plano cartesiano disponível a seguir


    341   

    Encontre todos os números naturais $k$ para os quais a seguinte afirmação é verdadeira: Se $F(x)$ é um polinômio com coeficientes inteiros que satisfaz $0\leq F(c)\leq k$ para todo $c\in\{0,1,\ldots,k+1\}$ então $F(0)=F(1)=\cdots=F(k+1).$


    1538   

    Suponha que em uma máquina um pistão se desloque verticalmente tal que sua posição no instante $t$ (medido em segundos) seja dado por:

    $$s=A \cos(2 \pi b t ),$$

    onde $A>0$ é a amplitude do movimento, e $b>0$ é a frequência (número de vezes que o pistão se desloca de cima para baixo por segundo). Qual o efeito da duplicação da frequência sobre a velocidade, a aceleração e a sobreaceleração do pistão? Relacione a sua resposta com o fato de que uma máquina quebra quando funciona rápido demais.


    1693   

    Um projetista, incumbido da tarefa de projetar uma bacia com cerca de $3L$ de capacidade, resolveu fazê-la nos moldes de uma tampa de uma casca esférica de $r=16cm$, com $9cm$ de profundidade, conforme a figura abaixo. Calcule o volume da bacia projetada e veja se a estimativa do projetista foi adequada, dado que a margem de erro do volume estabelecida pela empresa era de $15\%$.

    Construa o gráfico da função 1 e da função 2 usando um único plano cartesiano disponível a seguir



    Podemos calcular o volume da bacia através da seguinte integral:

    $V=\int_{7}^{16}\pi\left(\sqrt{16^2-x^2}\right)^2\,dx=\left.\left[\pi(256x-\frac{x^3}{3})\right]\right\vert_7^{16}=1053\pi$

    Lembrando que $1L=1000cm^3$ e supondo $\pi\approx3$, temos $V=3159cm^3$ (O valor real é próximo de $V=3308cm^3$). Como a margem de erro do projetista era de $15\%$, vemos que este acertou em seus cálculos.


    935   

    A intensidade $I$ de um terremoto, medida na escala Richter, é um número que varia de $I=0$ até $I=8,9$ para o maior terremoto conhecido. $I$ é dado pela fórmula $I=\dfrac{2}{3} log {\left(\dfrac{E}{E_0}\right)}$, em que $E$ é a energia liberada pelo terremoto em quilowatt-hora e $E_0=7 \times 10^{-3}$ kwh.

    1. Qual a energia liberada por um terremoto de intensidade 8 na escala Richter?
    2. Aumentando uma unidade na intensidade do terremoto, por quanto fica multiplicada a energia liberada?


    1515   

    Determine $f$ de modo que $g(f(x))=x$ para todo $x \in D_f$, sendo $g$ dada por:

    1. $g(x)=\dfrac{1}{(x-2)^2}$
    2. $g(x)=\dfrac{1}{\sqrt{x}}$


    1201   

    Se a velocidade de um objeto em metros por segundo no instante $t$ segundos é $v(t)=-\sin(t)-\cos(t)$, qual a sua posição no instante $t=4$?


    $s(t)=\cos(4)-\sin(4)$


    176   

    O produto das idades de três amigos adolescentes (entre $12$ e $19$ anos) corresponde a $4080$ anos. Qual a soma das três idades, em anos?



    Decompondo o número $4080$ em fatores primos encontramos $4080=2^4 \cdot 15 \cdot 17=15 \cdot 16 \cdot 17$. Analisando essa decomposição, obtemos automaticamente que a única possibilidade que atende as exigências do enunciado é que as idades sejam $15,16$ e $17$ anos. A soma dessas idades é $15+16+17=48$ anos.


    940   

    Calcule, pela definição, o limite $ \lim_{x\to 0} \sin x= 0$ (Dica: use o fato que $|\sin x| \leq |x|$, sendo uma igualdade apenas para $x=0$.)



    Considere $\epsilon >0$ arbitrário. Queremos encontrar $\delta >0$ tal que quando $|x-0|<\delta$, $|f(x)-0|<\epsilon$. Em termos simples, queremos mostrar que quando $|x|<\delta$, $|\sin x| < \epsilon$.

    Considere $\delta = \epsilon$. Podemos presumir que $|x|<\delta$. Usando a dica do enunciado, temos que $|\sin x | < |x| < \delta = \epsilon$. Portanto, se $|x|<\delta$, sabemos imediatamente que $|\sin x| < \epsilon$.


    52   

    Construa os gráficos das funções indicadas e calcule os limites:

    1. $ f(x)=x^2$ quando $x\rightarrow\infty$

    2. $ h(x)=3x^5$  quando $x\rightarrow -\infty$

    3. $g(y)=\tan^{-1}(y)$ quando $y\rightarrow\infty$

    4. $f(x)=\frac{1}{x}$  quando $x\rightarrow -\infty$

    5. $f(x)=\frac{1}{x^7}$  quando $x\rightarrow \infty$

    6. $f(x)=\frac{1}{x^{-2}}$  quando $x\rightarrow \infty$


    518   

    Um recipiente cheio de água com a forma de um cone invertido está sendo esvaziado à razão de $6\,cm^3/min$. A altura do cone é $24cm$ e o raio da base é $12cm$. Encontre a velocidade com que baixa o nível da água quando está a $10cm$ do fundo.


    $\dfrac{dr}{dt}=-\dfrac{1}{25 \pi}$ cm/min


    797   

    Calcule $f'\left( x\right) $, pela definição:

    $f\left( x\right) =1/x$.


    $f'(x)=-\dfrac{1}{x^2}$.


    683   

    Calcule o limite:

    $\lim\limits_{x\rightarrow 1}\dfrac{\sqrt{x^{2}+3}-2}{x^{2}-1}$.


    1610   

    A velocidade, no tempo $t$, de um objeto de massa $m$ em queda é $v(t)=(mg/k)(1-e^{-(k/m)t})$, onde $k$ é uma constante e $g$ denota a força da gravidade. Calcule $\lim\limits_{m \to \infty}v(t)$ e conclua que $v(t)$ é aproximadamente proporcional ao tempo $t$ se a massa é muito grande.


    681   

    Calcule $\lim\limits_{x\rightarrow +\infty }\left( x-\sqrt{x^{2}+4x}\right)$.


    $-2$


    1093   

    Avalie a seguinte integral indefinida:
      $\int (10x^2-2)\ dx$


      $10/3x^3-2x+C$


    634   

    Determine $f\left(x\right)$ sabendo que: \begin{equation*} f\,^{\prime \prime }\left( x\right)  = \sin x-\cos x+x^{5},\;f\,^{\prime}\left( 0\right) =2\text{ e }f\left( 0\right) =0\text{ .} \end{equation*}


    1262   

    Use o Teorema Fundamental do Cálculo para calcular a derivada da função $g(x)={ \int_x^{x^2}\cos (t^2)dt}$.


    1382   

    Durante o primeiro mês de crescimento de produtos como milho, algodão e soja, a taxa de crescimento (em gramas/dia) é proporcional ao peso presente $W$. Para determinada espécie de algodão, $dW/dt=0,21W$. Preveja o peso de uma planta no término de um mês ($t=30$), se a planta pesa 70 miligramas no início do mês.


    1725   

    Substitua as interrogações por expressões envolvendo $\epsilon, x_0$ e $y_0$ de modo que a afirmação abaixo seja verdadeira. Se $y_0 \neq 0$, $|y-y_0|<??$ e $|x-x_0|<??$, então $y \neq 0$ e $\left| \dfrac {x}{y}-\dfrac{x_0}{y_0}\right|<\epsilon$.


    1326   

    O coeficiente angular da reta tangente, no ponto de abscissa x, ao gráfico de $y=f\left( x\right) $, é proporcional ao cubo da ordenada do ponto de tangência. Sabendo que $f\left( 0\right) =1$ e que $f\left(1\right) =1/\sqrt{2}$, determine $f$.


    1344   

    Verifique que a equação $x^{179}+\frac{163}{1+x^2+\sin^2x}=119$ possui pelo menos uma solução.


    1273   

    Calcule a seguinte integral:
        $\int{\frac{e^{2x}}{1+e^{2x}}dx}.$


    1771   

    Seja $P(x)$ um polinômio de qualquer grau. Mostre que:

    $$\displaystyle \int P(x) e^x \, dx = (P - P' + P'' -P''' + \ldots)e^x.$$


    1862   

    Use o método de Newton para calcular a raiz positiva de $x^2+x-1=0$ com duas casas decimais de precisão.


    1509   

    Utilizando a aproximação $ln\ 2 \approx 0,7$, pode-se derivar uma regra popular, conhecida como regra dos 70, que diz: "Para estimar quantos anos uma determinada quantia em dinheiro dobre ao ser investida a uma porcentagem $r$ composta continuamente, divida $r$ por $70$". Por exemplo, uma quantia em dinheiro investida a $7\%$ dobrará em cerca de $70/7=10$ anos. Se, em vez disso, você quiser que ela dobre em $5$ anos, deve investí-la a $70/5=14\%$. Mostre a dedução da regra dos 70.


    1533   

    Seja $g(x)=x^3+\dfrac{1}{x}$. Determine a equação da reta tangente ao gráfico de $g$ no ponto correspondente a $x=1$.


    $y=2x$.


    1169   

    Determine as abscissas dos pontos críticos das funções abaixo:

    1. $s(t) = 2t^3 + t^2-20t +4$

    2. $f(x) = 4x^3-5x^2-42x + 7$

    3. $g(w) = w^4-32w$


    64   

    Considere a função $f(x) = \left\{\begin{array}{cl} x+2 & x\leq 2 \\ 3x-5 & x>2 \end{array}\right.$. Mostre que $\lim\limits_{x\to 2} f(x)$ não existe.


    605   

    Encontre todos os números reais que satisfazem cada uma das desigualdades abaixo. Dê o intervalo solução e ilustre a solução sobre a reta real.

    1. $(x-1)^{2}<1-x$

    2. $(2x-1)^{15}\leq 0$


    1293   

    Seja $\mathcal{A}$ o subconjunto do plano limitado pelas retas $x=0$, $x=\frac{\pi}{2}$ e pelos gráficos de $y=\sin x$ e $y=\cos x$. Faça um esboço do conjunto $\mathcal{A}$ e calcule sua área. 


    54   

    Calcule os seguintes limites. Pode ser útil usar a relação de inversão que há em relação às funções logarítmicas e exponenciais (isto é, $\ln(x)=y \Leftrightarrow e^y=x$) e/ou gráficos.

    1. $\lim\limits_{x\rightarrow\infty}\log_3 x$

    2. $\lim\limits_{x\rightarrow 0^+}\ln x$

    3. $\lim\limits_{x\rightarrow -\infty}e^x$


    1337   

    Usando os limites fundamentais, encontre o limite $\lim\limits_{x\rightarrow0}\frac{sen(cosx)}{sec(x)}$.


    $\sin(1)$.


    1133   

    Seja $f(x)=2x^2-3$. Determine a equação da reta tangente ao gráfico de $f$ nos pontos:

    1. $(0,f(0))$
    2. $(2,f(2))$


    1241   

    Seja $x(t)$ a posição horizontal e $y(t)$ a posição vertical de um objeto no tempo $t$. Com $x(0)=y(0)=0$ e velocidade iniciais horizontal $v_x$ e vertical $v_y$, a   trajetória do objeto pode ser representada pelas equações $x(t)=v_xt$ e $y(t)=-5t^2+v_y t$. Suponha que o módulo da velocidade inicial seja igual a $1$.  Neste caso, o ângulo $\theta$ entre a linha horizontal (eixo $x$) e a tangente à parábola na origem $(0,0)$ satisfaz $v_x=\cos(\theta)$ e $v_y=\sin(\theta).$

    1. Use a identidade $\sin(\theta_1+\theta_2)=\sin(\theta_1)\cos(\theta_2)+\sin(\theta_2)\cos(\theta_1)$ para provar que $\cos(\theta)\sin(\theta)=\frac{1}{2}\sin(2\theta).$
    2. Para $v_x>0$, $v_y>0$, determine o tempo $t_f>0$ tal que $y(t_f)=0$. Escreva $t_f(\theta)$ como função de $\theta$ com domínio $]0,\frac{\pi}{2}[$.
    3. Definimos uma função $x_f$, também com dominio $]0,\frac{\pi}{2}[$, por $x_f(\theta)=x(t_f(\theta))$. Escreva $t_f(\theta)$ como função de $\theta$ e simplifique.
    4. Qual é a imagem de $x_f$?
    5. Quais são os ângulos $\theta\in\,]0,\frac{\pi}{2}[$ com valores $x_f(\theta)=\frac{\sqrt{3}}{20}$, $x_f(\theta)=\frac{1}{10}$ e $x_f(\theta)=\frac{1}{5}$?


    1778   

    Calcule a integral $\displaystyle \int \dfrac{dy}{\sqrt{y} e^{\sqrt{y}}}$.


    698   

    Calcule a integral $\int \dfrac{x+1}{x^{2}-3x+2}dx$.


    349   

    Seja $P(x)=a_0x^n+a_1x^{n-1}+\cdots+a_n$ um polinômio não nulo com coeficientes inteiros tal que $P(r)=P(s)=0$ para certos inteiros $r$ e $s$, com $0<r<s$. Prove que $a_k\leq -s$ para algum $k$.


    629   

    Dentre todos os retângulos de perímetro fixo $L$, determine aquele de maior área. Justifique a resposta.


    323   

    Classifique as afirmações em verdadeiras ou falsas:

    Mestre Florindo, raizeiro famoso, vende suas garrafas medicinais por 5 reais, na feira de Caruaru.

    1. Se ele vende $q$ unidades, então $R(q) = 5q$, que é a sua função receita.

    2. Se ele tem um custo em torno de $40\%$ de sua receita, o seu custo pode ser estimado pela equação $C(q) = 2q$.

    3. Se, além disso, o mestre gastou $R\$ 900,00$ em materiais para confecção do seu famoso produto, ele deverá vender $300$ garrafas para recuperar o seu custo total.

    4. O lucro do mestre é dado pela função afim $L(q)=5q-900$.


    1. Verdadeiro.
    2. Verdadeiro.
    3. Verdadeiro, pois o lucro total $L(q)$ é $L(q)=R(q)-C(q)-900=3q-900$ e temos que $T(q)=0$ se $q=300$.
    4. Falso, $L(q)=3q-900$.


    185   

    Prove que $\sqrt{3}$ é irracional.

    A teoria necessária para resolver esta questão pode não ser abordada em alguns cursos de Cálculo 1. Sendo, também pertinente, às disciplinas Teoria dos Números e Análise Real I.
    Para aprofundar seus conhecimentos, dentro do escopo de Cálculo 1, recomendamos a leitura do Cap. 1 de Guidorizzi, vol. 1 e /ou o Prólogo de Spivak (vide Bibliografia de Cálculo 1).


    1548   

    A posição $s$ de uma partícula em um instante $t \geq 0$, se deslocando em um movimento retilíneo, é dada por:

    $$s=10\cos(t+\pi/4).$$

    1. Encontre a posição inicial da partícula. Isto é, a posição em $t=0$.
    2. Quais são os pontos mais distantes da origem que a partícula pode alcançar? (à direita e à esquerda).
    3. Encontre a velocidade e a aceleração da partícula nos pontos do item anterior.
    4. Quando a partícula atinge a origem pela primeira vez? Encontre a velocidade, o módulo da velocidade e a aceleração neste instante.


    895   

    Resolva as equações:

    1. $|x-1|^2-2|x-1| =-1$
    1. $|x-10|-|x+10| =0$


    1540   

    Seja $f(x)=\sin{x}$. Calcule $f'(x)$ e $f'\left(\dfrac{\pi}{4}\right)$.


    $f'(x)=\cos(x)$ e $f'\left(\dfrac{\pi}{4}\right)=\dfrac{\sqrt{2}}{2}$.


    848   

    Calcule a derivada da função:


    $y=\dfrac{2\left( 4+3\sqrt[3]{x}\right) \left( 2-\sqrt[3]{x}\right)^{3/2}}{5}$.


    $y'=-\dfrac{\sqrt{2 - x^{1/3}}}{x^{1/3}}$.


    1199   

    Resolva a equação $\left| {\frac{3x+8}{2x-3}}\right| =4$.


    1710   

    1. Um tanque contém 5000 litros de água pura. Água salgada contendo $30$g de sal por litro de água é bombeada para dentro do tanque a uma taxa de $25$ L$/$min. Considerando o tempo $t$ em minutos, mostre que a concentração de sal $C$ em função de $t$ (em gramas por litro) é dada por:$$C(t) = \dfrac{30 t}{200+t}.$$

    2. O que acontece com a concentração para um tempo muito grande, isto é, para $t \to \infty$?


    1832   

    O Princípio de Fermat na óptica estabelece que a luz, viajando de um ponto para outro, segue aquele caminho  para o qual o tempo total de percurso é mínimo. Em um meio uniforme, os caminhos de "tempo mínimo" e de "menor distância" vêm a ser iguais; assim sendo, se não obstruída, a luz viaja em linha reta. Suponha que temos uma fonte de luz (ponto $A$), um espelho plano e um observador (ponto $B$) em um meio uniforme. Se um raio de luz deixa a fonte, bate num espelho e vai até o observador, então a sua trajetória consiste de dois segmentos de reta, conforme ilustrado na figura abaixo. De acordo com o princípio de Fermat, a trajetória é tal que o tempo gasto no percurso é mínimo ou, como o meio é uniforme, a trajetória será tal que a distância total percorrida de $A$ para $B$ é a menor possível. Supondo que o mínimo ocorre quando $\displaystyle dt/dx=0$, mostre que o raio de luz irá atingir o espelho em um ponto $P$, tal que o "ângulo de incidência" $\theta_1$ é igual ao "ângulo de reflexão" $\theta_2$.

    Construa o gráfico da função 1 e da função 2 usando um único plano cartesiano disponível a seguir


    343   

    Encontre todos os pares de inteiros $m,n\geq 3$ tais que existem infinitos inteiros positivos $a$ para os quais

    $\frac{a^m+a-1}{a^n+a^2-1}$ é um inteiro.


    1576   

    Determine $f'$, $f''$ e $f'''$ sendo $f(x)=x|x|$.


    682   

    Calcule o limite:

    $\lim\limits_{x\rightarrow p}\dfrac{tg(x-p)}{x^{2}-p^{2}}$.


    753   

      Responda os itens:

    1.   Dada $f:{\mathbb{R} \to \mathbb{R}}$, defina (em termos de $\varepsilon $  e $\delta $) $\lim\limits_{x\rightarrow p}f\left( x\right) =L.$ Ilustre elaborando um gráfico para uma função genérica.
    2.   Qual é a condição sobre esse limite para que a função seja contínua?


    1630   

    Prove que $\displaystyle\int (\ln(x))^m dx=x (\ln(x))^m -m \displaystyle\int (\ln(x))^{m-1}dx$, para $m$ inteiro positivo.


    1728   

    A naftalina pode ser utilizada como repelente de insetos, embora possa trazer malefícios à saúde. Este composto tem a capacidade de sublimar, isto é: passa do estado sólido diretamente para o gasoso. Se uma bolinha de naftalina evapora a uma taxa proporcional à área de sua superfície, mostre que o seu raio decresce a uma taxa constante.


    158   

    Sejam $f,g:\mathbb{R} \to \mathbb{R}$ funções contínuas tais que $f(a)<g(a)$ e $f(b)>g(b)$. Mostre que existe $c \in (a,b)$ tal que $f(c)=g(c)$.


    611   

    Suponha que $x$ e $y$ sejam notas de provas bimestrais. Mostre que a chamada {\it média geométrica} entre $x$ e $y$, dada por $\sqrt{xy}$, poderia, se adotada como critério avaliativo, prejudicar a nota final de alguns alunos, isto é, elaé menor que ou igual à chamada {\it média aritmética} entre $x$ e $y$, que é dada por $\frac{x+y}{2}.$



    Elevando ambos os membros da expressão $\sqrt{xy}\leq \dfrac{x+y}{2}$ ao quadrado obtemos $xy\leq \dfrac{x^2+2xy+y^2}{4}$. Simplificando chegamos a $x^2-2xy+y^2=(x-y)^2 \geq0$. Como a última expressão obtida é verdadeira e todas as expressões são equivalentes entre si, segue o resultado. Esse resultado diz que a média geométrica entre dois dois números reais positivos é sempre menor que, ou igual, à média aritmética entre eles. Sendo assim, se o professor adotar como critério de avaliação a média geométrica em vez da aritmética, ele pode prejudicar a nota final dos alunos que tivessem a nota $x$ diferente da $y$, pois quando $x=y$ as duas médias são iguais.


    807   

    Calcule $f^{\prime }\left( x\right)$:

    $\left( 1+\sqrt{x}\right) e^{x}\tan x$.


    $f'(x) = \left( 1+\sqrt{x}\right) e^{x}\tan x + \dfrac{e^x \tan x}{2 \sqrt{x}} + e^x(\sqrt{x} + 1) \sec^2 x$.


    1684   

    Calcule o valor de $p$ para a integral a seguir convergir:

    $\int_{2}^{\infty}{\frac{dx}{x\left(ln\ x\right)^p}}$


    1634   

    Prove que $\displaystyle\int \dfrac{1}{u\sqrt{a^2+u^2}}du=-\dfrac{1}{a}\ln \left|\dfrac{\sqrt{a^2+u^2}+a}{u}\right|+C$.


    754   

    É possível que uma função $f:{\mathbb{R} \to \mathbb{R}}$
      seja tal que $\lim\limits_{x\rightarrow 2^{+}}f\left( x\right) =\lim\limits_{x\rightarrow 2^{-}}f\left(x\right)$ e ao mesmo tempo não seja contínua em $2$? Justifique e/ou dê um exemplo.


    799   

    Calcule $f'\left( x\right) $, pela definição:

    $f\left( x\right) =1/x^{2}$.


    $f'(x) = -\dfrac{2}{x^3}$.


    650   

    Se $f(x+1)=\frac{x-1}{\pi -x}$, ache $f(x)$ e encontre o domínio de $f$.


    784   

    Consideremos a curva $y=-x^4 +2x^2+x$ e o ponto $P=(1,2)$ nessa curva. Verifique que a reta tangente a essa curva no ponto $P$ também é tangente à curva em outro ponto. Ache esse outro ponto.


    1658   

    O custo marginal da impressão de um pôster quando $x$ pôsteres são impressos é
    $\frac{dc}{dx}=\frac{1}{2\sqrt{x}}$ reais.

    Determine $c(100)-c(1)$, ou seja, a soma do custo dos pôsteres 2-100.


    640   

    Nos exercícios abaixo determine o domínio máximo de definição de cada uma das funções dadas.

    1. $y=\sqrt{x-2}$

    2. $y=\sqrt{2-x}$



    1. O domínio de $y$ é o conjunto de números reais em que o valor dentro da raiz é positivo. Calculando esses valores:
      $x-2 > 0 \Rightarrow x > 2$.
      Portanto o domínio de $y$ é: $\{x \in \mathbb{R}; x >2\}$.
    2. O domínio de $y$ é o conjunto de números reais em que o valor dentro da raiz é positivo. Calculando esses valores:
      $2-x > 0 \Rightarrow x < 2$.
      Portanto o domínio de $y$ é: $\{x \in \mathbb{R}; x <2\}$.


    31   

    Calcule o limite $\lim\limits_{x\rightarrow \infty }\dfrac{\sqrt[3]{3x^{3}+2x-1}}{\sqrt{x^{2}+x+4}}$.


      $\sqrt[3]{3}$


    576   

    Estude a função $f\left( x\right) =\sqrt[3]{x^{3}-x}$ com relação à concavidade, pontos de inflexão, máximos e mínimos, e esboce o seu gráfico.


    1517   

    Prove que a função $f(x)=\left\{\begin{array}{ll}
    x, & \text{se x é racional} \\
    -x, & \text{se x é irracional}  
    \end{array}\right.$ é contínua em $0$.


    1288   

    Considere:

    1.  Um cilindro $CI_r$ de raio $r$ e altura $r$.
    2.  Um cone  $CO_r$ de raio $r$ e altura $r$.
    3.  Uma pirâmide $P_r$ de base quadrada com diagonal de comprimento $2r$ e altura $r$.

    Para cada um destes três sólidos, expresse o volume em forma de integral e demonstre que a relação (proporção) entre estes volumes não depende do parâmetro $r$.


    211   

    Avalie os seguintes limites para a função definida por partes

    $ f(x) = \left\{\begin{array}{ccc}
    \frac{|x|}{x}, & & \text{ se } x\neq 0 \\
    0, & & \text{ se }  x=0
    \end{array}
    \right.$

    1. $ \lim\limits_{x\to 0^-} f(x)$
    2. $ \lim\limits_{x\to 0^+} f(x)$
    3. $ \lim\limits_{x\to 0} f(x)$
    4. $f(0)$


    1. $-1$
    2. $1$
    3. Não existe.
    4. $0$


    1287   

    Determine os intervalos para os quais a função
      \begin{equation*} f\left(  x\right)  =\left\{ \begin{array} [c]{c} x^{2}+1\text{ se }x\leq0\\ \cos x\text{ se }0<x<1\\ x^{2}+1\text{ se }1\leq x \end{array} \right. \end{equation*} é contínua. Justifique sua resposta.



    As funções $x^{2}+1$ e $\cos x$ são ambas contínuas e por isto $f\left(  x\right)  $ é contínua para todo $x\neq0,1$. É necessário verificar a continuidade nos pontos $x=0$ e $x=1$.

    Para $x=0$ temos que $\lim_{x\rightarrow0^{-}}f\left(x\right) =\lim_{x\rightarrow0^{-}}\left(  x^{2}+1\right)  =1$ e $\lim_{x\rightarrow0^{+}}f\left(  x\right)  =\lim_{x\rightarrow0^{+}}\cos x=1$, logo $f\left(  x\right)  $ é contínua em $x=0$, pois ambos oslimites laterais existem, são iguais e coincidem com o valor da função no ponto.

    Para $x=1$ temos que $\lim_{x\rightarrow1^{-}}f\left( x\right) =\lim_{x\rightarrow0^{-}}\cos x=\cos\left(  1\right)  $ e $\lim_{x\rightarrow0^{+}}f\left(  x\right)  =\lim_{x\rightarrow0^{+}}\left(x^{2}+1\right)  =2$, e como $\cos\left(  1\right)  \neq2$ temos que $f\left(x\right)  $ não é contínua em $x=1$, pois apesar dos limites laterais existirem estes são distintos.


    1629   

    Prove que $\displaystyle\int x^m \sin(x)dx=-x^m \cos(x)+m \displaystyle\int x^{m-1} \cos(x)dx$, para $m$ inteiro positivo.


    633   

    Determine $f\left(x\right)$ sabendo que: \begin{equation*} f\,^{\prime \prime }\left( x\right)  = 12\sin 2x+\cos 3x+1,\;f\,^{\prime}\left( 0\right) =1\text{ e }f\left( 0\right) =0\text{ .} \end{equation*}


    893   

    Enuncie e prove a desigualdade triangular envolvendo números reais.


    1807   

    Em matemática, a função piso, denotada por $\lfloor x\rfloor$, converte um número real ${\displaystyle x}$ no maior número inteiro menor ou igual a ${\displaystyle x}$ Essa função é importante em computação para truncamento ou arredondamento de números. Considere a função $f(x)=\lfloor 1/x\rfloor$, $x \neq 0$. Esboce o gráfico dessa função para $\dfrac{1}{4} \leq x \leq 2$ e também para $-2 \leq x \leq -\dfrac{1}{4}$. Como se comporta $f(x)$ quando $x$ tende a zero pelo lado direito? E pelo lado esquerdo? O limite $\lim\limits_{x \to 0}f(x)$ existe?


    224   

    Estime numericamente os seguintes limites para a função $f(x)=\frac{x^2-9 x+18}{x^2-x-6}$:

    1. $\lim\limits_{x \to 3^-} f(x)$

    2. $\lim\limits_{x \to 3^+} f(x)$

    3. $\lim\limits_{x \to 3} f(x)$


    1. \begin{tabular}{cc}

      $x$ & $f(x)$ \\ \hline

      $2.9$ & $-0.632$ \\

      $2.99$ & $-0.6032$ \\

      $2.999$ & $-0.60032$ \\

      \end{tabular}

      A tabela parece indicar que $\lim\limits_{x\to3^-}f(x) =-0.6$.

    2. \begin{tabular}{cc}

      $x$ & $f(x)$ \\ \hline

      $ 3.1$ & $-0.5686$ \\

      $3.01$ & $-0.5968$ \\

      $3.001$ & $-0.59968$ \\

      \end{tabular}

      A tabela parece indicar que $\lim\limits_{x\to3^+}f(x) =-0.6$.

    3. Ao analisar as duas tabelas, parece que $\lim\limits_{x\to3}f(x) =-0.6$.


    1669   

    Calcule a integral a seguir:

    $\int{\cos^3 x\sin\ x dx}$


    $\dfrac{1}{4}cos^4(x)+C$


    1177   

    Se $0<x<y$ prove que $\sqrt[3]{y-x}>\sqrt[3]{y}-\sqrt[3]{x}$.


    1393   

    O calor específico de um metal como a prata é constante a temperaturas $T$ acima de 200° K. se a temperatura do metal aumenta de $T_1$ a $T_2$, a área sob a curva $y=c/T$ de $T_1$ a $T_2$ é chamada variação de entropia $\Delta S$, que é uma medida da desordem molecular do sistema. Expresse $\Delta S$ em termos de $T_1$ e $T_2$,


    1340   

    Avalie o limite $\lim\limits_{x\rightarrow 0}\dfrac{\sin \left(7x\right) }{\sin \left( 23x\right) }$.


    $7/23$.


    48   

    Classifique as afirmações a seguir como verdadeiras ou falsas:

    1. Se $ \lim\limits_{x\to \infty} f(x) = 5$, então estamos implicitamente afirmando que o limite em questão existe.

    2. $\infty/0$ não é uma forma indeterminada.


    1. Verdadeira

    2. Verdadeira


    960   

    Calcule e justifique os seguintes limites, quando existirem, ou justifique a inexistência:
    $\lim\limits_{x\rightarrow 0}\dfrac{\sin 4x}{x}$

    $\lim\limits_{x\rightarrow 0}\dfrac{x}{\sin x}$


    1. $4$.
    2. $1$.


    1913   

    A região entre a curva $y^2=kx$ e a reta $x=\dfrac{1}{4}k$ é feita girar em torno da reta $x=\dfrac{1}{2}k$. Use camadas cilíndricas para encontrar o volume do sólido resultante.


    $\frac{\pi  k^4}{48}$


    1345   

    Use o Teorema do Valor Intermediário para provar que a equação $\tan x= 2-4x$ possui uma solução no intervalo $\bigl(-\frac{\pi}{2}, \frac{\pi}{2}\bigr).$


    216   

    Avalie os seguintes limites para a função definida por partes
    $ f(x) = \left\{\begin{array}{ccc}
    \cos x, & & \text{ se }  x<\pi \\
    \sin x, & & \text{ se }  x\geq \pi
    \end{array}
    \right.$

    1. $ \lim\limits_{x\to \pi^-} f(x)$

    2. $ \lim\limits_{x\to \pi^+} f(x)$

    3. $ \lim\limits_{x\to \pi} f(x)$

    4. $f(\pi)$


    1. $-1$
    2. 0
    3. Não existe.
    4. 0


    695   

    Calcule a integral $\int \frac{1}{x^2-x} dx$.


    1172   

    Determine os pontos de máximo, de mínimo e de inflexão das seguintes funções (se existirem).

    1. $y = 6x^3 + 15x^2-12x -5$

    2. $f(x) = - 9x^2 + 14x +15$


    1605   

    Deixa-se cair de um balão um objeto de massa $m$. Se a força de resistência do ar é diretamente proporcional à velocidade $v(t)$ do objeto no instante $t$, então pode-se mostrar que $v(t)=(mg/k)(1-e^{-(k/m)t})$, onde $k>0$ e $g$ é uma constante gravitacional. Determine $\lim\limits_{k \to 0^+}s(t)$.


    678   

    Encontre as assíntotas horizontais e verticais ao gráfico de $f(x)=\sqrt{\frac{4x^2+1}{x^2-1}}$.


    687   

    Calcule o limite justificando as passagens.

    $\lim\limits_{x\rightarrow \infty }\dfrac{-x^{4}-2x+1}{2x^{4}+2x+3}$.


    1109   

    Encontre $f(x)$ que satisfaça o seguinte problema de valor inicial:
      $f'(x) = 7^x$ e $f(2)= 1$


    $7^x/\ln 7 + 1-49/\ln 7$


    220   

    Calcule e justifique os seguintes limites, quando existirem, ou justifique a inexistência:

    1. $\lim\limits_{x\rightarrow 3^{+}}\dfrac{5}{3-x};$

    2. $\lim\limits_{x\rightarrow 3^{-}}\dfrac{5}{3-x};$

    3. $\lim\limits_{x\rightarrow 0^{+}}\dfrac{5}{x^{2}-x};$

    4. $\lim\limits_{x\rightarrow 0^{-}}\dfrac{5}{x^{2}-x};$

    5. $\lim\limits_{x\rightarrow 0^{+}}\dfrac{\sin x}{x^{3}-x^{2}};$

    6. $\lim\limits_{x\rightarrow -1^{+}}\dfrac{3x^{2}-4}{1-x^{2}}$


    1276   

    Seja $f\left(  x\right)  =\frac{2x-1}{x-1}$

    1.   Encontre o domínio de $f$, os pontos de intersecção do gráfico de $f$ com os eixos, o sinal de $f$ e analise a simetria de $f$.

    2.   Caso existam, determine as assíntotas horizontais, verticais e oblíquas de $f$.

    3.   Determine os intervalos de crescimento e decrescimento de $f$, seus pontos de máximo e mínimo locais.

    4.   Determine os intervalos onde $f$ tem concavidade para cima e para baixo e os pontos de inflexão.

    5.   Esboce o gráfico de $f$ usando as informações obtidas nos itens anteriores.



    1. Domínio: Dom$\left(  f\right)  =\left\{  x|x\neq1\right\}  =\left(  -\infty,1\right)  \cup\left(  1,\infty\right)  $

    2. Zeros e inteceptos: $f\left(  x\right)  =0\iff2x-1=0\iff  x=1/2$

    3. Simetrias: Não há.

    4. Assíntotas:

      \begin{align*}
        \lim_{x\rightarrow\pm\infty}f\left(  x\right)   &  =\lim_{x\rightarrow
        \pm\infty}\frac{2x-1}{x-1}\\
        &  =\lim_{x\rightarrow\pm\infty}\frac{2-1/x}{1-1/x}=2
        \end{align*}
        \begin{align*}
        \lim_{x\rightarrow1^{-}}f\left(  x\right)   &  =\lim_{x\rightarrow1^{-}}
        \frac{2x-1}{x-1}=-\infty\\
        \lim_{x\rightarrow1^{+}}f\left(  x\right)   &  =\lim_{x\rightarrow1^{-}}
        \frac{2x-1}{x-1}=+\infty
        \end{align*}
    5. Intervalos de crescimento e decrescimento:

       \begin{align*}
        f^{\prime}\left(  x\right)   &  =\frac{2\left(  x-1\right)  -\left(
        2x-1\right)  \left(  1\right)  }{\left(  x-1\right)  ^{2}}\\
        &  =\frac{-1}{\left(  x-1\right)  ^{2}}<0,\forall x\in Dom\left(  f\right)
        \end{align*}

        ou seja, $f$ é estritamente decrescente.

    6. Valores máximo e mínimo locais: Não há, pois a derivada não se anula

    7. Concavidade e pontos de Inflexão:


        \[

        f"\left(  x\right)  =\frac{2}{\left(  x-1\right)  ^{3}}>0\iff x-1>0\iff x>1

        \]

        ou seja, $f$ tem concavidade para cima para $x>1$ e concavidade para baixo para $x<1$

    8. Esboço do Gráfico:

      Construa o gráfico da função 1 e da função 2 usando um único plano cartesiano disponível a seguir


    186   

    Prove que $\sqrt{p}$, onde $p$ é primo, é um número irracional.

    A teoria necessária para resolver esta questão pode não ser abordada em alguns cursos de Cálculo 1. Sendo, também pertinente, às disciplinas Teoria dos Números e Análise Real I.
    Para aprofundar seus conhecimentos, dentro do escopo de Cálculo 1, recomendamos a leitura do Cap. 1 de Guidorizzi, vol. 1 e /ou o Prólogo de Spivak (vide Bibliografia de Cálculo 1).


    567   

    Estude a função $f\left( x\right) =\dfrac{3x^{2}+4x}{1+x}$ com relação à concavidade, pontos de inflexão, máximos e mínimos, e esboce o seu gráfico.


    1758   

    Enuncie e demonstre o primeiro Teorema Fundamental do Cálculo.


    1919   

    O laço de de $9y^2=x(3-x)^2$ é girado ao redor do eixo $y$. Calcule a área da superfície gerada por essa maneira.


    550   

    Faça um esboço completo do gráfico da função $y=\ln (9-x^{2}).$ Suas derivadas são: $y^{\prime }=-2x/\left( 9-x^{2}\right) $ e $y^{\prime \prime }=-\left( 18+2x^{2}\right) /\left( 9-x^{2}\right) ^{2}$. Determine explicitamente:

    1. Domínio de definição;

    2. Assíntotas verticais e horizontais (se houver);

    3. Intervalos de crescimento e decrescimento;

    4. Pontos de máximo e mínimo locais e absolutos;

    5. Pontos de inflexão;

    6. Concavidade.


    1810   

    1.  Mostre que as funções $f(x)=(x-1)^4$ e $g(x)=x^3-3x^2+3x-2$ têm pontos estacionários em $x=1$.

    2.  O que o teste da derivada primeira diz sobre a natureza destes pontos?


    523   

    Uma escada de $5 m$ de altura está apoiada numa parede vertical. Se a base da escada é arrastada horizontalmente da parede a $3 m/s$, a que velocidade desliza a parte superior da escada ao longo da parede quando a base encontra-se a $3 m$ da parede?


    1333   

    Determine uma primitiva para cada uma das funções:

    1. $f(x)=1+2x+3x^2+4x^3+5x^4$

    2. $f(x)=1+x+x^2+\ldots +x^{1000000}$


    1. $F(x)=1+x+x^2+x^3+x^4+x^5$
    2. $F(x)=x+\frac{x^2}{2}+\frac{x^3}{3}+\ldots+\frac{x^1000001}{1000001}$


    1596   

    Se uma função ímpar $f(x)$ possui um valor máximo local em $x=c$, pode-se dizer algo sobre o valor de $f$ quando $x=-c$?


    Ela terá um mínimo local em $x=-c$. É uma questão de simetria de seu gráfico em relação à origem.


    1489   

    O produto de um racional diferente de zero com um irracional é racional ou irracional? Justifique.


    É irracional.


    620   

    Um fabricante produzirá caixas fechadas (com tampa) de volume igual a $27$ litros e cuja base é um retângulo com comprimento igual ao triplo da largura. Encontre as dimensões da caixa de forma que o consumo de material seja mínimo.


    561   

    Determine os intervalos de decrescimento e crescimento e esboce o gráfico da função  $f\left( x\right) =\left\{\begin{array}{cc} e^{-\dfrac{1}{x^{2}}} & \text{se }x\neq 0 \\ 0 & \text{se }x=0 \end{array} \right. $


    1189   

    Calcule a derivada da seguinte função:
     $f\left(  x\right)  =\frac{\left(  x^{2}-1\right)  ^{2}}{\sqrt{x^{2}+1}}.$


    $f'(x) = \dfrac{x(x^2-1)(3x^2+5)}{(x^2+1)^{3/2}}$.


    773   

    Determine uma reta que seja paralela a $x+y=1$ e tangente à curva $x^{2}+xy+y^{2}=3$


    900   

    Sabendo que $x$ é um número negativo, simplifique a expressão $\sqrt{(x-3)^2}+\sqrt{x^2}+\sqrt{(4-3x)^2}$.


    157   

    Seja $f:\mathbb{R} \to \mathbb{R}$ contínua e tal que $f(x).f(f(x))=1$, para todo $x$. Se $f(1000)=999$, calcule $f(500)$.


    655   

    Esboce o gráfico de cada uma das funções abaixo.

    1. $y=\sqrt{9-(2-x)^{2}}$

    2. $y=7/2-\sqrt{13-(2+x)^{2}}$


    964   

    Calcule e justifique os seguintes limites, quando existirem, ou justifique a inexistência:

    1. $\lim\limits_{x\rightarrow 0}\left( 1+2x\right)^{\dfrac{1}{x}}$
    2. $\lim\limits_{x\rightarrow 0}\dfrac{e^{2x}-1}{x}$
    3. $\lim\limits_{x\rightarrow 0}\dfrac{e^{x^{2}}-1}{x}$


    618   

    Determine o domínio da seguinte função:

    $f\left( x\right) =\sqrt{x-1}+\sqrt{3-x}$.


    $\left\{ 1\leq x\leq 3\right\} $.


    1804   

    Encontre os valores de $p$ tais que a integral $\displaystyle \int_0^{+\infty} e^{px} \, dx$ converge.


    332   

    O  consumo  de  combustível  de um automóvel  é  função  da  sua velocidade média. Para  certo  automóvel, essa   função é aproximadamente dada por $y = 0,03x^2-2x + 20$, sendo $y$ o consumo de combustível, em mililitros por quilômetros por hora. Nessas condições, para esse automóvel, qual velocidade média corresponde a um consumo de $120 ml/km$?


    1254   

    Determine a derivada de ordem $999$ da função $f(x)=\sin(x)+\cos(x)$.


    1570   

    Seja $f(x)=\left\{\begin{array}{ll}
    x^2, & \text{se } x \leq 0 \\
    -x^2, & \text{se } x>0  
    \end{array}\right.$

    1. $f$ é contínua em $0$. Por quê?
    2. $f$ é derivável em $0$. Por quê?


    1. Sim.
    2. Sim.


    1777   

    Calcule a integral $\displaystyle \int (1+ \sin t)^9 \cos t \, dt$, utilizando a substituição $u=1+\sin t$.


    1487   

    Sejam $a<b$ dois reais e $p \in \left]a,b \right[$. Determine $r>0$ de modo que $\left]p-r,p+r \right[ \subset \left]a,b \right[$.


    184   

    Prove que $\sqrt{2}$ é irracional.

    A teoria necessária para resolver esta questão pode não ser abordada em alguns cursos de Cálculo 1. Sendo, também pertinente, às disciplinas Teoria dos Números e Análise Real I.
    Para aprofundar seus conhecimentos, dentro do escopo de Cálculo 1, recomendamos a leitura do Cap. 1 de Guidorizzi, vol. 1 e /ou o Prólogo de Spivak (vide Bibliografia de Cálculo 1).


    1696   

    Determine a área da superfície gerada pela rotação da curva a seguir em torno do eixo indicado.

    $y=x^3/9$,  $0\leq x \leq 2$, eixo $x$


    168   

    Seja $f$ uma função contínua em $[-1,1]$ sendo que $f(-1) = -10$ e $f(1) = 10$. Existe um valor $-1<c<1$ tal que $f(c) = 11$? Por quê?



    Não se pode dizer. O Teorema do Valor Intermediário apenas se aplica, neste caso, para valores entre $-10$ e $10$; como $11$ não pertence a este intervalo, o teorema não nos permite afirmar nada sobre a possibilidade da existência de $c$.


    685   

    Calcule o limite justificando as passagens.

    $\lim\limits_{x\rightarrow +\infty }\dfrac{-x^{3}+2}{4x^{2}+89}$.


    1248   

      Seja $ f\left( x\right) =\dfrac{x^{2}+7x+3}{x^{2}}$

    1.   Encontre o domínio de $f$, os pontos de intersecção do gráfico de $f$ com os eixos, o sinal de $f$ e analise a simetria de $f$.

    2.   Caso existam, determine as assíntotas horizontais, verticais e oblíquas de $f$.

    3.   Determine os intervalos de crescimento e decrescimento de $f$, seus pontos de máximo e mínimo locais.

    4.   Determine os intervalos onde $f$ tem concavidade para cima e para baixo e os pontos de inflexão.

    5.   Esboce o gráfico de $f$ usando as informações obtidas nos itens anteriores.


    328   

    Na fabricação de um lote de peças de certo produto, o custo total é igual à soma de um valor fixo de $R\$ 400,00$ com o custo de produção unitário de $R\$ 0,50$. Se o preço unitário de venda dessas peças for de $R\$ 0,85$, qual o número mínimo de peças que devem ser fabricadas e vendidas para que se comece a ter lucro?


    1099   

    Avalie a seguinte integral indefinida:
      $\int (\sec x\tan x + \csc x\cot x)\  dx$


      $\sec x - \csc x+C$


    598   

    Mostre que $|x-y|<1/2,|x+2|<1/3\Longrightarrow |y+2|<5/6$.


    86   

    Calcule os limites:

    1. $\lim\limits_{x\to6} \frac{x^2-4 x-12}{x^2-13 x+42}$

    2. $\lim\limits_{x\to0} \frac{x^2+2 x}{x^2-2 x}$

    3. $\lim\limits_{x\to2} \frac{x^2+6 x-16}{x^2-3 x+2}$


    1. $-8$

    2. $-1$

    3. $10$


    554   

    Determine os intervalos de decrescimento e crescimento e esboce o gráfico da seguinte função  $f\left( x\right) =x^{3}-3x^{2}+1$.


    191   

    Classifique as afirmações em verdadeiras ou falsas.

    1. Um número racional qualquer tem sempre um numero finito de ordens (casas) decimais.

    2. Um número racional qualquer tem sempre um numero infinito de ordens (casas) decimais.

    3. Um número racional qualquer não pode expressar-se em forma decimal exata.

    4. Um número racional qualquer nunca se expressa em forma de uma decimal inexata. 


    1. F

    2. F

    3. F

    4. F


    1242   

    Encontre os dois pontos onde a curva $x^2+xy+y^2=7$  cruza o eixo x e mostre que as tangentes à curva nesses pontos são paralelas. Qual é o coeficiente angular comum dessas retas?


    1744   

    Escreva o número $e$ como uma soma (com a notação $\Sigma$), com um erro menor que $10^{-4}$.


    641   

    Nos exercícios abaixo determine o domínio máximo de definição de cada uma das funções dadas.

    1. $y=\sqrt{x^{2}-9}$

    2. $y=\sqrt{-x}$


    1. $\{x \in \mathbb{R}; x<-3 \text{ ou }x>3\}$.
    2. $\{x \in \mathbb{R}; x<0\}$.


    1923   

    O volume de água em um tanque varia de acordo com a função $V(t)= 10 - |4-2t| -|2t - 6|$, onde $V$ é o volume medido em $m^3$ após $t$ horas, contadas a partir de $8$ h da manhã. 

    1. Atribua um domínio para $V(t)$, considerando que um volume negativo não tem sentido na realidade. 
    2. Faça o gráfico de $V(t)$ com $t$ no domínio estabelecido no item anterior.
    3. Para que valores de $t$ o tanque está enchendo?
    4. Para que valores de $t$ o tanque está esvaziando?
    5. Em qual horário o volume do tanque é constante?


    841   

    O que podemos dizer sobre uma função $f\left( x\right) $ tal

    que $f^{\prime }\left( f\left( x\right) \right) =\left( f\left( f\left(x\right) \right) \right) ^{\prime }$ para todo $x$?



    Pela aplicação direta da Regra da cadeia, temos que:

    $\left( f\left( f\left(x\right) \right) \right) ^{\prime }=f^{\prime }\left( f\left( x\right) \right)f^{\prime}(x)$

    Para $f(x)$, portanto, temos que:

    $f^{\prime }\left( f\left( x\right) \right) =f^{\prime }\left( f\left( x\right) \right)f^{\prime}(x)$

    Para que a igualdade seja verdadeira, há duas possibilidades. Ou:

    $f^{\prime}(x)=0,\,\forall x$

     i.e., a função é uma constante (o que resultaria em $0=0$). Ou: 

    $f^{\prime}(x)=1,\,\forall x$

    i.e., $f(x)=x+a$, sendo que $a$ é uma constante (o que resultaria em $f^{\prime }\left( f\left( x\right) \right)=f^{\prime }\left( f\left( x\right) \right)$).


    1555   

    Para uma população de elefantas africanas, o peso $W(t)$ (em quilogramas) e a idade $t$ (em anos) pode ser aproximado por uma função de crescimento de Fertanlanffy $W$ tal que $W(t)=2600(1-0,51e^{-0,075t})^3$.

    1. Dê uma aproximação do peso e da taxa de crescimento de um elefante recém-nascido.
    2. Supondo que uma elefanta adulta pese $1800$ $kg$, estime sua idade e sua taxa de crescimento presente.
    3. Calcule e interprete $\lim\limits_{t \to \infty}W(t)$.
    4. Mostre que a taxa de crescimento é máxima entre as idades de $5$ e $6$ anos.


    137   

    Para a função a seguir, dê os intervalos nos quais ela é contínua:

     $ g(x) = \sqrt{x^2-4}$.


    $(-\infty,-2]\cup [2,\infty)$


    1861   

    Uma bola esférica oca de raio $2m$ tem densidade específica $\dfrac{1}{4}$, de modo que flutua na água deslocando $\dfrac{1}{4}$ de seu próprio volume. Mostre que a profundidade $x$ à qual fica submersa é uma raiz da equação $x^3-6x^2+8=$ e use o método de Newton para calcular essa raiz com duas casas decimais de precisão. Sugestão: o volume de um segmento esférico de altura $h$ retirado de uma esfera de raio $r$ é $\pi h^2 \left(r-\dfrac{h}{3}\right)$.


    1088   

    Defina o termo antiderivada com suas próprias palavras.


    A antiderivada de uma função $f$ é uma função $F$ cuja derivada é a função $f$ original.


    1553   

    A corrente $I(t)$ em um circuito elétrico composto de um resistor e um indutor, no instante $t$, é dada por $I(t)=I_0e^{-Rt/L}$, onde $R$ é a resistência, $L$ a indutância e $I_0$ é a corrente no instante $t=0$. Mostre que a taxa de variação da corrente no instante $t$ é proporcional a $I(t)$.


    1269   

    Calcule a seguinte integral:
       $\int e^{x}\sin xdx.$


    $\dfrac{1}{2}e^x(sinx-cosx)+C$


    1609   

    Se uma quantia $P$ é aplicada à taxa de juros de $100r \%$ ao ano, composta $m$ vezes por ano, então o montante, ao cabo de $t$ anos, é dado por $P(1+rm^{-1})^{mt}$. Considerando $m$ como um número real e fazendo m crescer indefinidamente, diz-se que a taxa é composta continuamente. Mostre que, neste caso, o montante após $t$ anos é $Pe^{rt}$.


    1819   

    Seja $f(x)=\sin{x}+\cos{x}$, $0 \leq x \leq 2 \pi$.
    Estude o sinal de $f'(x)$.
    Faça um esboço do gráfico de $f$.


    1125   

    Seja:

    • $\int_0^2{f(x)dx} = 5$
    • $\int_0^3{f(x)dx} = 7$
    • $\int_0^2{g(x)dx} = -3$ e
    • $\int_0^3{g(x)dx} = 5$

    A partir destes valores, calcule as seguintes integrais:

    1. $\int_0^2 \big(f(x)+g(x)\big) \ dx$
    2. $\int_0^3 \big(f(x)-g(x)\big) \ dx$
    3. $\int_2^3 \big(3f(x)+2g(x)\big) \ dx$
    4. Encontre valores para $a$ e $b$ tal que:
      $\int_0^3 \big(af(x)+bg(x)\big) \ dx=0$


    1. $2$
    2. $2$
    3. $22$
    4. $a=-\frac{5}{7}b,\quad b\in\mathbb{R}$


    560   

    Determine os intervalos de decrescimento e crescimento e esboce o gráfico da seguinte função  $f\left( x\right) =x-e^{x}$.


    942   

    O que há de errado com a seguinte ``definição'' de limite?

    "O limite de $f(x)$, quando $x$ tende a $a$, é $K$'' significa que para qualquer $\delta>0$, existe $\epsilon>0$ tal que $|f(x)-K|< \epsilon$, tem-se $|x-a|<\delta$."



    $\epsilon$ deve ser apresentado antes, e a restrição $|x-a|<\delta$ implica em $|f(x)-K|< \epsilon$, e não o contrário.


    730   

    Calcule o seguinte limite:

    $\lim\limits_{x\rightarrow \infty }\log _{3}x$.


    $\infty$.


    1695   

    Demonstre que, ao se cortar uma cebola em fatias de igual largura, todas as fatias terão a mesma quantidade de casca.
    Para isso, considere o semicírculo dado pela equação $y=\sqrt{r^2-x^2}$. A rotação deste em torno do eixo $x$ resultará numa esfera. Se escolhermos $x_0 >0$ e $h>0$ tal que $-r \leq x_0 < r$ e $x_0+h \geq r$, e o arco $AB$ localizado acima deste intervalo.

    Demonstre que a área gerada pela rotação do arco $AB$ não depende de $x_0$, apenas de $h$.

    Construa o gráfico da função 1 e da função 2 usando um único plano cartesiano disponível a seguir


    1561   

    Determine a equação da reta tangente ao gráfico de $f(x)=\ln{x}$ no ponto de abscissa $1$. Esboce os gráficos de $f$ e da reta tangente.


    752   

    Dada uma função $f:{\mathbb{R} \to \mathbb{R}}$, defina sua continuidade no ponto $p\in \mathbb{R}.$


    639   

    Sejam $f(x)=\sqrt{\displaystyle{\frac{x+3}{x-3}}}$ e $g(x)=\displaystyle{\frac{\sqrt{x+3}}{\sqrt{x-3}}}$. Determine o domínio da função $f$ e o domínio da função $g$. É verdade que $f=g$?


    577   

    Esboce o gráfico e encontre os zeros da função $f\left( x\right) =\left| x-3\right| -\left| x+4\right| +\left| 5-x\right| $.


    1715   

    Se você investir $1000$ reais em uma aplicação que paga $7$% de juros compostos em $n$ vezes por ano, então em $10$ anos sua aplicação terá no total $1000(1+0,07/n)^{10n}$ reais.

    1. Quanto dinheiro você terá em $10$ anos se a taxa de juros é composta trimestralmente ($n=4$)?

    2. Quanto dinheiro você terá em $10$ anos se a taxa de juros é composta mensalmente ($n=12$)?

    3. Quanto dinheiro você terá em $10$ anos se a taxa de juros é composta mensalmente ($n=365$)?

    4. Pesquise a taxa de juros paga pela poupança, e o período em que ela é composta. Calcule a quantidade de dinheiro que você terá se investir uma certa quantia de dinheiro (pense no dinheiro você tem disponível para investir) em $1$, $2$, $5$ e $10$ anos com essa taxa e período de composição. Interprete os resultados pensando em seu futuro!

    5. Quanto dinheiro você terá em $10$ anos se os juros forem compostos continuamente, isto é, se $n\to\infty$?


    115   

    Considere uma função contínua $\phi:\mathbb{R} \to \mathbb{R}$ tal que

     \[ \forall \quad {x \in \mathbb{R}},\quad \phi(x)\geq x^2.\]

    Mostre que existe $a\geq 0$ tal que $\left[a,+\infty\right[$ é o contradomínio de $\phi$.


    1298   

    Utilize o método das cascas cilíndricas para calcular o volume de um cone circular reto de altura $h$ e base com raio $r$.



    Podemos pensar no cone como a superfície de revolução obtida pela rotação de um segmento de reta. A reta em questão pode ser equacionada, por semelhança de triângulos, como
      \[
      \frac{y}{x}=\frac{h}{r}\text{ ou }y:=y\left(  x\right)  =\frac{h}{r}x\text{.}%
      \]
      O segmento de reta é determinado ao restringirmos $x\in\left[  0,r\right]$. Observamos que, dado $x\in\left[  0,r\right]  $ temos que a altura $h\left(  x\right)  $ correspondente ao cilindro contido no cone é $h\left(  x\right)  =h-y\left(  x\right)  $. Chamando o volume de $V$, pelo método das cascas cilíndricas, obtemos que:
      \begin{align*}
      V  & =\int_{0}^{r}\left(  2\pi x\right)  \left(  h-\frac{h}{r}x\right)  dx\\
      & =\int_{0}^{r}2\pi h\left(  x-\frac{x^{2}}{r}\right)  dx\\
      & =2\pi h\left.  \left(  \frac{x^{2}}{2}-\frac{x^{3}}{3r}\right)  \right\vert
      _{0}^{r}\\
      & =2\pi h\left(  \frac{r^{2}}{2}-\frac{r^{3}}{3r}\right)  \\
      & =2\pi h\frac{r^{2}}{6}\\
      & =\frac{\pi hr^{2}}{3}.
      \end{align*}


    1547   

    Um aluno estudioso está sentado em uma sala de aula, ao lado da parede e de frente para a lousa, como na figura abaixo. A lousa tem $3$m de largura e começa a $1$m da parede à qual o aluno está próximo. Mostre que, se a distância da parede for $x$, o ângulo de visão é

    $$\alpha = \cot^{-1} \dfrac{x}{15} - \cot^{-1} \dfrac{x}{3}.$$

    Construa o gráfico da função 1 e da função 2 usando um único plano cartesiano disponível a seguir


    825   

    Mostre que $|\cos x-\cos y|\leq |x-y|$ quaisquer que sejam $x$ e $y$ reais, enunciando os teoremas utilizados.


    572   

    Estude a função $f\left( x\right) =e^{x}-e^{3x}$ com relação à concavidade, pontos de inflexão, máximos e mínimos, e esboce o seu gráfico.


    67   

    Mostre que $\lim\limits_{x\rightarrow p}f\left( x\right) =L$ se e somente se $\lim\limits_{x\rightarrow p}\left( f\left( x\right) -L\right) =0$.

    1.  Suponha que $f\left( x\right) \leq g\left( x\right) $ para todo $x$.   Demonstre que $\lim\limits_{x\rightarrow p}f\left( x\right) \leq   \lim\limits_{x\rightarrow p}g\left( x\right) $ sempre que os limites   existirem.

    2.  Suponha agora que $f\left( x\right) <g\left( x\right) $ para todo $x$. Podemos afirmar que $\lim\limits_{x\rightarrow p}f\left( x\right)   <\lim\limits_{x\rightarrow p}g\left( x\right) $ sempre que os limites  existirem?


    1291   

    Calcule a área no plano entre os gráficos de $f\left( x\right) =x^{3}-x$ e $g\left( x\right) =sen\left( \pi x\right) $ no intervalo $[0,1]$.


    104   

    Determine os valores para os quais a função \begin{align*} f(x) =\left\{ \begin{array} [c]{c} x^{2}+1,\text{ se }x\leq0 \\ \cos x, \text{ se } 0<x<1 \\ x^{2}+1, \text{ se }1 \leq x \end{array} \right.\end{align*} é contínua. Justifique sua resposta.


    1272   

    Calcule a seguinte integral:
       $ \int_{-\infty}^{e^4}\frac{\ln (x)}{x}dx$.


    557   

    Determine os intervalos de decrescimento e crescimento e esboce o gráfico da seguinte função  $f\left( x\right) =\dfrac{x^{3}-x^{2}+1}{x}$.


    175   

    Calcule o valor de $\frac{2}{0,666 \ldots}$.



    $$\begin{array}{rcl} \dfrac{2}{0.666 \ldots} &=& \dfrac{2}{\dfrac{6}{9}} \\ &=& 2 \dfrac{9}{6} \\ &=& \dfrac{9}{3} \\ &=& 3. \end{array}$$


    1092   

    Avalie a seguinte integral indefinida:
      $\int 3x^3\ dx$


     $3/4x^4+C$


    1813   

    Use a derivada dada para encontrar as coordenadas $x$ de todos os pontos críticos de $f$ e classifique-os em máximo relativo, mínimo relativo ou nenhum dos dois.

    1.  $\displaystyle f'(x)=x^3(x^2-5)$;

    2.  $\displaystyle f'(x)=xe^{-x}$.


    648   

    Verifique se as funções abaixo são pares, ímpares ou nenhuma das duas coisas.

    1. $f(x)=\tan x$

    2. $f(x)=x^{2}+1$


    787   

    Encontre o ponto de interseção da reta tangente ao gráfico de $y=x-\frac{1}{x}$ no ponto $(1,0)$ com o eixo $y$.


    $(1,0)$.


    1593   

    Suponha que em qualquer instante $t$ (em segundos) a corrente $i$ (em amperes) em um circuito de corrente alternada é $i = 2\ cos\ t+2\ sin\ t$. Qual a corrente de pico (magnitude máxim para este circuito?


    1719   

    A função de Heaviside (também conhecida como função degrau), cujo gráfico pode ser visto abaixo, é muito utilizada para modelar chaves que ligam e desligam em circuitos elétricos (e também diversas aplicações). O que você tem a dizer sobre a continuidade dessa função? E sobre a diferenciabilidade?

    Construa o gráfico da função 1 e da função 2 usando um único plano cartesiano disponível a seguir


    1591   

    Em uma esteira transportadora, areia é derrubada a uma taxa de $10$m$^3/$min no topo de um monte em formato de cone. A relação entre a altura do monte e o diâmetro da base é sempre de $3/8$.

    1. Qual a taxa de variação da altura?
    2. Qual a taxa de variação do raio, se o monte tiver $4$m de altura?


    1506   

    Se Fidelis investisse $R\$1500$ em uma conta aposentadoria que rende $8\%$ de juros compostos anualmente, em quanto tempo este investimento isoladamente aumentará para $R\$5000$?


    1519   

    Sabe-se que $f$ é contínua em $2$ e que $f(2)=8$. Mostre que existe $\delta>0$ tal que para todo $x \in D_f$  vale $2-\delta<x<2+\delta \rightarrow f(x)>7$.



    Considere $\epsilon =1$. Como $f$ é contínua em $2$, sabemos que existe $\delta >0$ tal que, para $|x-2|<\delta $ temos que $|f(x)-f(2)|<\epsilon =1$. Mas $|x-2|<\delta $ se, e somente se, $2-\delta<x<2+\delta$ e $|f(x)-f(2)|=|f(x)-8|<1$ se, e somente se, $7< f(x)<9$.


    128   

    Descreva três situações nas quais $\displaystyle \lim\limits_{x\to c}f(x)$ não existe.



    A função pode tender a valores diferentes pela esquerda e pela direita, a função pode crescer de maneira ilimitada, ou a função pode oscilar em torno de um valor.


    788   

    Encontre as equações das retas que passam pelo ponto $(-1,1)$ e são tangentes à curva $x^2+4y^2-4x-8y+3=0.$


    644   

    Nos exercícios abaixo determine o domínio máximo de definição de cada uma das funções dadas.

    1. $y=\sqrt{x+5}$

    2. $y=\sqrt{3-2x}$


    1. $[-5,\infty[$
    2. $]-\infty,\frac{3}{2}]$


    192   

    Classifique as afirmações em verdadeiras ou falsas.

    1. A soma de dois números racionais é sempre um número racional.

    2. A soma de dois números irracionais é sempre um número irracional.

    3. A soma de um número racional com um número irracional é sempre um número irracional.


    1. V

    2. F

    3. V


    1493   

    Dê o domínio e esboce o gráfico das seguintes funções:

    1.  $f(x)=-2+ 1/x$
    2.  $f(x)=-\dfrac{1}{x}$


    1185   

    Determine a derivada da seguinte função:
      $f\left( x\right) =\left( \left( \sin x\right) \left(\cos x\right) \right) ^{3}.$


    $f'(x)=3/8 \sin(2x) \sin(4x)$.


    1566   

    Em estatística, a função densidade de probabilidade para a distribuição normal é definida por $f(x)=\dfrac{1}{\sigma \sqrt{2 \pi}}e^{-z^2/2}$ com $z=\dfrac{x-\mu}{\sigma}$ para números reais $\mu$ e $\sigma>0$ ($\mu$ é a média e $\sigma^2$ é a variância da distribuição). Obtenha os extremos locais de $f$ e determine onde $f$ é crescente ou decrescente. Discuta a concavidade, ache os pontos de inflexão, determine $\lim\limits_{x \to \pm \infty}f(x)$ e esboce o gráfico de $f$.


    574   

    Estude a função $f\left( x\right) =e^{\dfrac{x-1}{x^{2}}}$ com relação à concavidade, pontos de inflexão, máximos e mínimos, e esboce o seu gráfico.


    42   

    Determine todas as assíntotas horizontais da função $f(x) = \frac{x^2-1}{-x^2-1}$.


    $y=-1$.


    1754   

    1. Mostre que, para toda parábola dada por $y=ax^2$, com $a>0$, temos: $$\displaystyle \int_0^b a x^2 \, dx = a \dfrac{b^3}{3}.$$

    2. Use este fato para provar que a área do setor parabólico delimitado por $y=ax^2$ e a reta $y=ab^2$ é igual a quatro terços da área do triângulo com vértices $(0,0)$, $(b,ab^2)$ e $(-b,ab^2)$. Este resultado é um caso particular do Teorema de Arquimedes sobre a área de um setor parabólico.


    1277   

    Esboce o gŕáfico de $f\left(  x\right)  =\frac{e^{-x}}{x}$ .Para fazê-lo:

    1. Domínio da função

    2. Zeros e inteceptos

    3. Simetrias

    4. Assíntotas horizontais e verticais

    5. Intervalos de crescimento e decrescimento

    6. Pontos de máximo e mínimo

    7. Concavidade

    8. Pontos de inflexão



      1. Dom$\left(  f\right)  =\left\{  x\in\mathbb{R}|x\neq0\right\}  $

      2. $f\left(  x\right)  \neq 0,\forall x$

      3. A função não possui simetrias não triviais

      4. $\lim_{x\rightarrow\infty}\frac{e^{-x}}{x}=0,\lim_{x\rightarrow-\infty}\frac{e^{-x}}{x}=\lim_{x\rightarrow-\infty}-e^{-x}=-\infty$ (este por L'Hôpital), $\lim_{x\rightarrow0^{-}}\frac{e^{-x}}{x}=-\infty$ e $\lim_{\times\rightarrow1^{+}}\frac{e^{-x}}{x}=+\infty$

      5. \[
          f^{\prime}\left(  x\right)  =\frac{-e^{-x}x-e^{-x}}{x^{2}}=-e^{-x}\frac
          {x+1}{x^{2}}%
          \]
          e temos que
          \begin{align*}
          f^{\prime}\left(  x\right)    & >0\Leftrightarrow x<-1\\
          f^{\prime}\left(  x\right)    & <0\Leftrightarrow x>-1
          \end{align*}
          logo $f$ é crescente para $x<-1$ $\ $e decrescente para $x>-1$ (lembrando que $x\neq0$).
      6. O único ponto crítico de $f$ é $x=-1$, o qual é ponto de máximo, pois a derivada passa de positiva a negativa.

      7. \begin{align*}
          f"\left(  x\right)    & =\frac{\left(  e^{-x}x-e^{-x}+e^{-x}\right)
          x^{2}-\left(  -e^{-x}x-e^{-x}\right)  2x}{x^{4}}\\
          & =\frac{e^{-x}x^{3}+2e^{-x}x^{2}+2e^{-x}x}{x^{4}}\\
          & =\frac{e^{-x}}{x^{3}}\left(  x^{2}+2x+2\right)
          \end{align*}

          Como $e^{-x}$ e $x^{2}+2x+2$ são sempre positivos, temos que $f"\left(  x\right)  >0$ se $x>0$ e $f"\left(  x\right)  <0$ se $x<0$, ou
          seja, "concavidade para baixo" se $x<0$ e "concavidade para cima" se $x>0$

      8. Esboço do Gráfico:
        Construa o gráfico da função 1 e da função 2 usando um único plano cartesiano disponível a seguir


      1868   

      A atmosfera da Terra absorve aproximadamente $32\%$ da radiação proveniente do Sol. A Terra também emite radiação (a maior parte em forma de calor) e a atmosfera absorve aproximadamente $93\%$ dessa radiação. A diferença entre a radiação que entra na Terra e a que sai é chamada efeito-estufa. Modoficações nesse equilíbrio podem afetar o clima da Terra. Seja $I_0$ a intensidade da radiação do Sol e $I$ a intensidade depois de percorrer uma distância $x$ na atmosfera. Se $p(h)$ é a densidade da atmosfera na altitude $h$, então a espessura ótica é $f(x)=k \displaystyle\int_0^x p(h) dh$, onde $k$ é uma constante de absorção e $I$ é dada por $I=I_0e^{-f(x)}$. Mostre que $dI/dx=-kp(x)I$. 


      794   

      Determine a equação da reta tangente em $\left( p,f\left(p\right) \right)$:

      $f\left( x\right) =1/x^{2},\;p=1$.


      $y=-2x+3$.


      723   

      Calcule o limite:

      $\lim\limits_{x\rightarrow +\infty }\left( x-\sqrt{x^{2}+4x}\right)$.


      $-2$.


      133   

      Para a função a seguir, responda se a mesma é contínua nos pontos abaixo (e, caso não o seja, justifique)

        $ f(x) = \left\{\begin{array}{ccc}
        x^3-x,  & & \text{se } x<1\\
      x-2, & & \text{se } x\geq 1
      \end{array}\right.$

      1.  $x=0$.
      2. $x=1$.


      1. Sim.
      2. Não: Os limites pela direita e pela esquerda não são iguais em $x=1$.


      737   

      Mostre que o volume de uma esfera de raio $R$ é $\dfrac{4}{3}\pi R^{3}$.


      1495   

      Dê o domínio e esboce o gráfico das seguintes funções:

      1.  $f(x)=|x|+1/x$
      2.  $f(x)=\sqrt{|x|}$


      1652   

      Um objeto é atirado do nível do mar para cima com uma velocidade inicial de $100m/s$.

      1. Supondo que a gravidade seja a única força que atua sobre este objeto superestime sua veocidade depois de $5$ segundos. Use $g=10m/s^2$.

      2. Calcule uma estimativa inferior para a altura atingida depois de $5s$.


      159   

      Prove que a única função contínua $f:\mathbb{R} \to \mathbb{R}$ que satisfaz $f(f(f(x)))=x$ é a função identidade $f(x)=x$. (Sugestão: Prove que se uma função é injetiva e contínua então ela é monótona).


      1194   

      Calcule a derivada da seguinte função:
       $f\left(  x\right)  =\frac{\left(  x^{3}+1\right)  ^{5}}{\left(x^{2}+1\right)  ^{4}}.$


      $\frac{x \left(x^3+1\right)^4 \left(7 x^3+15 x-8\right)}{\left(x^2+1\right)^5}$


      643   

      Nos exercícios abaixo determine o domínio máximo de definição de cada uma das funções dadas.

      1. $y=\sqrt[3]{x-2}$

      2. $y=\displaystyle{\frac{1}{x^{2}-4}}$


      1501   

      Esboce as curvas exponenciais transladadas:
      $y=3^x+2$ e $y=3^{-x}+2$.


      684   

      Calcule o limite justificando as passagens.

      $\lim\limits_{x\rightarrow 0}\dfrac{1-\cos x}{x}$.


      325   

      Mensalmente, pago pela prestação de minha casa $1/5$ do meu salário; metade do resto gasto em alimento e $1/3$ do que sobra coloco na poupança, restando-me ainda $R\$ 800,00$ para gastos diversos. Qual o valor colocado na poupança?


      119   

      Dê exemplo de uma função $f$ que seja descontínua, mas tal que $|f|$ seja contínua.


      914   

      Qual a solução geral da dupla desigualdade $-2<x^2-3<\frac{1}{5}$?


      1625   

       Algumas curvas são tão planas que, na prática, o Método de Newton não consegue se aproximar da raiz suficientemente para fornecer uma aproximação útil. Tente utilizar o Método de Newton em $f(x)=\left(x-1\right)^{40}$ com a estimativa inicial $x_0=2$ para observar a qualidade das aproximações. Utilizando recursos computacionais, observe o gráfico da função.


      735   

      Calcule o volume da esfera de raio $R$ de duas maneiras diferentes: a primeira através da rotação de um gráfico em torno do eixo $x$ e a segunda através da rotação de um gráfico em torno do eixo $y$.


      1801   

      Na teoria de eletromagnetismo, o potencial magnético de uma bobina circular em um ponto de seu eixo é dado por:

      $$\displaystyle u = \dfrac{2\pi N I r}{k} \int_a^{\infty} \dfrac{dx}{(r^2+x^2)^{3/2}},$$

      onde $N$, $I$, $r$, $k$ e $a$ são constantes com significados físicos apropriados. Calcule $u$.


      818   

      Sejam $f\left( x\right) $ e $g\left( x\right) $ funções
      diferenciáveis e suponha que esta assuma os seguintes valores:

      $\begin{array}{|c|c|}
      \hline x & f\left( x\right)  & g\left( x\right)  & f^{\prime }\left(
      x\right)  & g^{\prime }\left( x\right) \\\hline
        0 & 1 & 1 & 5 & 1/3 \\\hline
        1 & 3 & -9 & -1/3 & -8/3 \\\hline
      \end{array}$

      Encontre as derivadas de:

      1. $f\left( x\right) -3g\left( x\right) $ em $x=0;$

      2. $f\left( g\left( x\right) \right) $ em $x=0;$

      3. $\left( x^{11}+f\left( x\right) \right) ^{-2}$ em $x=1;$

      4. $f\left( e^{\sin \left( x-1\right) }\right) $ em $x=1;$


      1. $4$
      2. $8/9$
      3. $-1/3$
      4. $-1/3$


      89   

      Calcule os seguintes limites:

      1. $\lim\limits_{x\rightarrow p} \frac{\sin \left(x^{2}-p^{2}\right) }{x-p}$
      2. $\lim\limits_{y\rightarrow 3} \frac{\sin \left(y^{2}-9\right) }{y-3}$
      3. $\lim\limits_{x\rightarrow 4} \frac{\cos \left(x^{2}-16\right) }{x-4}$


      1583   

      Dados $f(x) = \sqrt[\leftroot{-2}\uproot{2}3]{x}$ e $x_0 = 8,5$, escolha um valor inteiro próximo a $x_0$ tal que $f(x_0)$ e $f'(x_0)$ sejam fáceis de calcular, e calcule uma linearização da função neste ponto.


      842   

      Calcule a derivada da função:

      $y=\sqrt{1+\sqrt{x}}$.


      $y'=\dfrac{1}{4\sqrt{\sqrt{x}+1}\sqrt{x}}$.


      680   

      Calcule os seguintes limites laterais (justifique cada passo da resolução):

      1. $\lim\limits_{x\rightarrow1^{+}}\dfrac{\sqrt{x^{2}-1}}{x-1}.$

      2. $\lim\limits_{x\rightarrow1^{-}}\dfrac{\sqrt{x^{2}-1}}{x-1}.$


      603   

      Encontre todos os números reais que satisfazem cada uma das desigualdades abaixo. Dê o intervalo solução e ilustre a solução sobre a reta real.

      1. $(2-x)(x-1)$

      2. $1-x^{2}<0$


      95   

      Calcule o limite a seguir. Justifique as passagens. 

      $\lim\limits_{x\rightarrow +\infty }\dfrac{-x^{3}+2}{4x^{2}+89}$



      552   

      Estude o sinal de $f^{\prime }\left( x\right) $, calcule os limites $\lim\limits_{x\rightarrow \infty }f\left( x\right) $ e $\lim\limits_{x\rightarrow -\infty }f\left( x\right) $ e, utilizando esses dados, esboce o gráfico de $f\left( x\right) =x^{3}+x^{2}-5x$.


      748   

      Calcule o seguinte limite:
      $\lim\limits_{x\rightarrow 0}\dfrac{e^{2x}-1}{x}$.


      $2$.


      1135   

      Suponha que $x(t)=e^{0,05t}$ e que $z(t)=e^{0,01t}$. Calcule a taxa de crescimento de $y(t)$ nos seguintes casos:

      1.    $y=xy$  
      2.    $y=x/y$


      894   

      Resolva as equações:

      1. $|x-2|^2-5|x-2| =-6$
      1. $|x-2|-|x-1| =0$


      1573   

      Determine $f'$, $f''$ e $f'''$ sendo $f(x)=4x^4+2x$.


      $f'(x)=16x^3+2$, $f''(x)=48x^2$ e $f'''(x)=96x$.


      1500   

      Esboce as curvas exponenciais transladadas:
      $y=2^x-1$ e $y=2^{-x}-1$.


      1877   

      Prove que $\displaystyle\int (\ln(x))^m dx=x (\ln(x))^m -m \displaystyle\int (\ln(x))^{m-1}dx$.


      658   

      Determine um número $0\leq b\leq 2$ tal que a reta $x=b$ divide a região delimitada por $y=\sqrt{4-x^{2}}$ e $y=0$ e $x=0$ em duas regiões de mesma área.


      1763   

      Prove que $\cosh'(x)=\sinh(x)$.


      330   

      Uma das raízes da equação $x^2-x-a = 0$ é também raiz da equação $x^2+x-(a + 20)=0$. Qual é o valor de $a$?


      792   

      Determine a equação da reta tangente em $\left( p,f\left(p\right) \right)$:

      $f\left( x\right) =x^{2}-x;\;p=1$.


      $y=x-1$.


      1740   

      Escreva o polinômio $p(x)=x^4-12x^3+44x^2+2x+1$ em $x$ como um polinômio em $(x-3)$. (Só é necessário calcular o polinômio de Taylor em $3$, do mesmo grau do polinômio original. Por quê?)


      546   

      Esboce o gráfico de $f(x)= \frac{x^2-2x^3}{x^2-1}$, indicando campo de definição, intervalos de crescimento e de decrescimento, assíntotas horizontais, verticiais e inclinadas (se houver), limites no infinito, extremos relativos, estudo da concavidade, pontos de inflexão e reta tangente à curva nos pontos de inflexão.


      2   

      Seja  $\ell$ a reta que passa pela origem do plano cartesiano e tangencia a curva $y = x^3 + x + 16$. Qual a inclinação de $\ell$?



      Dado que $\ell$ é uma reta que passa pela origem, sabemos que sua equação é do tipo $\ell(x)=ax$. Como ela tangencia a curva $y(x)$, sabemos que há um ponto $x^*$ tal que $\ell(x^*)=y(x^*)$.

      Além disso, sabemos que em $x^*$ a inclinação de $\ell$ é a mesma inclinação de $y$ (por quê?), o que é equivalente a $\ell'(x^*)=y'(x^*)$.

      Assim, temos:

      \begin{cases}
          \left.x^*\right.^3+x^*+16 = ax^* \\
          3\left.x^*\right.^2+1=a
          \end{cases}

      Resolvendo o sistema de equações obtemos:

      \begin{align*}
      x^* = 2\\
      a = 13
      \end{align*}

      Sendo, portanto, $a=13$ a resposta desejada.


      1676   

      Calcule a integral a seguir utilizando decomposição de quocientes em frações parciais:

      $\int{\frac{x^2dx}{(x-1)(x^2 + 2x + 1)}}$


      836   

      Determine a derivada da função:

      $f\left( x\right) =e^{\cos \left( x^{2}\right)}.$



      Pela regra da cadeia, temos que
      $f(g(x))' = f'(g(x))g'(x)$
      Assim, escolhendo $f(x) = e^x$ e $g(x)=\cos(x^2)$, temos:
      $(e^{\cos(x^2)}))' = e^{\cos(x^2)}(\cos(x^2))'$
      Para calcular $(\cos(x^2))'$, temos que aplicar novamente a regra da cadeia. Desta vez, podemos escolher $f(x)=\cos(x)$ e $g(x)=x^2$.
      Assim, 
      $(\cos(x^2))'= -2\sin(x^2)x$
      Portanto:
      $(e^{\cos(x^2)}))' = -2 e^{\cos(x^2)}x\sin(x^2)$


      1732   

      Em cada item, esboce o gráfico de uma função contínua $f$ com as propriedades indicadas no intervalo $(-\infty,+\infty)$.

      1. $f$ não tem extremos relativos nem absolutos.

      2. $f$ tem um mínimo absoluto em $x=0$, mas nenhum máximo absoluto.

      3.  $f$ tem um máximo e um mínimo absolutos em $x=-5$ e $x=5$, respectivamente.


      36   

      Calcule os seguintes limites:

      1. $\lim\limits_{x\rightarrow \infty }\left( x-\sqrt{x^{3}+2}\right)$

      2. $\lim\limits_{x\rightarrow \infty }\left( x-\sqrt{x^{2}+2}\right)$

      3. $\lim\limits_{x\rightarrow \infty }\left( x-\sqrt{x+2}\right)$


      1.   $-\infty$
      2. $0$
      3. $\infty$


      1767   

      Prove que $\log_{10} 2$ é irracional.


      1685   

      Demonstre que $\int_{-\infty}^{\infty}{f(x)\ dx}$ pode ser diferente de $\lim\limits_{b \rightarrow \infty }\int_{-b}^{b}{f(x)\ dx}$.


      Para isto, mostre que
      $\int_{0}^{\infty}{\frac{2x\ dx}{x^2 +1}\ dx}$


      diverge, e, portanto,
      $\int_{-\infty}^{\infty}{\frac{2x\ dx}{x^2 +1}\ dx}$


      também diverge. Depois, mostre que

      $\lim\limits_{b \rightarrow \infty }\int_{-b}^{b}{\frac{2x\ dx}{x^2 +1}\ dx}=0$


      1521   

      Dê exemplo de uma função definida em $\mathbb{R}$, que não seja contínua em $2$, mas que $\lim\limits_{x \to 2^+}f(x)=\lim\limits_{x \to 2^-}f(x)$.


      1828   

      Se uma função racional $P(x)/Q(x)$ é tal que o grau do numerador excede o grau do denominador em $1$, então o gráfico de $P(x)/Q(x)$ terá uma assíntota oblíqua, isto é, uma assíntota que não é nem horizontal nem vertical. Para ver por quê, efetuamos a divisão de $P(x)$ por $Q(x)$ obtendo $$ \dfrac{P(x)}{Q(x)}= (ax+b) + \dfrac{R(x)}{Q(x)}, $$ onde $(ax+b)$ é o quociente e $R(x)$ é o resto. Use o fato de que o grau do resto $R(x)$ é menor do que o grau do divisor $Q(x)$ para auxiliá-lo a provar que $$ \lim_{x\to \infty}\left[\dfrac{P(x)}{Q(x)}-(ax+b)\right] = 0 \quad \text{e} $$ $$ \lim_{x\to -\infty}\left[\dfrac{P(x)}{Q(x)}-(ax+b)\right] = 0. $$ Este resultado nos diz que o gráfico da equação $\displaystyle y =P(x)/Q(x)$ "tende"  à reta $y=ax+b$ (assíntota oblíqua) quando $x\rightarrow +\infty$ ou $x\rightarrow -\infty$.


      1537   

      Seja $f(x)=\sin{x}+\cos{x}$, $0 \leq x \leq 2 \pi$.

      1. Estude o sinal de $f'(x)$.
      2. Faça um esboço do gráfico de $f$.


      1514   

       Dê os domínios e esboce os gráficos de $f+g$ e $\dfrac{g}{f}$ nos seguintes casos:

      1. $f(x)=1$ e $g(x)=\dfrac{1}{(x-2)^2}$.
      2. $f(x)=1$ e $g(x)=\sqrt{x-1}$.


      742   

      Encontre o volume do sólido obtido pela rotação da região limitada pela curva dada em torno do eixo especificado. Esboce a região e o sólido.
        $y=x^{2},y^{2}=x$, ao redor do eixo $x$.


      734   

      Considere um cilindro com base de diâmetro $2R$ e altura também $2R$. Considere, inscrito neste cilindro, uma esfera de raio $R$ e um cone de base circular com diâmetro $2R$ e altura $2R$. Denote por $V_{cil}$, $V_{esf}$ e $V_{cone}$, respectivamente, os volumes desses sólidos.

      1. Verifique as relações $V_{esf}= \frac{2}{3}V_{cil}$ e $V_{cone}=\frac{1}{3}V_{cil}.$
      2. Calcule $V_{cil}$, $V_{esf}$ e $V_{cone}$ usando integrais. Explicite o método que está usando.


      583   

      Encontre todos os números reais que satisfazem a cada uma das desigualdades abaixo. Dê o intervalo solução e ilustre a solução sobre a reta real.

      1. ${\frac{|x-3|}{|x+7|}}>0$

      2. $|x+4|\geq |x+1|$


      1255   

       Considere a curva definida pela equação $x^2y+3\ln(1-y)+x^4=1.$ 

      1. Calcule $y'.$
      2. Encontre a aproximação linear à curva no ponto $(1,0).$


      334   

      Seja $P(x)$ um polinômio com coeficientes inteiros. Suponha que existam quatro inteiros distintos $a,b,c$ e $d$ tais que $P(a)=P(b)=P(c)=P(d)=5$. Prove que não existe inteiro $k$ tal que $P(k)=8$.


      1922   

      Ache a área da superfície gerada fazendo girar a curva paramétrica $x=t^2,y=2t,0 \leq t\leq 4$, em torno do eixo $x$.


      1720   

      1. Se uma massa cai uma distância $s(t)$ em $t$ segundos, e $s'$ é proporcional a $s$, então mostre que $s$ não pode ser uma função da forma $s(t)=ct^2$.

      2. Se $s(t)=\dfrac{a}{2} t^2$, mostre que $s''(t)=a$ (a aceleração é constante) e que $[s'(t)]^2=2as(t)$ (observe que obtivemos isso trocando ligeiramente a expressão de $s(t)$).

      3. Assumindo $a=9,8$m$/$s$^2$ (aceleração da gravidade), quantos segundos você tem para fugir de um lustre em um castelo que cai de um teto de $100$m? Se você não conseguir fugir, quão rápido o lustre vai estar quando te atingir? A que altura estava o lustre quando estava se movendo com metade desta velocidade?


      165   

      Uma importante aplicação do Teorema do Valor Intermediário é o Método da Bissecção.

      Suponha que estamos interessados em encontrar as raízes de uma função contínua $f(x)$. O Método da Bissecção é uma alternativa que pode resultar em boas aproximações para as raízes, após sucessivas aplicações do método.

      Para iniciar o método, precisamos encontrar dois valores $a$ e $b$ tais que $f(a) \cdot f(b) < 0$.

      Sem perda de generalidade, vamos assumir $f(a) < 0$, $f(b) > 0$ e $a<b$. O Teorema do Valor Intermediário afirma que existe um valor $c$ no intervalo $[a,b]$ tal que $f(c) = 0$. O teorema não afirma nada a respeito da localização de $c$ dentro do intervalo, apenas que ele existe.

      O Método da Bissecção é, portanto, uma maneira sistemática de obter este valor $c$. Seja $d=\frac{a+b}{2}$ o meio do intervalo. Existem três possibilidades:

      1. $f(d) = 0 $ - Por sorte, encontramos a raiz e não é necessário prosseguir com o método.
      2. $f(d) < 0$ - Como $f(b)>0$, sabemos que há uma raiz no intervalo $[d,b]$. Este intervalo tem metade do tamanho do intervalo original, então estamos mais próximos de obter uma boa aproximação para a raiz.
      3. $f(d) > 0$ - Como $f(a)<0$, sabemos que há uma raiz no intervalo $[a,d]$. Novamente, este intervalo tem metade do tamanho do intervalo original, então estamos mais próximos de obter uma boa aproximação para a raiz.

      O Método da Bissecção é a aplicação sucessiva dos passos descritos até que se esteja próximo o suficiente da raiz de $f(x)$ para a aplicação desejada. Nota-se que para o caso em que $f(a)>0$ e $f(b)<0$ o método ainda funciona, mas no caso 2 o intervalo escolhido seria $[a,d]$ e no caso e $[d,b]$ (por quê?).

      Utilize o Método da Bissecção para encontrar as raízes de $f(x) = \sin x - 1/2$ no intervalo $[0.5,0.55]$.


       A raiz aproximada é $x=0.52$.

      Os intervalos utilizados são:

      $[0.5,0.55] \quad [0.5,0.525] \quad [0.5125,0.525]$

      $[0.51875,0.525]\quad [0.521875,0.525]$.


      1899   

      Prove que se $f$ é contínua em $\left[a,b\right]$ , então $\displaystyle\int_{a}^{b}f(x)dx=(b-a)f(\xi)$ para algum $\xi \in \left[a,b\right]$.


      1336   

      Usando os limites fundamentais, encontre o limite  $\lim\limits_{x\rightarrow1}\frac{sen(x-1)}{x^{2}+x-2}$.
       


      $1/3$.


      902   

      Resolva a equação $\displaystyle \frac{x}{1-x} + \frac{x-2}{x}-1 = 0$.


      35   

      Calcule os seguintes limites:

      1. $\lim\limits_{x\rightarrow -\infty }\dfrac{5-x}{2x+3}$

      2. $\lim\limits_{x\rightarrow \infty }\dfrac{\sqrt{x}+1}{x+3}$


      1.   $-1/2$
      2.   $0$


      1731   

      Em um reservatório cônico (com vértice para baixo), água é evaporada a uma taxa proporcional à área da superfície exposta ao ar. Mostre que a profundidade da água decresce a uma taxa constante que não depende das dimensões do reservatório.


      686   

      Calcule o limite justificando as passagens.

      $\lim\limits_{x\rightarrow \infty }\left( x-\sqrt{3x^{3}+2}\right) $.


      614   

      Esboce o gráfico da função abaixo e resolva a inequação:

       $f\left( x\right) =\left\vert x-2\right\vert +\left\vert x-1\right\vert >1$.


      897   

      Esboce o gráfico da função $f(x)=|x^3+3x^2+3x-2|$.


      1900   

      Ache o comprimento exato do arco formado pela curva $x=\dfrac{1}{8}y^4+\dfrac{1}{4}y^{-2}$ de $y=1$ até $y=4$.


      1327   

      Encontre a equação da reta tangente ao gráfico da função $f(x)=12\sqrt[6]{x}-\frac{1}{2x^2}+\log_5(x)$ no ponto cuja coordenada horizontal é $3$.


      1712   

      1. Seja $N$ um número positivo tal que, para cada $x$ no intervalo $(N,+\infty)$, os valores da função $f(x)=1/x^2$ estejam no máximo a $0,1$ unidade de $L=0$. Encontre $N$.

      2. Seja $N$ um número positivo tal que, para cada $x$ no intervalo $(N,+\infty)$, os valores da função $f(x)=x/(x+1)$ estejam no máximo a $0,01$ unidade de $L=0$. Encontre $N$.

      3. Seja $N$ um número positivo tal que, para cada $x$ no intervalo $(-\infty,N)$, os valores da função $f(x)=1/x^3$ estejam no máximo a $0,001$ unidade de $L=0$. Encontre $N$.

      4. Seja $N$ um número positivo tal que, para cada $x$ no intervalo  $(-\infty,N)$, os valores da função $f(x)=x/(x+1)$ estejam no máximo a $0,001$ unidade de $L=0$. Encontre $N$.


      51   

      Avalie os seguintes limites de acordo com o gráfico da função:

      $f(x) = \cos (x)$

      Construa o gráfico da função 1 e da função 2 usando um único plano cartesiano disponível a seguir

      1. $\lim\limits_{x\to -\infty} f(x)$

      2. $\lim\limits_{x\to \infty} f(x)$


      915   

      Determine o conjunto de todos os números reais para os quais a expressão $\sqrt{x}{1-x^2}$ está definida.


      1139   

      Um agricultor possui $140$ metros de cerca para construir dois currais: um deles quadrado eu outro retangular, com comprimento igual ao quádruplo da largura. Se a soma das áreas dos currais deve ser a menor possível, calcule a área do curral quadrado, apresentando todos os cálculos e/ou justificativas.


      663   

      Derive a função $g\left( x\right)  = \int_{\tan x}^{x^{4}}\dfrac{u^{2}+1}{\sqrt{u^{2}+2u}}du$.


      1112   

      Encontre $f(x)$ que satisfaça o seguinte problema de valor inicial:
        $f''(x) = 5$ e $f'(0)= 7$, $f(0) = 3$


        $5/2x^2+7x+3$


      636   

      Sejam $f\left( x\right) =\frac{x^{2}-25}{x^{2}-1}$ e $g\left(x\right) =\sqrt{x}$. Dê o domínio das seguintes funções: $f,$ $g$, $f\circ g$ e $g\circ f$.


      1702   

      1. Prove que se $\lim_{x\to 0}\dfrac{f(x)}{x}=l$ e $b\neq 0 $, então $\lim_{x\to 0}\dfrac{f(bx)}{x}=bl$. Dica: Escreva $\dfrac{f(bx)}{x}=b\dfrac{f(bx)}{bx}$.

      2. O que acontece se $b=0$?

      3. O item 1. nos permite determinar $\lim_{x\to 0}\dfrac{\sin(2x)}{x}$ em termos de $\lim_{x\to 0}\dfrac{\sin(x)}{x}$. Determine este limite de um outro modo.


      593   

      Encontre os valores de $x$ para os quais cada o número $\sqrt{x^{2}+x+3}$ é real.



      Este número será real se o valor dentro da raiz for maior ou igual a zero. Como $x^2 + x + 3$ têm raízes complexas e concavidade para cima, seu valor é sempre maior que zero. Portanto $\sqrt{x^2 + x + 3}$ é real para qualquer $x$ real.


      1597   

       A função
      $V(x)=x(10-2x)(16-2x),\quad 0<x<5$
      modela o volume de uma caixa.

      1. Determine os valores extremos de $V$.
      2. Interprete quaisquer valores encontrados no item anterior em termos do volume da caixa.


      200   

      Prove que $\log2$ é um número irracional.

      A teoria necessária para resolver esta questão pode não ser abordada em alguns cursos de Cálculo 1. Sendo, também pertinente, às disciplinas Teoria dos Números e Análise Real I.

      Para aprofundar seus conhecimentos, dentro do escopo de Cálculo 1, recomendamos a leitura do Cap. 1 de Guidorizzi, vol. 1 e /ou o Prólogo de Spivak (vide Bibliografia de Cálculo 1).


      Dica: Suponha que existam inteiros $p$ e $q$ tais que $log2=p/q$, com $p/q$ sendo fração irredutível. Use a definição de logaritmo e o teorema fundamental da aritmética para chegar a um absurdo.


      1549   

      Se uma droga é injetada em uma corrente sanguínea, sua concentração $C$, $t$ minutos depois, é dada por $C(t)=\frac{k}{a-b}(e^{-bt}-e^{-at})$, para constantes positivas $a$, $b$ e $k$.

      1. Em que instante ocorre a concentração máxima?
      2. Que se pode dizer sobre a concentração após um longo período de tempo? 


      725   

      Calcule o seguinte limite:

      $\lim\limits_{x\rightarrow \infty }3^{x}$.


      $\infty$.


      1917   

      Um buraco redondo de raio $a$ é feito através do centro de uma esfera sólida de raio $r$. Use camadas cilíndricas para encontrar o volume da parte removida (suponha $r>a$).


      1633   

      Prove que $\displaystyle\int \sqrt{a^2+u^2}du=\dfrac{u}{2}\sqrt{a^2+u^2}+\dfrac{a^2}{2}\ln{|u+\sqrt{a^2+u^2}|}+C$.


      1130   

      Aproxime numericamente o seguinte limite
        $ f(x)= \frac{x^2+5 x-36}{x^3-5 x^2+3 x+9}$


      1.  \begin{array}{cc}
          x & f(x) \\ \hline
          2.9 & -335.64 \\
           2.99 & -30350.6 \\
            \end{array}
           A tabela parece indicar que $\lim\limits_{x\to3^-}f(x) =-\infty$.  
      2.   \begin{array}{cc}
          x & f(x) \\ \hline
           3.1 & -265.61 \\
           3.01 & -29650.6 \\
            \end{array}
            A tabela parece indicar que $\lim\limits_{x\to3^+}f(x) =-\infty$.
      3. As tabelas parecem indicar que $\lim\limits_{x\to3}f(x) =-\infty$.


      1559   

      As distribuições gamma, importantes em teoria das probabilidades, são determinadas por $f(x)=cx^ne^{-ax}$ para $x>0$, um inteiro positivo $n$, uma constante positiva $a$ e $c=\dfrac{a^{n+1}}{n!}$.

      1. Mostre que $f$ tem exatamente um máximo local.
      2. Supondo $n=4$, determine onde $f(x)$ cresce mais rapidamente.


      516   

      Seja $f:\mathbb{R\rightarrow R}$ uma função.

      1. Defina continuidade de $f$ no ponto $p\in \mathbb{R}$.
      2. Defina a derivada de $f$ no ponto $p\in \mathbb{R}$. O que é a função derivada $f^{\prime }\left( x\right) ?$
      3. Calcule, pela definição, a derivada $g^{\prime }\left( 0\right) $ onde    \begin{equation*}    g\left( x\right) =\left\{    \begin{array}{cc}    x^{2}\sin \left( \dfrac{1}{x^{2}}\right)  & \text{se }x\neq 0 \\    0 & \text{ se }x=0    \end{array}    \right.    \end{equation*}


      1624   

      Suponha que, em uma aplicação do Método de Newton, o valor de $x_0$ escolhido coincidiu com uma raiz. Suponho que $f'(x_0)$ exista e não seja nula, o que acontecerá com $x_1$ e as aproximações subsequentes?


      1203   

      Demonstre que a derivada da função cosseno é a oposta da função seno.


      1631   

      Prove que $\displaystyle\int (sec(x))^m dx=x \dfrac{(sec(x))^{m-2}tg(x)}{m-1}+\dfrac{m-2}{m-1}\displaystyle\int (sec(x))^{m-2} dx$, para $m$ inteiro positivo.


      82   

      Calcule os limites:

      1. $\lim\limits_{x\to3} x^2-3x+7$

      2. $\lim\limits_{x\to3} x^3-3x-7$



      1. Como a função está definida em $x=3$, o limite pode ser calculado diretamente por substituição:
        $\lim\limits_{x\rightarrow 3} x^2-3x+7 = 3^2 - 3.3 + 7 = 7$.
      2. Como a função está definida em $x=3$, o limite pode ser calculado diretamente por substituição:
        $\lim\limits_{x\rightarrow 3} x^3-3x+7 = 3^3 - 3.3 - 7 = 11$.


      1496   

       Resolva os itens:

      1. Verifique que $\sqrt{1+x^2}-|x|=\dfrac{1}{|x|+\sqrt{1+x^2}}$. Conclua que à medida que $|x|$ resce a diferença $\sqrt{1+x^2}-|x|$ se aproxima de zero.
      2. Esboce o gráfico de $y=\sqrt{1+x^2}$.


      1116   

      Dado que os números no gráfico representam o valor das áreas demarcadas, avalie as seguintes integrais:

      Construa o gráfico da função 1 e da função 2 usando um único plano cartesiano disponível a seguir

      1. $\int_0^1 f(x)\ dx$
      2. $\int_0^2 f(x)\ dx$
      3. $\int_0^3 f(x)\ dx$
      4. $\int_1^2 -3f(x)\ dx$


      1. $-59$
      2. $-48$
      3. $-27$
      4. $-33$


      1257   

      Calcule a seguinte integral:
       $ \int_4^{\infty}e^{-\frac{y}{2}}dy$.


      61   

      Considere a função $ f(x)=\left\{\begin{array}{ll} 1-2x &\text{ se} x\ne -1\\0&\text{ se } x=-1\end{array}\right.$.

      1. Trace o gráfico de $f$.

      2. Usando limites laterais, determine se o limite $\lim\limits_{x\rightarrow -1}f(x)$ existe ou não.


      1780   

      Verifique que $\displaystyle \int \text{cotg} (x) \, dx = \ln |\sin x| + k$.


      805   

      Calcule $f^{\prime }\left( x\right)$:

      $f\left( x\right) =xe^{x}\cos x$.


      $f'(x) = e^x ((x+1) \cos x - x \sin x)$.



      Usando a regra da derivada do produto de duas funções, escolhendo considerar $x e^x$ como uma delas e, consequentemente, $\cos x$ como a outra, obtemos:

      \[ (x e^x \cos x)^\prime = (x e^x)^\prime \cdot \cos(x)+ x e^x \cdot (\cos x)^\prime .\]

      Para calcular $(x e^x)^\prime$, vamos usar novamente a regra da derivada do produto:

      \[(x e^x)^\prime = (x^\prime) \cdot e^x + x\cdot (e^x)^\prime = e^x(1+x),\]

      em que usamos que $(x)^\prime=1$ e $(e^x)^\prime=e^x$, além de colocar em evidência o fator comum $e^x$.

      Substuindo essas expressões na igualdade inicial, temos que

      \[ (x e^x \cos x)^\prime = e^x(1+x)\cos(x)  - x e^x \sin x,\]

      já que $(\cos x)^\prime = -\sin x$. Ou seja, obtivemos que

      \[f'(x) = e^x ((x+1) \cos x - x \sin x).\]


      347   

      Encontre o número de polinômios de grau $5$ com coeficientes distintos pertencentes ao conjunto $\{1,2,\ldots,9\}$ que são divisíveis por $x^2-x+1$.


      959   

      Calcule o seguinte limite, caso exista:

      $\lim\limits_{x\rightarrow 0}\dfrac{\sin \left( \pi x\right) }{\sin\left( 23x\right) }$



      $\begin{array}{rcl} \lim\limits_{x\rightarrow 0}\dfrac{\sin \left( \pi x\right) }{\sin\left( 23x\right) } &=& \lim\limits_{x\rightarrow 0} \dfrac{ \dfrac{\sin \left( \pi x\right)}{x} }{ \dfrac{\sin\left( 23x\right)}{x} } \\ &=& \lim\limits_{x\rightarrow 0} \dfrac{ \dfrac{\pi \sin \left( \pi x\right)}{\pi x} }{ \dfrac{23\sin\left( 23x\right)}{23x} } \\ &=& \lim\limits_{x\rightarrow 0} \dfrac{\pi}{23} \dfrac{ \dfrac{ \sin \left( \pi x\right)}{\pi x} }{ \dfrac{\sin\left( 23x\right)}{23x} }. \end{array}$

      Fazendo as mudanças de variáveis $y = \pi x$ e $t = 23x$, temos que 

      $\lim\limits_{x\rightarrow 0}\dfrac{\sin \left( \pi x\right) }{\pi x} = \lim\limits_{y\rightarrow 0}\dfrac{\sin \left( y\right) }{y} = 1 $.

      $\lim\limits_{x\rightarrow 0}\dfrac{\sin \left( 23 x\right) }{23 x} = \lim\limits_{y\rightarrow 0}\dfrac{\sin \left( t\right) }{t} = 1 $.

      Onde nas últimas passagens usamos o limite fundamental do seno. Desse modo, sabendo que os limites existem, podemos substituí-los na expressão anterior:

      $\begin{array}{rcl} \lim\limits_{x\rightarrow 0}\dfrac{\sin \left( \pi x\right) }{\sin\left( 23x\right) } &=& \lim\limits_{x\rightarrow 0} \dfrac{\pi}{23} \dfrac{ \dfrac{ \sin \left( \pi x\right)}{\pi x} }{ \dfrac{\sin\left( 23x\right)}{23x} } \\ &=& \dfrac{\pi}{23} \dfrac{1}{1} \\ &=& \dfrac{\pi}{23}. \end{array}$


      181   

      Seja o número inteiro $AB$, no qual $A$ e $B$ são os algarismos das dezenas e das unidades, respectivamente. Invertendo-se a posição dos algarismos $A$ e $B$, obtém-se um número que excede $AB$ em $27$ unidades. Se $A+B$ é um quadrado perfeito, qual o valor de $B$?



      Temos que "$BA-AB$"$=10B+A-10A-B=9B-9A$. De acordo com o enunciado essa diferença é igual a $27$. Logo, $B-A=3$. Temos, portanto,  $7$ possibilidades: "$BA$"$=30, 41, 52, 63, 74, 85$ ou $96$. Dentre essas possibilidades, a única em que $A+B$ é um quadrado perfeito é o caso $B=6$, $A=3$.


      1699   

      Determine a área da superfície gerada pela rotação da curva a seguir em torno do eixo indicado.

      $x=\frac{(e^y+e^{-y})}{2}$,  $0\leq y \leq ln\ 2$, eixo $y$


      1615   

      Calcule o limite $\lim\limits_{x \to \infty}\dfrac{\ln({\sin(x)})}{\ln({\sin(2x)})}$.


      1706   

      Embora limites como $\displaystyle \lim_{n \to \infty} \sqrt[\leftroot{-2}\uproot{2}n]{n}$ e $\displaystyle \lim_{n \to \infty} a^n$ possam ser avaliados utilizando conhecimentos sobre as funções logaritmo e exponencial, estes não são necessários. Neste exercício vamos calcular esses tipos de limite por meio de argumentos ``elementares''. As ferramentas básicas são desigualdades provenientes do teorema binomial, principalmente:

      $$(1+h)^n \geq 1+nh, \text{ para } h > 0.$$

      1. Mostre que $\displaystyle \lim_{n \to \infty} a^n = \infty$ se $a>1$, fazendo $a=1+h$, onde $h>0$.

      2. Mostre que $\displaystyle \lim_{n \to \infty} a^n= 0$ se $0<a<1$.


      604   

      Encontre todos os números reais que satisfazem cada uma das desigualdades abaixo. Dê o intervalo solução e ilustre a solução sobre a reta real.

      1. $2\leq {\frac{2}{3x-1}}\leq {\frac{20}{3}}$

      2. ${\frac{1}{2x+3}}\leq {\frac{x-1}{3}}\leq {\frac{1}{5}}$


      1675   

      Calcule a integral a seguir utilizando decomposição de quocientes em frações parciais:

      $\int{\frac{dx}{1-x^2}}$



      Podemos escrever:

      $\frac{1}{1-x^2}=\frac{A}{1-x}+\frac{B}{1+x}=\frac{A+Ax+B-Bx}{1-x^2}=\frac{(A+B)+(A-B)x}{2-x^2}$

      Portanto, sabemos que $A+B=1$ e $A-B=0$. Temos, portanto, $A=B=\frac{1}{2}$.

      Assim, reescrevemos a integral como

      $\int\left(\frac{1/2}{1-x}+\frac{1/2}{1+x}\right)\,dx=\frac{1}{2}ln(1+x)+\frac{1}{2}ln(1-x)$


      1833   

      O princípio de Fermat também explica por que um raio de luz passando entre ar e água sofre um desvio de trajetória (refração). Imagine dois meios uniformes (como ar e água) e um raio de luz viajando de uma fonte $A$ em um meio para um observador $B$ em outro meio (figura abaixo). Sabe-se que a luz viaja a uma velocidade constante em um meio uniforme, porém mais vagarosamente no meio mais denso (como a água) do que no meio menos denso (como o ar). Conseqüentemente, o percurso de menor tempo entre $A$ e $B$ não é necessariamente uma reta, mas a união de dois segmentos $AP$ e $PB$, permitindo assim que a luz tome vantagem de sua maior velocidade no meio mais esparso. A Lei de Refração de Snell estabelece que a trajetória do raio de luz é tal que $$ \dfrac{\sin\theta_1}{\nu_1}= \dfrac{\sin\theta_2}{\nu_2}, $$ onde $\nu_1$ é a velocidade da luz no primeiro meio  e $\nu_2$ no segundo, $\theta_1$ e $\theta_2$ são os ângulos de incidência e de refração, respectivamente (figura abaixo). Mostre que isso decorre da hipótese de que o caminho de tempo mínimo ocorre quando $\displaystyle dt/dx=0$.

      Construa o gráfico da função 1 e da função 2 usando um único plano cartesiano disponível a seguir


      1811   

      As funções da forma $$f(x)=cx^ne^{-x},\quad x>0,$$ onde $n$ é um inteiro positivo e $c=1/n!$ surgem no estudo estatístico do fluxo de tráfego.

      1.  Use um recurso gráfico computacional  para gerar o gráfico de $f$ com $n=2,3,4$ e $5$ e faça uma conjectura sobre o número e a localização dos extremos relativos de $f$.

      2.  Confirme a sua conjectura usando o teste da derivada primeira.


      1113   

      Encontre $f(x)$ que satisfaça o seguinte problema de valor inicial:
        $f'(x) = \sin x$ e $f(0)= 2$


        $-\cos x+3$


      1779   

      Seja $f(x)=\dfrac{1}{1+x}$. Determine:

      1. $f(f(x))$
      2. $f\left(\dfrac{1}{x}\right)$
      3. $f(cx)$
      4. $f(x+y)$
      5. $f(x)+f(y)$


      1175   

      O que se pode dizer sobre os pontos de inflexão de uma curva cúbica? Justifique.


      98   

      Calcule os limites:

      1. $\lim\limits_{h\rightarrow 0}\left( x^{2}+5xh^{2}\right) $
      2. $\lim\limits_{x\rightarrow 2}\dfrac{1/x-1/2}{x-2}$


      107   

      Mostre que a função $f\left( x\right) =x^{n}$ é contínua em seu domínio.



      O domínio da função é $\mathbb{R}$. Logo, para $x \in \mathbb{R}$, temos:

      $\lim_\limits{x \to a} x^n = a^n$

      e

      $f(a) = a^n$.

      Isto é, $\lim_\limits{x \to a} f(x) = f(a)$, e portanto a função é contínua.


      208   

      Utilizando o gráfico a seguir, avalie os seguintes limites

      Construa o gráfico da função 1 e da função 2 usando um único plano cartesiano disponível a seguir

      1. $ \lim\limits_{x\to 1^-} f(x)$
      2. $ \lim\limits_{x\to 1^+} f(x)$
      3. $ \lim\limits_{x\to 1} f(x)$
      4. $f(1)$


      1. $2$
      2. $0$
      3. Não existe.
      4. $1$


      1751   

      1. Utilizando somas superiores, mostre que a área sob o gráfico de $y=x^3$ no intervalo $[0,b]$ é $b^4/4$.

      2. Mostre o mesmo resultado utilizando somas inferiores.


      1174   

      O que se pode dizer sobre os pontos de inflexão de uma curva quadrática? Justifique.



      Os pontos de inflexão são os pontos nos quais a curvatura de uma curva troca de sinal, portanto, está associado com trocas de sinal da segunda derivada da função associada à curva. Curvas quadráticas terão derivadas lineares e segundas derivadas constantes. Sendo assim, como a segunda derivada de uma curva quadrática nunca trocará de sinal, uma curva quadrática nunca apresentará pontos de inflexão


      1134   

      Suponha que $x(t)=e^{0,05t}$ e que $z(t)=e^{0,01t}$. Calcule a taxa de crescimento de $y(t)$ nos seguintes casos:

      1.    $y=x$  
      2.    $y=z$
        


      1198   

      Calcule a derivada da seguinte função:
       $f\left(  x\right)  =\frac{\sqrt{x^{3}+1}}{\left(  x^{2}+1\right)  ^{4}}.$


      1608   

      A média geométrica de dois números reais positivos $a$ e $b$ é definida como $\sqrt{ab}$. Prove que $\sqrt{ab}=\lim\limits_{x \to \infty}\left(\dfrac{a^{1/x}+b^{1/x}}{2}\right)^x$.


      1816   

      Calcule $F'(x)$ sendo $F(x)$ igual a:

      1. $xe^x\cos{x}$.
      2. $e^x \sin{x} \cos{x}$.


      1. $-e^x x^2 \sin (x)+e^x x^2 \cos (x)+2 e^x x \cos (x)$.
      2. $-e^x \sin ^2(x)+e^x \cos ^2(x)+e^x \sin (x) \cos (x)$.


      653   

      Esboce o gráfico de cada uma das funções abaixo.

      1. $y=-\sqrt{7-x^{2}}$

      2. $y=1+\sqrt{10-x^{2}}$


      1787   

      1.  Mostre que se $f$ e $g$ forem funções para as quais $$ f'(x)=g(x) \quad\text{e}\quad g'(x)=f(x)$$ para todo $x$, então $f^2(x)-g^2(x)$ é uma constante.

      2.  Mostre que as funções $\displaystyle f(x)=\dfrac{1}{2}(e^x+e^{-x})$ e $\displaystyle g(x)=\dfrac{1}{2}(e^x-e^{-x})$  têm esta propriedade.


      1803   

      Se $f$ e $g$ são funções contínuas tais que $0 \leq f(x) \leq g(x)$, para $x\geq a$, temos:
      Se $\displaystyle \int_a^{+\infty} f(x) \, dx$ diverge, então $\displaystyle \int_a^{+\infty} g(x) \, dx$ diverge.

      Se $\displaystyle \int_a^{+\infty} g(x) \, dx$ converge, então $\displaystyle \int_a^{+\infty} f(x) \, dx$ converge e $\displaystyle \int_a^{+\infty} f(x) \, dx \leq \int_a^{+\infty} g(x) \, dx$.

      1. Mostre (graficamente e algebricamente) que para $x \geq 1$, temos $e^{-x^2} \leq e^{-x}$.

      2. Calcule a integral $\displaystyle \int_1^{+\infty} e^{-x}\, dx$.

      3. O que podemos afirmar sobre a integral $\displaystyle \int_1^{+\infty} e^{-x^2}\, dx$?


      1662   

      Uma força de retardamento freia o movimento de uma massa presa a uma mola alinhada com o eixo $y$, de modo que a posição da massa no instante $t$ é
      $y=3 e^{-t}\cos\ t,\ \ t\geq 0$.

      Calcule o valor médio de $y$ no intervalo $ 0 \leq y \leq 2\pi$


      Aproximadamente $0,2383$.


      1494   

      Dê o domínio e esboce o gráfico das seguintes funções:

      1.  $f(x)=1/x^2$
      2.  $f(x)=\dfrac{1}{(x-1)^2}$


      750   

      Calcule o seguinte limite:
      $\lim\limits_{x\rightarrow 0^{+}}\log _{\dfrac{1}{3}}x$.


      $\infty$.


      146   

      Para a função a seguir, dê os intervalos nos quais ela é contínua:

       $ f(x) = \sin(e^x+x^2)$.


      $(-\infty,\infty)$


      786   

      Calcule os valores de $a,b$ e $c$ de modo que as parábolas $y=x^2+ax+b$ e $y=-x^2 +cx$ sejam tangentes uma a outra no ponto $(1,2)$.


      1793   

      Utilize uma substituição trigonométrica para mostrar que $\displaystyle \int \dfrac{u^2}{\sqrt{u^2 - a^2}} \, du = \dfrac{u}{2}\sqrt{u^2-a^2}+\dfrac{a^2}{2} \ln | u + \sqrt{u^2-a^2} | + C $.


      909   

      Resolva a inequação $|ax-b|<r$ na variável x, com $r>0$ e $a\neq 0$.



      Se $ax-b\geq0$: $|ax-b| = ax-b$, logo $ax-b<r \Rightarrow x < \dfrac{b+r}{a}$.
      Se $ax-b<0$: $|ax-b| = -(ax-b)$, logo $-ax+b<r \Rightarrow x > \dfrac{b-r}{a}$.
      Portanto $x<\dfrac{b+r}{a}$ ou $x>\dfrac{b-r}{a}$.


      1599   

      Um termômetro de mercúrio demorou $14s$ para subir de $-19° C$ para $100° C$ após ser retirado de um congelador e colocado em água fervendo. Considerando que, no termômetro em questão, a distância entre dois graus subsequentes é de $1mm$, demonstre que em algum instante a coluna de mercúrio subia a $8,5mm/s$.


      637   

      Dada a função $f\left( x\right) =$ $\left| x\right| -2x$, calcule $f\left( -1\right) $, $f\left( 1/2\right) $, $f\left( -2/3\right) $. Mostre que $f\left( \left| a\right| \right) =-\left| a\right| $.


      544   

      Esboce o gráfico da função $f\left( x\right) =\frac{x^{2}}{x-1}$, indicando domínio de definição, limites laterais e no infinito, assíntotas verticais e inclinadas, intervalos de crescimento e decrescimento e estudo da concavidade.


      1564   

      O efeito da luz sobre a taxa de fotossíntese pode ser descrito por $f(x)=x^a e^{(a/b)(1-x^b)}$ para $x>0$ e constantes positivas $a$ e $b$.

      Mostre que $f$ tem um máximo em $x=1$.

      Conclua que, se $x_0>0$ e $y_0>0$, então $g(x)=y_0f(x/x_0)$ tem máximo em $g(x_0)=y_0$.


      553   

      Estude o sinal de $f^{\prime }\left( x\right) $, calcule os limites $\lim\limits_{x\rightarrow \infty }f\left( x\right) $ e $\lim\limits_{x\rightarrow -\infty }f\left( x\right) $ e, utilizando esses dados, esboce o gráfico de $f\left( x\right) =\dfrac{x}{x^{2}+1}$.


      1304   

      Determine os valores de $\lambda$ que tornam contínua a função

      \begin{equation*} f\left(  x\right)  =\left\{ \begin{array} [c]{c} x^{2}+cx\text{ se }x\leq1\\ \left(  cx\right)  ^{2}-1=c^{2}x^{2}-1\text{ se }x>1 \end{array} \right.  \text{.} \end{equation*}


      1171   

      Utilize o Teorema de Valor Médio (ou o caso particular do Teorema de   Rolle) para mostrar que, para qualquer valor de $c\in\mathbb{R}$, o   polinômio $p\left(  x\right)  =x^{4}+4x+c$ tem no máximo duas raízes reais.


      331   

      Uma das raízes da equação $x^2+mx+m^2-m-12=0$ é nula, e a outra é positiva. Qual o valor do parâmetro $m$?


      838   

      Derive a função $f\left( x\right) =\left( 3^{2x+3}\right)\sqrt{\cos \left( x^{3}+x^{1/3}\right) }.$


      $ 2.3^{2 x + 3} \sqrt{\cos(x^3 + x^{1/3})} \log 3 - (3^{2 x + 3} (1/(3 x^{2/3}) + 3 x^2) \sin(x^3 + x^{1/3}))/(2 \sqrt{cos(x^3 + x^{1/3})})$.


      1791   

      Calcule $\displaystyle \int \dfrac{1}{2+\sin x} \, dx$.


      $\frac{2 \tan ^{-1}\left(\frac{2 \tan \left(\frac{x}{2}\right)+1}{\sqrt{3}}\right)}{\sqrt{3}}$


      1463   

      Prove que $|x+y|=|x|+|y| \Leftrightarrow xy \geq 0$.


      194   

      Dados os números naturais $a, b$, prove que existe um número natural $m$ tal que $m \cdot a > b$.

      A teoria necessária para resolver esta questão pode não ser abordada em alguns cursos de Cálculo 1. Sendo, também pertinente, às disciplinas Teoria dos Números e Análise Real I.

      Para aprofundar seus conhecimentos, dentro do escopo de Cálculo 1, recomendamos a leitura do Cap. 1 de Guidorizzi, vol. 1 e /ou o Prólogo de Spivak (vide Bibliografia de Cálculo 1).


      1123   

      Com base no gráfico, avalie as seguintes integrais:

      Construa o gráfico da função 1 e da função 2 usando um único plano cartesiano disponível a seguir

      1. $\int_0^1 (x-1)\ dx$
      2. $\int_0^2 (x-1)\ dx$
      3. $\int_0^3 (x-1)\ dx$
      4. $\int_2^3 (x-1)\ dx$
      5. $\int_1^4 (x-1)\ dx$
      6. $\int_1^4 \big((x-1)+1\big)\ dx$


      1. $-1/2$
      2. $0$
      3. $3/2$
      4. $3/2$
      5. $9/2$
      6. $15/2$


      724   

      Calcule o limite:


      $\lim\limits_{x\rightarrow -\infty }\left( x-\sqrt{x^{2}+4x}\right)$.


      $-\infty$.


      732   

      Encontre a área limitada pela elipse $\frac{x^{2}}{4}+\frac{y^{2}}{9}=1\text{.}$


      1743   

      Escreva o número $\sin 1/2$ como uma soma (com a notação $\Sigma$), com um erro menor que $10^{-20}$.


      912   

      Resolva a inequação $\frac{x^2+2x-1}{x^2-1} \geq \frac{1}{x+1}$.


      1176   

      Mostre que $x^{2}-xy+y^{2}\geq 0$, $\forall x,y\in R$ e que vale a igualdade se e somente se $x=y=0$.


      1266   

      Calcule a seguinte integral:
         $\int{x\cos x dx}.$


      $xsinx+cosx+C$


      1655   

      Quais valores de $a$ e $b$ minimizam o valor de

      $\int_a^b\left(x^4-2x^2\right)dx$?


      91   

      Mostre $\lim\limits_{x\to 0} \frac{x+1}{x^2+3x}$ não existe.


      142   

      Para a função a seguir, dê os intervalos nos quais ela é contínua:

       $ f(x) = e^x$.


      668   

      Calcule a integral $\int x^{2}\ln xdx$.


      $\dfrac{1}{9}x^3(3lnx-1)+C$.


      1789   

      Prove que a função $f(x)=\left\{\begin{array}{ll}
      x, & \text{se x é racional}\\
      -x, & \text{se x é irracional}
      \end{array}\right.$ é contínua em $0$.


      1507   

      Demonstre que $x^{ln(2)}=2^{ln(x)}$ utilizando propriedades de logaritmos e exponenciais. Utilizando recursos computacionais, observe os gráficos das duas funções, assim como a diferença entre elas. Qual seria uma explicação para o comportamento observado no gráfico de $f(x)=x^{ln(2)}-2^{ln(x)}$?


      1204   

      Demonstre que a derivada da função tangente é igual ao quadrado da função secante.


      1586   

      Dados $f(x) =\sin^{-1}x$ e $x_0 = \pi/12$, escolha um valor inteiro próximo a $x_0$ tal que $f(x_0)$ e $f'(x_0)$ sejam fáceis de calcular, e calcule uma linearização da função neste ponto.


      1912   

      Usando a fórmula do volume de uma calota esférica, encontre o volume do sólido que sobra quando um buraco de raio $\dfrac{r}{2}$ é feito através do centro de uma esfera de raio $r$ e verifique a sua resposta por integração. 


      1921   

      Mostre que a área da superfície de uma esfera de raio $r$ é $4 \pi r^2$.


      676   

      Mostre que a equação $x^2=x$ tem exatamente duas raízes reais.



      A equação pode ser escrita na forma $x^2-x=0$, i.e, $x(x-1)=0$. As suas únicas raízes reais são $x=0$ e $x=1$. Uma outra forma de atacar este problema é perceber que os gráficos de $f(x)=x$ e $g(x)=x^2$ se intersectam exatamente duas vezes!


      810   

       

      Calcule $f^{\prime }\left( x\right)$:

        $f\left( x\right) =\dfrac{\sqrt{x}}{x+1}$.


      $f'(x) = \dfrac{1-x}{2\sqrt{x}(x+1)^2}$.



      Queremos calcular a derivada da divisão da função $\sqrt{x}$ pela função $x+1$. Usando a regra da derivada do quociente, obtemos:

      \[\left( \dfrac{\sqrt{x}}{x+1} \right)^\prime = \dfrac{(\sqrt{x})^\prime \cdot (x+1) - \sqrt{x}\cdot (1+x)^\prime}{(x+1)^2}.\]

      Sabendo que

      \[(\sqrt{x})^\prime = \left(x^{1/2}\right)^\prime = \dfrac{1}{2} x^{\left(\tfrac{1}{2}-1\right)} = \dfrac{1}{2 \sqrt{x}}\]

      e que

      \[(x+1)^\prime = (x)^\prime + (1)^\prime = 1 + 0 = 1,\]

      podemos usar essas expressões na regra do quociente e, assim, obter que

      \[\dfrac{(\sqrt{x})^\prime \cdot (x+1) - (\sqrt{x})\cdot (1+x)^\prime}{(x+1)^2} = \dfrac{\dfrac{1}{2 \sqrt{x}}(x+1)-\sqrt{x}(1)}{(x+1)^2} = \dfrac{\dfrac{x}{2 \sqrt{x}} +\dfrac{1}{2 \sqrt{x}} -\dfrac{x}{\sqrt{x}}}{(x+1)^2}.\]

      Disso, podemos concluir que

      \[f'(x) = \dfrac{1-x}{2\sqrt{x}(x+1)^2}.\]


      348   

      Demonstre a fórmula de Báskhara usada para resolução de equações polinomiais de grau $2$.


      1091   

      A antiderivada de uma função aceleração é a função _________.



      Velocidade. A taxa de variação com a qual a velocidade varia de acordo com o tempo é, justamente, a aceleração.


      1705   

      Se você fosse um professor e seu(sua) aluno(a) te perguntasse ``Por que $\displaystyle \lim_{x \to 0}\dfrac{\sin x}{x}=1$?''

      1. Como você responderia com palavras?

      2. Que bibliografia você recomendaria?

      3. Qual a demonstração formal?


      116   

      Responda os seguintes itens:

      1. Calcule $\displaystyle\lim_{x\rightarrow 0}\frac{\frac{2}{x} - 5\cos(\frac{1}{x^2+2x})}{-\frac{5}{x} + 2\cos(\frac{1}{x^2+2x})}$.

      2. Existe algum número real $a$ tal que a função $f(x) = \left\{\begin{array}{ccl}\displaystyle\frac{\frac{2}{x} - 5\cos(\frac{1}{x^2+2x})}{-\frac{5}{x} + 2\cos(\frac{1}{x^2+2x})},& \mbox{se} & x\neq 0\\ a, & \mbox{se} & x=0 \end{array} \right.$ seja contínua?


      163   

      Enuncie e demonstre o Teorema do Confronto.


      1179   

      Mostre que:

      1. $|x-y|<1/2, |x+2|<1/3 \Longrightarrow |y+2|<5/6$
      2. $\sqrt{xy}\leq {\frac{x+y}{2}}$, $\forall x,y\geq 0$.


      1822   

      As curvas de crescimento logístico modelam a taxa de crescimento de uma certa população em função dos fatores ambientais. Em um período prolongado de tempo, a população tende a um valor limite que representa o máximo número de indivíduos que o espaço ou alimento pode sustentar. Estas curvas são da forma $$ y(t)=\dfrac{L}{1+Ae^{-kt}}, $$ onde $y$ é a população no momento $t$ ($t\geq 0$) e $A$, $k$ e $L$ são parâmetros positivos.

      Construa o gráfico da função 1 e da função 2 usando um único plano cartesiano disponível a seguir

      Mostre que o ponto de inflexão da curva de crescimento logístico (figura acima) ocorre no tempo $t$ solução da equação $$ \dfrac{L}{2}=\dfrac{L}{1+Ae^{-kt}}, $$ para $t$, ou seja, no instante $ t= \dfrac{\ln A}{k}$. 


      1575   

      Determine $f'$, $f''$ e $f'''$ sendo $f(x)=4x^4+2/x$.


      1679   

      Calcule a seguinte integral:

      $\int_{0}^{\infty}{\frac{dx}{x^2+1}}$



      Para resolver a integral, utilizamos a substituição $x=\tan(u)$, com $dx=\frac{du}{cos^2(u)}$. A integral equivalente, com os limites de integração escolhidos no primeiro quadrante, é:

      $\int_0^{\frac{\pi}{2}}\frac{du}{\cos^2(u)\left(\tan^2(u)+1\right)}=\int_0^{\frac{\pi}{2}}\frac{du}{1}=u\rvert_0^{\frac{\pi}{2}}=\frac{\pi}{2}$


      1512   

      Sejam $f(x)=log_x(2)$ e $g(x)=log_2(x)$:

      1. Utilize a propriedade de quociente de logaritmos para expressar $f(x)$ e $g(x)$ em termos de logaritmos naturais.
      2. Com o auxílio de recursos computacionais, compare os gráficos de $f(x)$ e $g(x)$.


      1650   

      Um carro está em uma rodovia a uma velocidade constante de $60mi/h$ quando vê um acidente a frente e aciona os freios. Que desaceleração constante é necessária para frear o carro em 242 pés?


      1557   

      Um modelo de densidade urbana é uma fórmula que relaciona a densidade populacional (em número de habitantes por $km^2$) com a distância $r$ (em $km$) do centro da cidade. É considerada apropriada  para certas cidades a fórmula $D=ae^{-br+cr^2}$, com $a,b$ e $c$ constantes positivas. Determine a forma do gráfico de $D$ para $r \geq 0$.


      1760   

      Seja $F(x)$ tal que $F(x)=\displaystyle \int_{\pi/4}^x \cos 2t \, dt$.

      1. Use alguma versão do Teorema Fundamental do Cálculo para encontrar $F'(x)$.

      2. Confira se seu resultado anterior foi correto integrando e diferenciando.


      1782   

      Verifique que $\displaystyle \int \text{cotg}^n (x) \, dx = -\dfrac{\text{cotg}^{n-1} (x) }{n-1} - \int \text{cotg}^{n-2} (x) \, dx, \, n \geq 2$.


      1563   

      Seja $g(x)=a^x$, em que $a>0$ e $a \neq 1$ é um real dado. Mostre que $g'(x)=a^x \ln{a}$.


      1311   

      Avalie o limite $\lim\limits_{x\rightarrow p}\dfrac{x^{n}-p^{n}}{x-p}$, onde $n$ é qualquer número natural.


      1665   

      A aceleração de uma partícula que se move de um lado para o outro sobre uma reta é $a=d^2s/dt^2 = \pi^2\sin\pi t\ m/s^2$ para qualquer $t$. Se $s=0$ e $v=8m/s$ quando $t=0$, determine o valor de $s$ para $t=1\ s$.


      217   

      Avalie os seguintes limites para a função definida por partes
      $ f(x) = \left\{\begin{array}{ccc}
      x^2-1, & & \text{ se }  x<-1 \\
      x^3+1, & & \text{ se }  -1\leq x\leq 1\\
      x^2+1, & & \text{ se }  x>1
      \end{array}
      \right.$

      1. $ \lim\limits_{x\to -1^-} f(x)$

      2. $ \lim\limits_{x\to -1^+} f(x)$

      3. $ \lim\limits_{x\to -1} f(x)$

      4. $f(-1)$

      5. $ \lim\limits_{x\to 1^-} f(x)$

      6. $ \lim\limits_{x\to 1^+} f(x)$

      7. $ \lim\limits_{x\to 1} f(x)$

      8. $f(1)$


      1. 0
      2. 0
      3. 0
      4. 0
      5. 2
      6. 2
      7. 2
      8. 2


      90   

      Calcule, se existir, o limite $\lim\limits_{x\to 0} \frac{x+1}{x^2+3x}.$


      1657   

      Demonstre que  $2\sqrt{2} \leq \int_{0}^{1}{\sqrt{x+8}dx} \leq 8$.


      1654   

      Quais valores de $a$ e $b$ maximizam o valor de

      $\int_a^b\left(x-x^2\right)dx$?


      $a=0$ e $b=1$.


      1907   

      Um retângulo com os lados paralelos aos eixos coordenados tem um vértice na origem e o vértice diagonalmente oposto está sobre a curva $y=kx^m$ no ponto onde $x=b$ ($b>0$, $k>0$, $m \geq 0$). Mostre que o quociente entre a área do retângulo compreendida entre a curva e o eixo $x$ depende de $m$, mas não depende de $k$ ou $b$.


      1560   

      Determine a equação da reta tangente ao gráfico de $f(x)=e^x$ no ponto de abscissa $0$.


      926   

      Calcule o valor das seguintes expressões:

      1. $sen(45^0)+cos(45^0)$
      1. $\dfrac {cos(30^0)sen(60^0)} {tg(45^0)}$
      1. $[sen^2(71,2^0)+cos^2(71,2^0)] \times cotg(45^0)$



      1. Usando o teorema fundamental da trigonometria sabemos que o valor da expressão $sen(45^0)+cos(45^0)$ é $1$.
      2. Este item se resolve por substituição direta: $cos(30^0)=sen(60^0)=\dfrac{\sqrt{3}}{2}$ e $tg(45^0)=1$:$\dfrac {cos(30^0)sen(60^0)} {tg(45^0)}=\left(\dfrac{\sqrt{3}}{2}\right)^2 \times 1=\dfrac{3}{4}$.
      3. Usando o teorema fundamental da trigonometria sabemos que o valor da expressão $sen^2(71,2^0)+cos^2(71,2^0)$ é $1$. Além disso, temos que $cotg(45^0)=\dfrac{1}{tg(45^0)}=\dfrac{1}{1}=1$. Então:\\ $[sen^2(71,2^0)+cos^2(71,2^0)] \times cotg(45^0)=1 \times 1=1$.


      531   

      Se uma bola de neve derrete de tal forma que a área de sua superfície decresce a uma taxa de $1cm^{2}/\min $, encontre a taxa segundo qual o diâmetro decresce quando o diâmetro for $5 cm$.


      562   

      Estude a função $f\left( x\right) =x^{3}-3x^{2}+3x$ com relação à concavidade, pontos de inflexão, máximos e mínimos, e esboce o seu gráfico.


      326   

      Duas pequenas fábricas de calçados, $A$ e $B$, têm fabricado, respectivamente, $3000$ e $1100$ pares de sapatos por mês. Se, a partir de janeiro, a fábrica $A$ aumentar sucessivamente a produção em $70$ pares por mês e a fábrica $B$ aumentar sucessivamente a produção em $290$ pares por mês, em que mês a produção da fábrica $B$ superará a produção de $A$ pela primeira vez?


      1911   

      Considere uma calota esférica de raio $s$ e altura $h$ cortada de uma esfera de raio $r$. Mostre que o volume $V$ da calota esférica pode ser expresso como $V=\dfrac{1}{3}\pi h^2(3r-h)$ ou $V=\dfrac{1}{6}\pi h(3 s^2+h^2)$.

      Construa o gráfico da função 1 e da função 2 usando um único plano cartesiano disponível a seguir


      1314   

      Você está planejando construir uma caixa retangular aberta com uma folha de papelão de $8 \times 15 pol$ recortando quadrados congruentes dos vértices da folha e dobrando suas bordas para cima. Quais são as dimensões da caixa de maior volume que você pode fazer dessa maneira? Qual é o volume?


      666   

      Derive a função $q\left( x\right)  = \int_{e^{-2x}}^{\mathrm{tg}x}e^{\theta }\cos \theta d\theta$.


      1268   

      Calcule a seguinte integral:
        $\int  \cos(x)\ln (\sin (x))dx   $.


      $sinx(ln(sinx)-1)+C$


      923   

      Esboce o gráfico de $f(x) =x^2+6x+10.$ Use completamento de quadrados.


      646   

      Seja $f\left( x\right) =\frac{1+x}{1-x}$. Mostre que $f\left( \frac{1}{1+x}\right) =\frac{2+x}{x}$, $f\left( \frac{1}{1-x}\right) =\frac{x-2}{x}$, $f\left( -x\right) =\frac{1}{f\left( x\right) }$, $f\left( 1/x\right)=-f\left( x\right) $ e que $f\left( f\left( x\right) \right) =-1/x$.


      679   

      Sabemos que a troca de calor entre um objeto a uma temperatura $T$ e o ambiente a uma temperatura $T_{a}$ é proporcional a diferença $(T-T_{a})$. Como a variação de temperatura é proporcional a troca de calor, temos a seguinte equação diferencial para $T\left( t\right) $ (temperatura em função do tempo $t$ ):$\frac{dT}{dt}=-\alpha \left( T-T_{a}\right) ,$ onde a constante $\alpha >0$ depende do calor específico e da condutividade térmica do objeto. Ache a solução dessa equação em função de $\alpha $ assumindo que a temperatura do ambiente $T_{a}=20^{o}C$ e a temperatura inicial $T_{0}=100^{o}C$. Qual é o limite $\lim\limits_{t\rightarrow +\infty }T\left( t\right) $?


      1413   

      A aceleração (no instante $t$) de um ponto em movimento sobre uma reta coordenada é $\sin^2 (t)\cos(t)$ $m/s^2$. Em $t=0$, o ponto está na origem e sua velocidade é 10 $m/s$. Determine sua posição no instante $t$.


      568   

      Estude a função  $f\left( x\right) =x^{3}-3x^{2}-9x$ com relação à concavidade, pontos de inflexão, máximos e mínimos, e esboce o seu gráfico.


      70   

      Classifique as afirmações a seguir como verdadeiras ou falsas:

      1. Se $ \lim\limits_{x\to 5} f(x) = \infty$, então estamos implicitamente afirmando que o    limite em questão existe.

      2.  Se $ \lim\limits_{x\to \infty} f(x) = 5$, então estamos implicitamente afirmando que o     limite em questão existe.

      3.  Se $ \lim\limits_{x\to 1^-} f(x) = -\infty$, então $ \lim\limits_{x\to 1^+} f(x) = \infty$.

      4.  Se $ \lim\limits_{x\to 5} f(x) = \infty$, então $f$ tem uma assíntota vertical em $x=5$.

      5.  $\infty/0$ não é uma forma indeterminada.


      1.  Falsa.

      2. Verdadeira

      3. Falsa

      4. Verdadeira

      5. Verdadeira


      1190   

      Calcule a derivada da seguinte função: 
       $f\left(  x\right)  =\frac{e^{x}+e^{-x}}{2}.$


      $f'(x) = \frac{e^{x}-e^{-x}}{2}$.


      654   

      Esboce o gráfico de cada uma das funções abaixo.

      1. $y=2-\sqrt{16-x^{2}}$

      2. $y=-1+\sqrt{6-(x-1)^{2}}$


      154   

      Sejam  $f$  uma função contínua num intervalo  $I$,  $a$  e  $b$  valores em  $I$. Se $f(a)$ e $f(b)$ são valores com sinais contrários, mostre que a equação $f(x)=0$ tem pelo menos uma raiz real no intervalo $\left[a,b\right]$.


      1774   

      A respiração tem um ciclo rítmico que consiste em períodos alternados de inalação e exalação. Em condições normais, um adulto tem um ciclo em média a cada $5$ segundos. Denotando por $V$ o volume de ar nos pulmões no instante $t$, a taxa de fluxo é dada por $\dfrac{dV}{dt}$.

      1. Se a taxa máxima de fluxo é $0,6$ L$/$seg, encontre valores de $a$ e $b$ para que a fórmula $\dfrac{dV}{dt}=a \sin (bt)$ modele a respiração com os dados acima.

      2. Utilize a expressão obtida para estimar a quantidade de ar inalada durante um ciclo completo de respiração.


      173   

      Classifique cada uma das afirmações abaixo em verdadeiras ou falsas.

      1. Nem todo primo é ímpar.

      2. Todo inteiro par pode ser escrito na forma $n^2+2, n \in N$. 

      3. A soma de dois inteiros ímpares é sempre um inteiro par.

      4. Todo inteiro ímpar pode ser escrito na forma $2n-9, n \in N$. 

      5. Se $n$ é um inteiro ímpar, então $n^2$ também é ímpar.


      1. V

      2. F

      3. V

      4. V

      5. V


      1165   

      Sabemos que a troca de calor entre um objeto a uma temperatura $T$ e o ambiente a uma temperatura $T_{a}$ é proporcional a diferença $(T-T_{a})$. Como a variação de temperatura é proporcional a troca de calor, temos a seguinte equação diferencial $\frac{dT}{dt}=-\alpha \left( T-T_{a}\right)$ para $T\left( t\right) $ (temperatura em função do tempo. A  constante $\alpha >0$ depende do calor específico e da condutividade térmica do objeto. Ache a soluçao dessa equação em função de $\alpha $ assumindo que a temperatura do ambiente $T_{a}=20^{o}C$ e a temperatura inicial $T_{0}=100^{o}C$. Qual é o limite $\lim_{t\rightarrow +\infty }T\left( t\right) $?


      908   

      Resolva a equação $|2x+1|=3$.



      Temos dois casos: $2x+1=3$ ou $2x+1=-3$. Resolvendo cada uma dessas equações de primeiro grau obtemos $x=1$ e $x=-2$.


      1812   

      (Teste da Derivada Primeira) Suponha $f$ contínua em um ponto crítico $x_0$.

      1.  Se $f'(x)>0$ em um intervalo aberto ampliando-se à esquerda de $x_0$ e $f'(x)<0$ em um intervalo aberto ampliando-se à direita de $x_0$, então $f$ tem um máximo relativo em $x_0$.

      2.  Se $f'(x)<0$ em um intervalo aberto  ampliando-se à esquerda de $x_0$ e $f'(x)>0$ em um intervalo aberto ampliando-se  à direita de $x_0$, então $f$ tem um mínimo relativo em $x_0$. 

      3.  Se $f'(x)$ tiver o mesmo sinal $\displaystyle [f'(x)>0\ \text{ou}\ f'(x)<0]$ em um intervalo aberto ampliando-se à esquerda de $x_0$ e em um intervalo aberto ampliando-se à direita de $x_0$, então $f$ não tem extremo relativo em $x_0$.

      Esboce algumas curvas para mostrar que as três partes do teste da derivada primeira podem ser falsas, sem a hipótese de que $f$ é contínua em $x_0$.


      565   

      Estude a função   $f\left( x\right) =xe^{-3x}$ com relação à concavidade, pontos de inflexão, máximos e mínimos, e esboce o seu gráfico.


      899   

      Esboce o gráfico da função $f(x)=||(x-1)^2-3|-1|$.


      887   

      Resolva a inequação $|ax-b|<r$ na variável x, com $r>0$ e $a\neq 0$.



      Se $ax-b\geq0$: $|ax-b| = ax-b$, logo $ax-b=r \Rightarrow x = \dfrac{b+r}{a}$.
      Se $ax-b<0$: $|ax-b| = -(ax-b)$, logo $-ax+b=r \Rightarrow x = \dfrac{b-r}{a}$.
      Portanto $x=\dfrac{b+r}{a}$ ou $x=\dfrac{b-r}{a}$.


      1488   

      Prove que a soma de um racional com um irracional é um irracional.


      1119   

      Dado que os números no gráfico representam o valor das áreas demarcadas, avalie as seguintes integrais:

      Construa o gráfico da função 1 e da função 2 usando um único plano cartesiano disponível a seguir

      1. $ \int_{0}^{2} 5x^2\ dx$
      2. $ \int_0^2 (x^2+3)\ dx$
      3. $ \int_{1}^3 (x-1)^2\ dx$
      4. $ \int_2^4 \big((x-2)^2+5\big)\ dx$


      1. $40/3$
      2. $26/3$
      3. $8/3$
      4. $38/3$


      49   

      Avalie os seguintes limites de acordo com o gráfico da função:

      $f(x) = \frac{1}{e^x+1}$

      Construa o gráfico da função 1 e da função 2 usando um único plano cartesiano disponível a seguir

      1. $\lim\limits_{x\to -\infty} f(x)$

      2. $\lim\limits_{x\to \infty} f(x)$

      3. $\lim\limits_{x\to 0^-} f(x)$

      4. $\lim\limits_{x\to 0^+} f(x)$


      1691   

      Determine a área da região no primeiro quadrante limitada à esquerda pelo eixo $y$, abaixo pela curva $x=2\sqrt{y}$, acima à esquerda pela curva $x=\left(y-1\right)^2$ e acima à direita pela reta $x=3-y$.

      Construa o gráfico da função 1 e da função 2 usando um único plano cartesiano disponível a seguir



      Primeiramente, devemos escrever as curvas na forma $y=f(x)$, tomando cuidado com o sinal. Após este procedimento, uma análise da figura nos permite resumir o cálculo da área $A$ como $A=A_1+A_2$, sendo que:

      $A_1=\int_0^1\left (1+\sqrt{x}-\frac{1}{4}x^2\right)\,dx$

      $A_2=\int_1^2\left (3-x-\frac{1}{4}x^2\right)\,dx$

      Assim, temos

      $A_1=\left.\left(x + \frac{2}{3} x^{3/2} - x^3/12\right)\right\vert_0^1=\frac{19}{12}$

      $A_2=\left.\left(-\frac{x^3}{12}-\frac{x^2}{2}+3 x\right)\right\vert_1^2=\frac{11}{12}$

      O que nos leva a $A=\frac{5}{2}=2.5$


      558   

      Determine os intervalos de decrescimento e crescimento e esboce o gráfico da seguinte função  $f\left( x\right) =\dfrac{\ln x}{x}$.


      1499   

      Esboce juntas as curvas dadas no plano cartesiano e identifique cada uma com sua equação:
      $y=3^x$, $y=8^x$,$y=2^{-x}$, e $y=\left( 1/4 \right)^{x}$.


      1281   

      Seja $a$ um número real positivo e suponha que $|x|<a.$ Use o método de frações parciais para obter a fórmula $\int \frac{1}{a^2-x^2}dx=\frac{1}{2a}\ln\left(\frac{a+x}{a-x}\right) +C.$


      342   

      Encontre as raízes do polinômio $x^4-10x^3+17x^2-17x+6.$
      Sugestão: Utilize o teste das raízes racionais.


      344   

      Um ponto no plano cartesiano é chamado ponto misto se uma de suas coordenadas é racional e a outra irracional. Encontre todos os polinômios com coeficientes reais tais que seus gráficos não contêm nenhum ponto misto.


      1300   

      Como o parabolóide é obtido pela rotação ao redor do eixo $y$ temos que o raio $r:=d/2=4$ corresponde a variação de $x$ e a altura corresponde a variação de $y$, ou seja, para $x=4$ devemos ter $y=ax^{2}=4$ donde obtemos que $a=y/x^{2}=4/4^{2}=1/4$, e consideramos a parábola $y=\frac{x^{2}}{4}$.



      Como o parabolóide é obtido pela rotação ao redor do eixo $y$ devemos considerar $x$ como funçao de $y$, ou seja, $x=x\left(  y\right)  =2\sqrt{y}$. Temos então que:
        \begin{align*}
        x^{\prime}\left(  y\right)    & =\frac{1}{\sqrt{y}}\\
        \sqrt{1+\left(  x^{\prime}\left(  y\right)  \right)  ^{2}}  & =\sqrt
        {1+\frac{1}{y}}=\frac{\sqrt{y+1}}{\sqrt{y}}%
        \end{align*}
        Temos então que a área $S$ da superfície é dada por:
        \begin{align*}
        S  & =\int_{0}^{4}2\pi\left(  2\sqrt{y}\right)  \frac{\sqrt{y+1}}{\sqrt{y}%
        }dy\\
        & =4\pi\int_{0}^{4}\sqrt{y+1}dy
        \end{align*}
        Substituindo $u=y+1,~du=dy$ obtemos:
        \begin{align*}
        S  & =4\pi\int_{1}^{5}\sqrt{u}du\\
        & =4\pi\frac{2}{3}\left.  u^{3/2}\right\vert _{1}^{5}\\
        & =\frac{8\pi}{3}\left(  5^{3/2}-1\right)
        \end{align*}


      849   

      Calcule a derivada da função:

      $y=\ln \left(\dfrac{\cos \sqrt{x}}{1+\sin \sqrt{x}}\right)$.


      $y'=(\sin(\sqrt{x}) + 1) \sec(\sqrt{x}) \left(-\dfrac{\sin(\sqrt{x})}{2 \sqrt{x} (\sin(\sqrt{x}) + 1)} - \dfrac{\cos^2(\sqrt{x})}{2 \sqrt{x} (\sin(\sqrt{x}) + 1)^2}\right)$.


      198   

      Mostre que $Q$ é um conjunto enumerável.

      A teoria necessária para resolver esta questão pode não ser abordada em alguns cursos de Cálculo 1. Sendo, também pertinente, às disciplinas Teoria dos Números e Análise Real I.

      Para aprofundar seus conhecimentos, dentro do escopo de Cálculo 1, recomendamos a leitura do Cap. 1 de Guidorizzi, vol. 1 e /ou o Prólogo de Spivak (vide Bibliografia de Cálculo 1).


      Dica: Pesquise sobre a diagonal de Cantor!


      93   

      Calcule o limite $\lim\limits_{x\to -1} \frac{x^2+8x+7}{x^2+6x+5}$.


      $3/2$


      1334   

      Considere a área entre a curva $y=x^{4}$ e o eixo $x$, primeiro no intervalo $\left[ -1,1\right] $ e depois no intervalo $\left[1,a\right] $. Determinar $a\geq 1$ tal que estas áreas sejam iguais.


      1124   

      Com base no gráfico, avalie as seguintes integrais:

      Construa o gráfico da função 1 e da função 2 usando um único plano cartesiano disponível a seguir

      1. $\int_0^2 f(x)\ dx$
      2. $\int_2^4 f(x)\ dx$
      3.  $\int_0^4 f(x)\ dx$
      4. $\int_0^4 5f(x)\ dx$


      1. $\pi$
      2. $\pi$
      3. $2\pi$
      4. $10\pi$


      571   

      Estude a função $f\left( x\right) =x\ln x$ com relação à concavidade, pontos de inflexão, máximos e mínimos, e esboce o seu gráfico.


      755   

      Dê um exemplo de uma função definida em $\mathbb{R}$ que não seja contínua em $0$ mas que $\lim\limits_{x\rightarrow0^{+}}f\left( x\right) =\lim\limits_{x\rightarrow 0^{-}}f\left( x\right) .$


      1289   

      Utilize a fórmula

        \[
        s\left(  x\right)  =\int_{a}^{x}\sqrt{1+\left(  f^{\prime}\left(  t\right)
        \right)  ^{2}}dt
        \]
      para mostrar que o perímetro de uma circunferência de raio $R$ é $2\pi R$.



      Uma circunferência de raio $R$ centrada na origem pode ser vista como a união dos gráficos das funções $f\left(t\right)  =\sqrt{R^{2}-t^{2}}$ e  $g\left(  t\right)  =-\sqrt{R^{2}-t^{2}}$, com $t\in\left[  -R,R\right]  $ Por simetria, estes dois arcos têm o mesmo comprimento, digamos $L$, e o perímetro $p$ é dado por $p=2L$.
        Considerando $f\left(  t\right)  =\sqrt{R^{2}-t^{2}}$ temos
        que:
        \[
        f^{\prime}\left(  t\right)  =-\frac{t}{\sqrt{R^{2}-t^{2}}}\text{.}%
        \]
        Usando a fórmula acima temos que:
        \begin{align*}
        L  & =\int_{-R}^{R}\sqrt{1+\left(  f^{\prime}\left(  t\right)  \right)  ^{2}%
        }dt\\
        & =\int_{-R}^{R}\sqrt{1+\frac{t^{2}}{R^{2}-t^{2}}}dt\\
        & =\int_{-R}^{R}\sqrt{\frac{\left(  R^{2}-t^{2}\right)  +t^{2}}{R^{2}-t^{2}}%
        }dt\\
        & =\int_{-R}^{R}\sqrt{\frac{R^{2}}{R^{2}-t^{2}}}dt\\
        & =R\int_{-R}^{R}\frac{1}{\sqrt{R^{2}-t^{2}}}dt
        \end{align*}

        Fazendo a mudança de variável $t=R\sin\theta$, com  $-\pi/2\leq\theta\leq\pi/2$, temos que:
        \begin{align*}
        \sqrt{R^{2}-t^{2}}  & =\sqrt{R^{2}-R^{2}\sin^{2}\theta}\\
        & =R\sqrt{1-\sin^{2}\theta}\\
        & =R\cos\theta,\\
        dt  & =R\cos\theta d\theta
        \end{align*}

        Obtemos assim que:
        \begin{align*}
        L  & =R\int_{-R}^{R}\frac{1}{\sqrt{R^{2}-t^{2}}}dt\\
        & =R\int_{-\pi/2}^{\pi/2}\frac{R\cos\theta}{R\cos\theta}d\theta\\
        & =R\int_{-\pi/2}^{\pi/2}d\theta\\
        & \left.  R\theta\right\vert _{-\pi/2}^{\pi/2}\\
        & =\pi R
        \end{align*}
        e concluimos que:
        \[
        p=2L=2\pi R
        \]


      125   

      Dê um exemplo de uma função $f(x)$ para a qual $\ \lim\limits_{x\to 0} f(x)$ não exista.


      $f(x)=1, x \neq 0$; $f(0)=2$.


      1313   

      Um fazendeiro planeja cercar um pasto retangular vizinho a um rio. O pasto deve conter $180000$ metros quadrados para fornecer grama suficiente para o rebanho. Quais as dimensões do pasto para gastar a quantidade mínima de cerca se não há necessidade de cerca ao longo do rio?


      1384   

      Um invertimento de \$500,00 da juro de 7% ao ano, capitalizado continuamente, e apót $t$ anos o investimento valerá $500e^{0,07t}$.

      1. Aproximadamente, quando o investimento valerá \$1000,00?
      2. Quando o valor do investimento estará crescendo à razão de \$50,00 por ano?


      1167   

      Encontre os intervalos abertos nos quais $f(x)=x^3-\frac{3}{2}x^2$ é crescente e os intervalos abertos nos quais é decrescente.


      795   

      Determine a equação da reta tangente em $\left( p,f\left(p\right) \right)$:

      $f\left( x\right) =\sqrt[3]{x},\;p=1$.


      $y=\dfrac{x+2}{3}$.


      700   

      Calcule a integral imprópria $\int_0^1 \frac{1}{\sqrt{1-x^2}} dx.$


      $\pi/2$.


      1118   

      Dado que os números no gráfico representam o valor das áreas demarcadas, avalie as seguintes integrais:

      Construa o gráfico da função 1 e da função 2 usando um único plano cartesiano disponível a seguir

      1. $ \int_{-2}^{-1} f(x)\ dx$
      2. $ \int_1^2 f(x)\ dx$
      3. $ \int_{-1}^1 f(x)\ dx$
      4. $ \int_0^1 f(x)\ dx$


      1. $4$
      2. $4$
      3. $-4$
      4. $-2$


      1132   

      Classifique as afirmações a seguir como verdadeiras ou falsas:

      1.      Se $ \lim\limits_{x\to 5} f(x) = \infty$, então estamos implicitamente afirmando que o limite em questão existe.
      2.      Se $ \lim\limits_{x\to 1^-} f(x) = -\infty$, então $ \lim\limits_{x\to 1^+} f(x) = \infty$.
      3.      Se $ \lim\limits_{x\to 5} f(x) = \infty$, então $f$ tem uma assíntota vertical em $x=5$.
      4.      $\infty/0$ não é uma forma indeterminada.


      1.  Falsa.
      2.  Falsa
      3.  Verdadeira
      4.  Verdadeira


      1414   

      Um toro em forma de um cilindro circular reto de raio $a$ está apoiado sobre um lado. Remove-se do toro uma cunha fazendo-se um corte vertical e outro corte a um ângulo de 45°; ambos os cortes se interceptam no centro do toro (veja a figura). Ache o volume da cunha.

      Construa o gráfico da função 1 e da função 2 usando um único plano cartesiano disponível a seguir


      327   

      Uma pequena indústria vende normalmente, a cada semana, $60$ caixas de certo produto, por $30$ reais a caixa. Foi feita uma experiência e observou-se que cada real de desconto nesse preço fez as vendas aumentarem em $5$ caixas. Assim, a experiência mostrou que, dentro de certos limites, a quantidade $C$ de caixas vendidas é uma função do desconto $x$, em reais. Determine uma expressão para essa função.


      1503   

      Utilizando as leis de exponenciação, simplifique a expressão a seguir:
      $16^2\cdot16^{1,75}$


      661   

      Use o teorema fundamental do cálculo e a regra da cadeia para calcular a derivada da função $f(x)=\int_1^{\sin x}e^{t^2} dt$. Indique claramente a justificativa de cada passagem e, em seguida, calcule $f'(\pi)$.


      1781   

      Verifique que $\displaystyle \int \text{cosec}^n (x) \, dx = -\dfrac{\text{cosec}^{n-2} (x) \text{cotg} (x) }{n-1} + \dfrac{n-2}{n-1} \int \text{cosec}^{n-2} (x) \, dx, \, n \geq 2$.


      1307   

      O limite $\lim_{x\rightarrow +\infty} x^3(1+\sin x)$ existe? Explique.


      1875   

      Prove que $\displaystyle\int x^me^xdx=x^me^x-m \displaystyle\int x^{m-1}e^xdx$.


      694   

      Calcule a integral $\int_{0}^{r}\sqrt{r^{2}-x^{2}}dx$.


      $\frac{1}{4}\pi r^4$


      547   

      Esboce o gráfico de $f(x)=x^3-x^2+1$, indicando campo de definição, intervalos de crescimento e de decrescimento, assíntotas horizontais, verticiais e inclinadas (se houver), limites no infinito, extremos relativos, estudo da concavidade, pontos de inflexão e reta tangente à curva nos pontos de inflexão.


      207   

      Utilizando o gráfico a seguir, avalie os seguintes limites

      Construa o gráfico da função 1 e da função 2 usando um único plano cartesiano disponível a seguir

      1. $ \lim\limits_{x\to 1^-} f(x)$
      2. $ \lim\limits_{x\to 1^+} f(x)$
      3. $ \lim\limits_{x\to 1} f(x)$
      4. $f(1)$
      5. $ \lim\limits_{x\to 2^-} f(x)$
      6. $ \lim\limits_{x\to 0^+} f(x)$


      1. Não existe.
      2. Não existe.
      3. Não existe.
      4. Indefinido.
      5. $0$
      6. $0$


      1697   

      Determine a área da superfície gerada pela rotação da curva a seguir em torno do eixo indicado.

      $y=\sqrt{2}$,  $3/4\leq x \leq 15/4$, eixo $x$


      1784   

      Prove que a conclusão do Teorema do Valor Médio de Cauchy pode ser escrita da seguinte forma $$ \dfrac{f(b)-f(a)}{g(b)-g(a)} = \dfrac{f'(x)}{g'(x)}, $$ sob as hipóteses adicionais de que $g(b)\neq g(a)$ e que $f'(x)$ e $g'(x)$ nunca são simultaneamente nulas sobre $(a,b)$.


      1859   

      Calcule $\sqrt{5}$ com duas casas decimais de precisão, resolvendo a equação $x^2-5=0$ e use esse resultado na fórmula quadrática para obter as raízes de $x^2+x-1=0$.


      852   

      Determine as derivadas das seguintes funções:

      1. $f\left( x\right) =e^{\tan \left( x^{3}\right) }$.

      2. $f\left( x\right) =\left( a\sin x+\cos bx\right)^{3};$

      3. $f\left( x\right) =\dfrac{xe^{-3x}}{1+\cos x}.$


      647   

      Verifique se as funções abaixo são pares, ímpares ou nenhuma das duas coisas.

      1. $f(x)=\sin x$

      2. $f(x)=\cos x$



      1. A função $\sin x$ é ímpar pois $f(-x) = \sin (-x) = -\sin(x) = -f(x)$.
      1. A função $\cos x$ é par pois $f(-x) = \cos (-x) = \cos(x) = f(x)$.


      1136   

      Suponha que $x(t)=e^{0,05t}$ e que $z(t)=e^{0,01t}$. Calcule a taxa de crescimento de $y(t)$, sabendo que $y=x^{\beta }z^{1-\beta }$, com $\beta =1/2$.


      1096   

      Avalie a seguinte integral indefinida:
        $\int x^8\ dx$


      $1/9x^9+C$


      132   

       Para a função a seguir, responda se a mesma é contínua nos pontos abaixo (e, caso não o seja, justifique)

        $ f(x) = \left\{\begin{array}{ccc}  1,  & & \text{se } x=0\\  \frac{\sin x}{x}, & &\text{se } x>0  \end{array}\right.$

      1.  $x=0$
      2.  $x=\pi$


      1. Sim.
      2. Sim.


      1412   

      A velocidade (no instante $t$) de um ponto em movimento sobre uma reta coordenada é $\cos^2 (\pi t)$ $m/s$. Qual a distância percorrida pelo ponto em 5 segundos?


      542   

      Mostre que um polinômio de terceiro grau $p\left( x\right)=ax^{3}+bx^{2}+cx+d$ ($a\neq 0$) sempre possui uma raiz real. Ilustre através de contra-exemplo que isto não é válido para polinômios de grau par, ou seja, para todo $n=2k$ par, existem polinômios de grau $n$ que não possuem raiz real.


      1730   

      Suco de maracujá (um bom calmante natural) é derramado a uma taxa uniforme de $20$cm$^3/$s em um copo de vidro em forma de um cone truncado (veja a figura abaixo). Se os raios superior e inferior do copo forem de $4$ e $3$cm e a altura $12$cm, com que rapidez estará subindo o nível de suco quando ele estiver na metade do copo? (Sugestão: estenda o copo para baixo para formar um cone.)

      Construa o gráfico da função 1 e da função 2 usando um único plano cartesiano disponível a seguir


      904   

      Resolva a equação $|x + 1| = 3$.



      Temos dois casos: $x+1=3$ ou $x+1=-3$. Resolvendo cada uma dessas equações de primeiro grau obtemos $x=2$ e $x=-4$.


      1768   

      Escreva $a^x$ em função de $e^x$. Use esse resultado para escrever $\log_a(x)$ em função de $\ln(x)$.


      135   

      Para a função a seguir, responda se a mesma é contínua nos pontos abaixo (e, caso não o seja, justifique)

        $ f(x) = \left\{\begin{array}{ccc}
       \frac{x^2-64}{x^2-11 x+24}, & &  \text{se } x\neq 8\\
      5, & & \text{se } x=8
      \end{array}\right.$

      1. $x=0$
      2. $x=8$


      1. Sim.
      2. Não. $\lim_{x\to 8} f(x) = 16/5 \neq f(8) = 5$.


      1878   

      Prove que $\displaystyle\int (sec(x))^m dx=x \dfrac{(sec(x))^{m-2}tg(x)}{m-1}+\dfrac{m-2}{m-1}\displaystyle\int (sec(x))^{m-2} dx$.


      635   

      Se $f(x+1)=\frac{x-1}{\pi -x},$ ache $f\left( x\right) $ e encontre o domínio de $f$.



      Calculando $f((x-1)+1)$:
      $f((x-1)+1)=\dfrac{(x-1)-1}{\pi-(x-1)}$
      $f(x) = \dfrac{x-2}{\pi+1-x}$.
      O domínio de $f$ é o conjunto de números reais menos os pontos em que o denominador é zero. Calculando esses valores:
      $\pi + 1 - x  = 0 \Rightarrow x = \pi + 1$.
      Portanto o domínio de $f$ é: $\{x \in \mathbb{R}; x \neq \pi + 1\}$.


      34   

      Calcule o limite $\lim\limits_{x\rightarrow \infty }\dfrac{5x^{3}-6x-3}{6x^{2}+28x+2}$.


        $\infty$


      1324   

      Ache os extremos da função $f(x)=x+3x^{3/2}$.


      1558   

      A taxa de crescimento $R$ de certo tipo de tumor pode ser relacionada com seu tamanho $x$, de modo aproximado, pela equação $R=r\cdot x\cdot ln(K/x)$, em que $r$ e $K$ são constantes positivas. Mostre que o tumor cresce mais rapidamente quando $x=e^{-1}K$.


      1328   

      Sejam $x_0,c\in\mathbb R$ e considere a função $f(x)=e^{cx}$. Encontre $f'(x_0)$ usando a definição de derivada.


      $ce^{cx_0}$.


      6   

      Calcule o limite $\lim\limits_{x\rightarrow \infty }\left( 5+\dfrac{1}{x}+\dfrac{4}{x^{2}}\right)$.


      $5$


      1316   

      Uma área retangular em uma fazenda será cercada por um rio e, nos outros três lados, por uma cerca elétrica feita de um fio. Com $800 m$ de fio à disposição, qual é a maior área que você pode cercar e quais são suas dimensões?


      96   

      Calcule os limites:

      1. $\lim\limits_{x\rightarrow 0^{+}}\log _{\frac{1}{3}}x$
      2. $\lim\limits_{x\rightarrow 1}\ln \dfrac{x^{2}-1}{x-1}$
      3. $\lim\limits_{x\rightarrow p}\dfrac{x^{n}-p^{n}}{x-p}$


      522   

      Uma viatura de polícia, vindo do norte e se aproximando de um cruzamento em ângulo reto, está perseguindo um carro em alta velocidade, que, no cruzamento, toma a direção leste. Quando a viatura está a $0,6 km$ ao norte do cruzamento e o carro fugitivo a $0,8 km$ a leste, o radar da polícia detecta que a distância entre a viatura e o fugitivo está aumentando a $20 km/h$. Se a viatura está se deslocando a 60 km/h no instante dessa medida, qual é a velocidade do fugitivo?


      1550   

      Se um raio de luz de intensidade $k$ é projetado verticalmente para baixo na água, então a sua intensidade $I(x)$ à profundidade de $x$ metros é $I(x)=ke^{-1,4x}$.

      1. A que taxa de intensidade o raio de luz está variando em relação à profundidade a $1$ metro?
      2. A que profundidade a intensidade é a metade de seu valor na superfície?


      740   

      Encontre o volume do sólido obtido pela rotação da região limitada pela curva dada em torno do eixo especificado. Esboce a região e o sólido.
        $y=e^{-x^{2}},y=0,x=0,x=1$, ao redor do eixo $y.$


      533   

      Verifique que, para todo $x>0$, verificam-se as desigualdades:

      1. $e^{x}>x+1;$
      2. $\cos x>1-\dfrac{x^{2}}{2};$
      3. $\sin x<x-\dfrac{x^{3}}{3!}+\dfrac{x^{5}}{5!}.$


      564   

      Estude a função $f\left( x\right) =\dfrac{x^{2}}{x+1}$ com relação à concavidade, pontos de inflexão, máximos e mínimos, e esboce o seu gráfico.


      130   

      Mostre que  função $f\left( x\right) =\dfrac{1}{x^2}$ é contínua em seu domínio.


      600   

      Para cada uma das afirmações abaixo, demonstre-a, se verdadeira, ou dê um contra-exemplo, se for falsa.

      1. $x<y\Longleftrightarrow 1/y<1/x$.

      2. $\sqrt{x^{2}}=x,\forall x\in \mathbb{R}$.


      1704   

      1. Use um recurso gráfico computacional para gerar os gráficos da função $f(x)=\dfrac{x-\sin x}{x^3}$, vide exercício ID 1703, e veja o que acontece.

      2. Você esperaria que um problema similar ocorresse nos arredores de $x=0$ para a função $f(x)=\dfrac{1-\cos x}{x}$? Verifique se tal ocorre. Vide questão ID 958.


      1746   

      1. Mostre que o polinômio de Taylor de $f(x)=\sin(x^2)$ de grau $4n+2$ em $0$ é:$$x^2-\dfrac{x^6}{3!}+\dfrac{x^{10}}{5!}-\ldots+(-1)^n\dfrac{x^{4n+2}}{(2n+1)!}.$$ Dica: se $p$ é o polinômio de Taylor de grau $2n+1$ para $\sin$ em $0$, então $\sin x=P(x) + R(x)$, onde $\displaystyle \lim_{x \to 0} \dfrac{R(x)}{x^{2n+1}}=0$. O que isto implica em $\displaystyle \lim_{x \to 0} \dfrac{R(x^2)}{x^{4n+2}}$?

      2. Calcule $f^{(k)}(0)$ para todo $k$.

      3. Em geral, se $f(x)=g(x^m)$, calcule $f^{(k)}(0)$ em termos das derivadas de $g$ em $0$.


      1915   

      Use camadas cilíndricas para encontrar o volume do sólido resultante quando se faz girar a área entre as curvas $y=\dfrac{1}{x^2+1}$, $x=0$, $x=1$ e $y=0$ em torno do eixo $y$.


      665   

      Derive a função $p\left( x\right)  = \int_{2x^{-5}}^{\sin 3x}\left( v^{3}-2\right) \cos vdv$.


      1332   

      Determine uma primitiva para cada uma das funções:

      1. $f(x)=cosx$

      2. $f(x)=tgx$


      1243   

      Calcule a derivada da seguinte função:
         $f\left(  x\right)  =\tan\left(  x\right)  \arcsin\left(  x^{2}\right).$


      1202   

      Demonstre que a derivada da função seno é a função cosseno.


      1594   

      A função $f(x)=|x|$ tem valor mínimo absoluto quando $x=0$, mesmo que $f$ não seja derivável em $x=0$. Isto é consistente com o Teorema de Fermat sobre máximos e mínimos locais?


      727   

      Calcule o seguinte limite:

      $\lim\limits_{x\rightarrow \infty }\left( 0,27\right) ^{x}$.


      $0$.


      1723   

      Uma escada de $4$m está apoiada em uma parede fazendo um ângulo $\theta$ com o chão. Considerando $h$ como a altura do chão até o ponto em que a escada encosta na parede, expresse $h$ em função de $\theta$ e, então, use $dh$ para estimar a variação em $h$ se $\theta$ varia de $60^\circ$ a $59^\circ$, de $60^\circ$ a $58^\circ$, e de $60^\circ$ a $55^\circ$. Interprete estes resultados.


      1601   

      Um atleta percorreu as $26,2$ milhas da maratona de Nova York em $2,2$ horas. Demonstre que em pelo menos duas ocasiões o maratonista correu a exatas $11mi/h$, supondo que as velocidades inicial e final tenham sido zero.


      919   

      Esboce o gráfico de $f(x) = |x-1|+3.$


      136   

      Para a função a seguir, dê os intervalos nos quais ela é contínua:

       $f(x) = x^2-3x+9$.


      $(-\infty,\infty)$


      166   

      Uma importante aplicação do Teorema do Valor Intermediário é o Método da Bissecção.

      Suponha que estamos interessados em encontrar as raízes de uma função contínua $f(x)$. O Método da Bissecção é uma alternativa que pode resultar em boas aproximações para as raízes, após sucessivas aplicações do método.

      Para iniciar o método, precisamos encontrar dois valores $a$ e $b$ tais que $f(a) \cdot f(b) < 0$.

      Sem perda de generalidade, vamos assumir $f(a) < 0$, $f(b) > 0$ e $a<b$. O Teorema do Valor Intermediário afirma que existe um valor $c$ no intervalo $[a,b]$ tal que $f(c) = 0$. O teorema não afirma nada a respeito da localização de $c$ dentro do intervalo, apenas que ele existe.

      O Método da Bissecção é, portanto, uma maneira sistemática de obter este valor $c$. Seja $d=\frac{a+b}{2}$ o meio do intervalo. Existem três possibilidades:

      1. $f(d) = 0 $ - Por sorte, encontramos a raiz e não é necessário prosseguir com o método.
      2. $f(d) < 0$ - Como $f(b)>0$, sabemos que há uma raiz no intervalo $[d,b]$. Este intervalo tem metade do tamanho do intervalo original, então estamos mais próximos de obter uma boa aproximação para a raiz.
      3. $f(d) > 0$ - Como $f(a)<0$, sabemos que há uma raiz no intervalo $[a,d]$. Novamente, este intervalo tem metade do tamanho do intervalo original, então estamos mais próximos de obter uma boa aproximação para a raiz.

      O Método da Bissecção é a aplicação sucessiva dos passos descritos até que se esteja próximo o suficiente da raiz de $f(x)$ para a aplicação desejada. Nota-se que para o caso em que $f(a)>0$ e $f(b)<0$ o método ainda funciona, mas no caso 2 o intervalo escolhido seria $[a,d]$ e no caso e $[d,b]$ (por quê?).

       Utilize o Método da Bissecção para encontrar as raízes de $f(x) = e^x - 2$ no intervalo $[0.65,0.7]$.


      A raiz aproximada é $x=0.69$.

        Os intervalos utilizados são:

        $[0.65,0.7] \quad [0.675,0.7] \quad [0.6875,0.7]$

        $[0.6875,0.69375]\quad [0.690625,0.69375]$


      1195   

      Calcule a derivada da seguinte função:
       $f\left(  x\right)  =\tan\left(  x\right)  \cos^{2}\left(  x\right)  .$


      1527   

      Uma das propriedades da potenciação é que $a^0=1$, $\forall a \neq 0$. Além disso, também sabe-se que $0^n=0,\quad \forall n>0$. A extensão destas regras para incluir, respectivamente, $a=0$ e $n=0$ levam a resultados conflitantes quanto ao valor de $0^0$(O que não implica em contradição, dado que as propriedades não foram estabelecidas para $a=0$ e $n=0$).

      Sendo assim, avalie $x^x$ para $x=0,1;0,01;0,001;\ldots$. Qual o padrão observado? Com o auxílio de recursos computacionais, observe o gráfico de $y=x^x$ para valores positivos de $x$, se aproximando da origem. Para qual valor a função parece convergir para $x=0$?

      Sugestão: Procure, no site, o exercício 1528. Compare os resultados obtidos.


      196   

      Prove que todo conjunto não-vazio de inteiros limitado superiormente contém um elemento máximo.

      A teoria necessária para resolver esta questão pode não ser abordada em alguns cursos de Cálculo 1. Sendo, também pertinente, às disciplinas Teoria dos Números e Análise Real I.

      Para aprofundar seus conhecimentos, dentro do escopo de Cálculo 1, recomendamos a leitura do Cap. 1 de Guidorizzi, vol. 1 e /ou o Prólogo de Spivak (vide Bibliografia de Cálculo 1).


      1765   

      Prove que $1+x+\dfrac{x^2}{2!}+\dfrac{x^3}{3!}+\ldots+\dfrac{x^n}{n!} \leq e^x$. Conclua que $\lim\limits_{x \to \infty} e^x/x^n=\infty$.


      1632   

      Discuta a seguinte "demonstração'':

      Dada a integral $\displaystyle\int (1/x)dx$, seja $dv=dx$ e $u=1/x$, de modo que $v=x$ e $du=(-1/x^2)dx$.
      Então $\displaystyle\int (1/x)dx=(1/x)x-\displaystyle\int x (-1/x^2) dx \Rightarrow \displaystyle\int (1/x)dx=1+\displaystyle\int (1/x)dx \Rightarrow 0=1.$


      57   

      Calcule, se existir, o limite $\lim\limits_{x\rightarrow 0}\sqrt x$.


      $0$.


      152   

       Determine um intervalo de comprimento $\pi/2$ cuja equação $$2x^3+3x^2-\sqrt{|\cos(x)|}=0$$ admita uma solução real.


      1826   

      Uma caixa com base quadrada e sem tampa deve ser feita a partir de uma folha de metal, de forma que o seu volume seja de $500$ cm$^3$. Seja $S$ a área da superfície da caixa e $x$ o comprimento de um lado da base quadrada. Mostre que $\displaystyle S=x^2+2000/x$, para $x>0$, e esboce o gráfico de $S$ em função de $x$ para este caso.


      1545   

       Demonstre as seguintes regras de derivação:

      1. $(\sin{x})'=cos{x}$
      2. $(\cos{x})'=-\sin{x}$
      3. $(tg{x})'=sec^2{x}$


      1251   

      Calcule a derivada de ordem $1000$ da função $f(x)=e^{kx}, k \in R$.


      $f^{1000}(x)=k^{1000}e^{kx}$


      1170   

      Encontre os valores máximo e mínimo da função $f\left(x\right)  =xe^{-x}$ no intervalo $\left[  -10,10\right]$.



      $f^{\prime}\left(  x\right)  =e^{-x}-xe^{-x}=e^{-x}\left(  1-x\right)  $.

      Como $e^{-x}>0$ temos que $f\left(  x\right)  =0$ se e somente se $1-x=0$, ou seja, se $x=1$.

      Os pontos de máximo e mínimo devem ser pontos onde $f^{\prime}\left(  x\right)  =0$ ou os extremos do intervalo.

      Avaliamos:

      $f\left(  -10\right)     =-10e^{10}$

      $f\left(  1\right)    =\frac{1}{e}$

      $f\left(  10\right)     =\frac{10}{e^{10}}$

      Como

      $-10e^{10}<\frac{10}{e^{10}}<\frac{1}{e}$

      temos que o valor máximo é $f\left(  1\right)  =\frac{1}{e}$ e o valor mínimo é $f\left(  -10\right)  =-10e^{10}$.


      1620   

      Este exercício pretende se debruçar sobre os limites

      $\lim\limits_{x \rightarrow \infty} \left(1+\frac{1}{x^2}\right)^x$ e $\lim\limits_{x \rightarrow \infty} \left(1+\frac{1}{x}\right)^x = e$.

      1. Use a regra de L'Hospital para mostrar que $\lim\limits_{x \rightarrow \infty} \left(1+\frac{1}{x}\right)^x = e$.
      2. Com o auxílio de recursos computacionais, observe as curvas de $f(x)=\left(1+\frac{1}{x^2}\right)^x$ e $g(x)=\left(1+\frac{1}{x}\right)^x$ em um único gráfico, para $x \geq 0$. Como o comportamento de $f$ se relaciona com o de $g$? Estime o valor de $\lim\limits_{x \rightarrow \infty} f(x)$.
      3. Confirme sua estimativa de $\lim\limits_{x \rightarrow \infty} f(x)$ através da regra de L'Hôspital.


      744   

      Encontre o volume de uma pirâmide de base quadrada com lado $L$ e altura $h$.


      922   

      Partindo do gráfico de $h(x)=x^2$, esboce os gráficos de $f(x) =(x-1)^2$ e $ g(x) = (x +1)^2.$


      1520   

      Sabe-se que $f$ é contínua em $1$ e que $f(1)=2$. Mostre que existe $\delta>0$ tal que para todo $x \in D_f$  vale $1-\delta<x<1+\delta \rightarrow \dfrac{3}{2}<f(x)<\dfrac{5}{2}$.


      215   

      Avalie os seguintes limites para a função definida por partes

      $ f(x) = \left\{\begin{array}{ccc}
      1-\cos^2 x, & & \text{ se }  x<a \\
      \sin^2 x, & & \text{ se }  x\geq a
      \end{array},
      \right.$
      sendo que $a$ é um número real.

      1. $ \lim\limits_{x\to a^-} f(x)$

      2. $ \lim\limits_{x\to a^+} f(x)$

      3. $ \lim\limits_{x\to a} f(x)$

      4. $f(a)$


      1. $1-\cos^2 a = \sin^2 a$
      2. $\sin^2 a$
      3. $\sin^2 a$
      4. $\sin ^2 a$


      1565   

      O número relativo de moléculas de gás em um recipiente que se movem a uma velocidade de $v$ $cm/s$ pode ser calculado por meio da distribuição de velocidade de Maxwell-Boltzmann, $F(v)=cv^2e^{-mv^2/(2kT)}$, sendo que $T$ é a temperatura em Kelvins, $m$ é a molécula e  e $c$ e $k$ são constantes positivas. Mostre que o valor máximo de $F$ ocorre quando $v=\sqrt{2kT/m}$.


      1323   

      Um fabricante de óleo deseja confeccionar latas cilindricas de volume igual a $1,5$ litro. Quais são as dimensões da lata para que o consumo de material seja o mínimo possível?


      62   

      Calcule, quando existirem, os seguintes limites (caso um limite tenda a $\pm \infty $ justifique a resposta):

      1.   $\lim\limits_{x\rightarrow 2}\dfrac{x^{2}+x-6}{\left( x-2\right) ^{3}}$

      2.    $\lim\limits_{x\rightarrow - \infty }\dfrac{5x^{5}+7x^{2}+3x+\pi }{\sqrt{7}x^{5}+4x+2}$

      3.    $\lim\limits_{x\rightarrow 0}x^{3}\cos \left( \frac{1}{x}\right)e^{x^{2}+1}$


      1800   

      Foi pedido a um torneiro mecânico que fabricasse um disco de metal circular com área de $1000cm^2$.

      1. Qual o raio do disco produzido?
      2. Se for permitido ao torneiro uma tolerância do erro de $\pm 5 cm^2$ na área do disco, quão próximo do raio ideal da parte (a) o torneiro precisa controlar o raio?
      3. Em termos da definição $\epsilon, \delta$ de $\lim\limits_{x \to a} f(x)=L$, o que é $x$? O que é $f(x)$? O que é $a$? O que é $L$? Qual valor de $\epsilon$ é dado? Qual o valor correspondente de $\delta$?


      350   

      Enuncie o Teorema Fundamental da Álgebra (de Gauss).



      "Qualquer polinômio $p(z)$ com coeficientes complexos de uma variável e de grau $n \geq 1$ tem alguma raiz complexa."


      117   

      Prove que se $f$ e $g$ são ambas funções contínuas, então $f+g$ é contínua.


      1734   

      A trajetória de uma mosca é descrita pelas seguintes equações de movimento $$x=\dfrac{\cos t}{2+\sin t}, \quad y=3+\sin(2t)-2\sin^2t\quad (0\leq t\leq 2\pi).$$

      1.  Quais são os pontos mais alto e mais baixo do vôo?

      2.  A que distância à esquerda e à direita da origem ela voa?


      845   

      Calcule a derivada da função:

      $y=\left( 2+\sin x\right) ^{x}$.


      $y' = (\sin x + 2)^x (\log(\sin x + 2) + (x \cos x )/(\sin x + 2))$.


      939   

      Calcule, pela definição, o limite $ \lim_{x\to 4} x^2+x-5 = 15$



      Considere $\epsilon >0$ arbitrário. Queremos encontrar $\delta >0$ tal que quando $|x-4|<\delta$, $|f(x)-15|<\epsilon$.
      Considere $|f(x)-15|<\epsilon$, lembrando que o objetivo é afirmar algo sobre $|x-4|$:
      \begin{gather*}
      |f(x) -15 | < \epsilon \\
      |x^2+x-5 -15 |<\epsilon \\
      | x^2+x-20 | < \epsilon \\
      | x-4 |\cdot|x+5| < \epsilon \\
      | x-4 | < \epsilon/|x+5| \\
      \end{gather*}
      Assumindo $x$ próximo de $4$, podemos assumir, por exemplo, que, $3<x<5$. Portanto
      \begin{gather*}
      3+5<x+5<5+5 \\
      8 < x+5 < 10 \\
      \frac{1}{10} < \frac{1}{x+5} < \frac{1}{8} \\
      \frac{\epsilon}{10} < \frac{\epsilon}{x+5} < \frac{\epsilon}{8} \\
      \end{gather*}
      Seja $\delta =\frac{\epsilon}{10}$. Então:
      \begin{gather*}
      |x-4|<\delta \\
      |x-4| < \frac{\epsilon}{10}\\
      |x-4| < \frac{\epsilon}{x+5}\\
      |x-4|\cdot|x+5| < \frac{\epsilon}{x+5}\cdot|x+5|\\
      \end{gather*}
      Assumindo $x$ próximo de 4, $x+5$ é positivo e podemos eliminar o módulo do lado direito da equação.
      \begin{gather*}
      |x-4|\cdot|x+5| < \frac{\epsilon}{x+5}\cdot(x+5)\\
      |x^2+x-20| < \epsilon\\
      |(x^2+x-5) -15| < \epsilon,
      \end{gather*}

      que é o que desejávamos provar.


      630   

      Determine $f\left(x\right)$ sabendo que:
      \begin{equation*} f\,^{\prime \prime }\left( x\right)  = \cos 2x+6x+4,\;f\,^{\prime }\left(0\right) =2\text{ e }f\left( 0\right) =0\text{ .}\end{equation*}



      Primeiramente, calcula-se a integral indefinida

      $f\,^\prime(x)=\int  \left(\cos 2x+6x+4\right)\,dx = 3 x^2+4 x+\frac{1}{2} \sin (2 x)+C_1$

      Pelo dado do enunciado $f\, ^\prime(0)=2$.  Avaliando a expressão acima para $x=0$, vê-se que $C_1=2$. Para obter $f(x)$, calcula-se novamente a integral indefinida:

      $f(x)=\int \left(3 x^2+4 x+\frac{1}{2} \sin (2 x)+2\right)\,dx =x^3+2 x^2+2 x-\frac{\cos ^2(x)}{2}+C_2 $

      De acordo com o enunciado, $f(0)=0$. Assim, obtém-se $C_2=\frac{1}{2}$.


      45   

      A função $f(x) = \left\{ \begin{array}{ccc} x^2-1 & & x < 3 \\x+5 & & x\geq 3 \end{array}\right.$ é contínua em todo o seu domínio? Justifique.


      Sim, é. O único ponto em que não poderia  (inicialmente) ser contínua é em $x=3$. Todavia, temos $\lim\limits_{x\to 3^-} f(x)=\lim\limits_{x\to 3^+} f(x)=f(3)=8$.


      75   

      Encontre todas as assíntotas horizontais e verticais da função $ f(x)=\frac{\sqrt{3x^2-5x+11}}{4x-7}$.


      1749   

      Seja $f:\mathbb{R} \to \mathbb{R}$ uma função derivável, tal que $f'(x)=\alpha f(x)$ para todo $x$ e sendo $\alpha$ uma constante diferente de zero. Mostre que existe uma constante $k$ tal que, para todo $x$:

      $$f(x) = k e^{\alpha x}$$


      103   

      Considere a função \begin{align*} f\left( x\right) =\left\{ \begin{array}{cc} a-x, & \text{se } x<-1 \\ x, & \text{se } -1\leq x<1 \\ \dfrac{2}{x}+b, & \text{se } 1\leq x \end{array} \right. . \end{align*}

      1. Encontre os limites laterais a direita e a esquerda de $f$ nos pontos $1$ e $-1.$
      2. Determine os valores de $a$ e $b$ que tornam $f$ contínua em toda a reta.
      3. Calcule $\lim\limits_{x\rightarrow \infty }f\left(x\right) $ e $\lim\limits_{x\rightarrow -\;\infty }f\left( x\right) $.


      178   

      Classifique as afirmações a seguir em verdadeiras ou falsas.

      1. No conjunto dos números inteiros existe um elemento que é menor do que todos os outros.

      2. O número real representado por 0,37222... é um número racional.

      3. Toda raiz de uma equação algébrica do 2º grau é um número real.

      4. O quadrado de qualquer número real é um número racional.


      1. F

      2. V

      3. F

      4. F


      1536   

      Sejam $f,g,h$ funções deriváveis. Verifique que $(fgh)'=f'gh+fg'h+fgh'$. Generalize.


      Dica: Derive a função $Fh$, onde $F=fg$. Use a regra do produto duas vezes. Para generalizar use o princípio da indução finita.


      525   

      Se o raio de um círculo cresce à taxa de $30 cm/s$, a que taxa cresce a sua área em relação ao tempo, em função do raio? Dica: Use a fórmula da área do círculo.


      1769   

      Calcule $\displaystyle \int \sin (\ln x) \, dx$ utilizando integração por partes.


      $-\dfrac{1}{2}x(cos(ln x)-sin(ln x))+C$


      528   

      Dois carros estão se encaminhando em direção a um cruzamento em ângulo reto, um seguindo a direção leste a uma velocidade de $90 km/h$ e o outro seguindo a direção sul, a $60 km/h$. Qual a taxa segundo a qual eles se aproximam um do outro no instante em que o primeiro carro está a $0,2 km$ do cruzamento e o segundo a $0,15 km$?


      1554   

       O modelo Jenss é considerado geralmente como a fórmula mais precisa para predizer a altura de uma criança em idade pré-escolar. Se $h(x)$ denota a altura (em cm) na idade $x$ (em anos) para $\frac{1}{4} \leq x \leq 6$, então $h(x)$ pode ser aproximada por $h(x)=79,041+6,39x-e^{3,261-0,993x}$.

      1. Preveja a altura e a taxa de crescimento quando uma criança atinge a idade de $1$ ano.
      2. Quando é maior e quando é menor a taxa de crescimento?


      118   

      Dê um exemplo para mostrar que o produto de uma função contínua por uma função descontínua, pode ser uma função contínua.


      615   

      Esboce o gráfico da função abaixo e resolva a inequação:

       $f\left( x\right) =\left\vert x-1\right\vert -\left\vert x+2\right\vert >x$.


      1543   

      Seja $f(x)=cossec{x}$. Calcule $f'(x)$ e $f'\left(\dfrac{\pi}{4}\right)$.



      Inicialmente, determinamos a primeira derivada da função $f$:

      $f'(x)=-cossec(x)cotg(x)$.
      Agora, substituímos $x$ por $\dfrac{\pi}{4}$ e obtemos

      $f'\left(\dfrac{\pi}{4}\right)$=-cossec\(\dfrac{\pi}{4}\)cotg\(\dfrac{\pi}{4}\)=-\dfrac{2}{\sqrt{2}}\cdot 1 = \dfrac{2}{\sqrt{2}}$.


      1623   

      Estime $\pi$ através da aplicação do Método de Newton na equação $tg(x)=0$. Qual cuidado deve ser tomado, neste caso, em relação à escolha do valor inicial?


      170   

      Seja $g$ uma função contínua em $[-3,7]$, sendo que $g(0) = 0$ e $g(2) = 25$. Existe um valor $-3<c<7$ tal que $g(c) = 15$? Por quê?



       Sim, pelo Teorema do Valor Intermediário. Na realidade, é possível ser ainda mais preciso e afirmar não só que um valor $c$ existe em $(3,7)$, mas ainda que existe um valor $x$ contido em $(0,2)$.


      1741   

      Escreva o número $\sin 1$ como uma soma (com a notação $\Sigma$), com um erro menor que $10^{-17}$.


      1173   

      Determine os pontos de máximo, de mínimo e de inflexão das seguintes funções (se existirem).

      1. $y = 8x^3 -51x^2 -90x +1$

      2. $y = -x^3 – 9x^2 + 81x - 6$


      1188   

      Calcule a derivada da seguinte função:
       $f\left(  x\right)  =\log_{2}\left(  2x\right)  \log_{3}\left(3x\right)  .$


      $f'(x) = \dfrac{2 \ln x + \ln 6}{x \ln 2 \ln 3}$.


      1335   

      Calcule o limite $\lim_{x\rightarrow 0} \dfrac{3x+\tan x}{\sin x + \tan^2 x}.$



      Temos que:
        $\dfrac{3x+\tan x}{\sin x + \tan^2 x}= \dfrac{x}{\sin x}\cdot\dfrac{3+\dfrac{\tan x}{x}}{1+ \dfrac{\sin x}{\cos^2 x}}.$

      Lembramos o limite fundamental $\lim_{x\rightarrow 0}\frac{\sin x}{x}=1$ e, além disso, observamos que
        \begin{equation*}
        \begin{split}
        &\lim_{x\rightarrow 0}\dfrac{\sin x}{\cos^2 x}=0 \\
        &\lim_{x\rightarrow 0}\dfrac{\tan x}{x}=\lim_{x\rightarrow 0}\dfrac{\sin x}{x}\cdot\dfrac{1}{\cos x}=1.
        \end{split}
        \end{equation*}
        Então:
        $\lim_{x\rightarrow 0}\dfrac{3x+\tan x}{\sin x + \tan^2 x}= \lim_{x\rightarrow 0}\dfrac{x}{\sin x}\cdot\dfrac{3+\dfrac{\tan x}{x}}{1+ \dfrac{\sin x}{\cos^2 x}} = 1\cdot\dfrac{3+1}{1+0}=4.$


      223   

      Estime numericamente os seguintes limites para a função $f(x)=\frac{x^2-11 x+30}{x^3-4 x^2-3 x+18}$:

      1. $\lim\limits_{x \to 3^-} f(x)$

      2. $\lim\limits_{x \to 3^+} f(x)$

      3. $\lim\limits_{x \to 3} f(x)$



      1. \begin{tabular}{cc}

        $x$ & $f(x)$ \\ \hline

        $2.9$ & $132.857$ \\

        $2.99$ & $12124.4$ \\

        \end{tabular}

        A tabela parece indicar que $\lim\limits_{x\to3^-}f(x) =\infty$.

      2. \begin{tabular}{cc}

        $x$ & $f(x)$ \\ \hline

        $ 3.1$ & $108.039$ \\

        $3.01$ & $11876.4$ \\

        \end{tabular}

        A tabela parece indicar que $\lim\limits_{x\to3^+}f(x) =\infty$.

      3. Ao analisar as duas tabelas, parece que $\lim\limits_{x\to3}f(x) =\infty$.


      540   

      Prove que a equação $x^{3}-3x^{2}+6=0$ admite uma única raiz real, enquanto $x^{3}+x^{2}-5x+1=0$ admite $3$ raízes.



      Para $f(x)=x^{3}-3x^{2}+6$ e $g(x)=x^{3}+x^{2}-5x+1$, queremos mostrar que $f(x)=0$ admite uma única raiz real enquanto $g(x)=0$ admite $3$ raízes.  Primeiramente, analisemos as raízes de $f(x)=0$. Temos:

      $f'(x)= 3x^2-6x=x(x-2)$

      $f''(x)=6(x-1)$

      Portanto, a segunda derivada nos diz que $f(x)$ é uma concavidade para baixo para $x<1$ e uma concavidade para cima para $x>1$, e os zeros da primeira derivada nos dizem que $f(x)$  apresenta um máximo local em $x=0$ e um mínimo local em $x=2$. Como $f(2)=2$, temos que não há raiz alguma para $x>1$, dado que a função é uma concavidade para baixo neste intervalo. A única raiz real, portanto, é algum valor $x<1$. O conhecimento do máximo local em $x=0$ nos permite inclusive dizer que é algum valor $x<0$.

      Repetiremos a análise para $g(x). Temos, portanto:

      $g'(x)= 2x^2+2x-5=2(x+5/3)(x-1)$

      $g''(x)=6(x+1/3)$

      Portanto, a segunda derivada nos diz que $g(x)$ é uma concavidade para baixo para $x<-1/3$ e uma concavidade para cima para $x>-1/3$, e os zeros da primeira derivada nos dizem que $g(x)$ apresenta um máximo local em $x=-5/3$ e um mínimo local em $x=1$. Como $g(0)=1$ e $g(1)=-2$, sabemos que há duas raízes reais  de $g(x)$ no intervalo aberto $x>0$. Como $g(x)$ claramente tende para $-\infty$ para $x\rightarrow \infty$, o valor de $g(0)$ nos diz que a terceira raiz real está localizada no intervalo $x<0$. O conhecimento do máximo local em $x=-5/3$ nos permite inclusive dizer que é algum valor no intervalo aberto $x<-5/3$


      801   

      Ache uma fórmula para a soma $1+2x+3x^2 +\cdots +nx^{n-1}$.


      $\dfrac{nx^{n+1}-(n+1)x^n+1}{(x-1)^2}$, $x \neq 1$. Se $x=1$, a soma dá $\dfrac{n(n+1)}{2}$.


      575   

      Estude a função $f\left( x\right) =\dfrac{x}{1+\tan x},x\in \lbrack 0,\dfrac{\pi }{2})$ com relação à concavidade, pontos de inflexão, máximos e mínimos, e esboce o seu gráfico.


      1772   

      Considere uma força $f(x)$ que atua sobre um corpo no ponto $x$. A força varia em função do ponto $x$, segundo a função $f(x)=x^5 \sqrt{x^3+1}$. Determine o trabalho realizado se o corpo se move do ponto $x=0$ ao ponto $x=1$.


      1818   

      Sejam $f,g,h$ funções deriváveis. Verifique que $(fgh)'=f'gh+fg'h+fgh'$. Generalize.


      1582   

      Dados $f(x) = 1+x$ e $x_0 = 8,1$, escolha um valor inteiro próximo a $x_0$ tal que $f(x_0)$ e $f'(x_0)$ sejam fáceis de calcular, e calcule uma linearização da função neste ponto.


      111   

      Mostre, usando a definição, que a função dada por $f(x) = 3x$ é contínua para todo $x$ real.


      1674   

      Calcule a integral a seguir utilizando decomposição de quocientes em frações parciais:

      $\int_{0}^{1}{\frac{x^3dx}{x^2 + 2x + 1}}$


      1552   

      Uma substância radioativa decai de acordo com a fórmula $q(t)=q_0e^{-ct}$, onde $q_0$ é a quantidade inicial da substância, $c$ é uma constante positiva, e $q(t)$ é a quantidade remanescente após o tempo $t$. Mostre que a taxa na qual a substância decai é proporcional a $q(t)$.


      1820   

      Sejam $f_1,f_2,\ldots,f_n$, $n \geq 2$, funções deriváveis em $p$. Prove, por indução finita, que $f_1+f_2+\ldots+f_n$ é derivável em $p$. Veja Guidorizzi, volume $1$, página $158$.


      514   

      Determine o domínio de definição das funções trigonométricas inversas a seguir e expresse suas derivadas em termos de funções polinomiais:

      1. $g\left( x\right) =\mathrm{\arccos }\left( x\right) $;
      2. $g\left( x\right) =\mathrm{arcsec}\left( x\right) $;
      3. $g\left( x\right) =\mathrm{arccot}\left( x\right) $.


      588   

      Se $a$ é racional e $b$ é irracional então podemos afirmar alguma coisa sobre $a+b$ em termos de racionalidade ou irracionalidade?


      Podemos afirmar que $a+b$ sempre será irracional.


      1649   

      Um foguete decola da superfície terrestre com uma aceleração constante de $20m/s^2$. Qual será sua velocidade 1 minuto depois?


      1603   

      O gráfico a seguir mostra a receita mensal da empresa Fidelis Ltda. nos últimos 12 anos. Durante aproximadamente quais intervalos de tempo a receita marginal foi crescente? E decrescente?

      Construa o gráfico da função 1 e da função 2 usando um único plano cartesiano disponível a seguir


      1577   

      Determine $f'$, $f''$ e $f'''$ sendo $f(x)=\left\{\begin{array}{ll}
      x^2+3x, & \text{se} x \leq 1 \\
      5x-1, & \text{se} x>1  
      \end{array}\right.$.


      153   

      Mostre que toda equação polinomial de grau ímpar, tem pelo menos uma raiz real.


      197   

      Sejam $a, b$ racionais positivos. Prove que $\sqrt{a}+\sqrt{b}$ é racional se, e somente se, $\sqrt{a}$ e $\sqrt{b}$ forem ambos racionais. (Sugestão: multiplique por $\sqrt{a}-\sqrt{b}$).

      A teoria necessária para resolver esta questão pode não ser abordada em alguns cursos de Cálculo 1. Sendo, também pertinente, às disciplinas Teoria dos Números e Análise Real I.

      Para aprofundar seus conhecimentos, dentro do escopo de Cálculo 1, recomendamos a leitura do Cap. 1 de Guidorizzi, vol. 1 e /ou o Prólogo de Spivak (vide Bibliografia de Cálculo 1).


      1256   

      O fluxo de um campo magnético através de uma bobina, em função do tempo, é dado por  $F=B \cdot l^2 \sin(\omega t)$ , onde $B$ é a intensidade do campo, $l$ o comprimento da espira e $\omega$ a velocidade angular da bobina. Pela "Lei de Faraday'', temos que a tensão $v$ do circuito associado a esse campo é dada por $v=-\frac{dF}{dt}$.

      1. Escreva a equação do fluxo para $B = 20$, $l = 2$ e  $\omega= 4$.
      2. Para a equação obtida no item anterior, determine a expressão de v em função de t.


      1677   

      Sociólogos utilizam a expressão "difusão social" para descrever o modo como a informação se espalha por uma população. A informação pode ser um boato, uma novidade cultural, ou notícias sobre uma inovação técnica Em uma população suficientemente grande, o número de pessoas $x$ que tem a informação é tratado como uma função derivável do tempo $t$, e a taxa de difusão é supostamente proporcional ao número de pessoas que têm a informação multiplicado pelo número de pessoas que não a tem, isto é,

      $\frac{dx}{dt} = kx\left(N-x\right)$, sendo que $N$ é o número total de pessoas da população.

      Suponha então que $t$ seja medido em dias, $k=1/250$ e que duas pessoas deram início a um boato no momento $t=0$ em uma população tal que $N=1000$.

      1. Determine $x(t)$.

      2. Quando metade da população terá ouvido o boato?


      1755   

      Avalie a integral $\displaystyle \int_{-1}^1 x^3 \sqrt{1-x^2} \, dx$ sem fazer nenhuma conta.


      1914   

      Use camadas cilíndricas para encontrar o volume do sólido resultante quando se faz girar a área entre as curvas $y=\cos(x^2)$, $x-0$, $x=\dfrac{1}{2}\sqrt{\pi}$ e $y=0$ em torno do eixo $y$.


      101   

      Calcule e justifique os seguintes limites, quando existirem, ou justifique a inexistência:

      1. $\lim\limits_{x\rightarrow -1}\sqrt[3]{\dfrac{x^{3}+1}{x+1}}$
      2. $\lim\limits_{x\rightarrow 1}\dfrac{\sqrt{x^{2}+3}-2}{x^{2}-1}$
      3. $\lim\limits_{x\rightarrow 1}\dfrac{\sqrt[3]{3x+5}-2}{x^{2}-1}$


      1279   

      Calcule a seguinte integral:
       $ \int_2^3 \frac{1}{x^2-1}dx$.


      $\tanh ^{-1}(2)-\tanh ^{-1}(3)$


      826   

      Calcule $f^{\prime }\left( x\right)$:

        $f\left( x\right) =2^{x}$.


      $f'(x)=ln(2)2^x$.


      657   

      Seja $f(x)=\frac{1+x}{1-x}$. Mostre que $f\left(\frac{1}{1+x}\right)=\frac{2+x}{x}$, $f\left(\frac{1}{1-x}\right)=\frac{x-2}{x}$, $f(-x)=\frac{1}{f(x)}$, $f(1/x)=-f(x)$,  $f(f(x))=-1/x$.


      1541   

      Calcule $f'(x)$ sendo

      1. $f(x)=tg{x}$
      2. $f(x)=sec{x}$


      1. $f'(x)=sec^2(x)$.

      2. $f'(x)=sec(x)tg(x)$.


      1578   

      Determine a derivada de ordem $n$ de:

      1. $f(x)=e^x$
      2. $f(x)=\cos{x}$
      3. $f(x)=\sin{x}$
      4. $f(x)=\ln{x}$


      944   

      Calcule, através da definição de limite, $\displaystyle \lim_{x\to 0} e^{2x}-1 = 0$.



      Seja $\epsilon >0$ dado. Queremos $\delta >0$ tal que, quO IMECC é responsável pelos cursando $|x-0|<\delta$, $|f(x)-0|<\epsilon$.
      Considerando $|f(x)-0|<\epsilon$, lembrando que o objetivo é afirmar algo sobre $|x-0|$ (i.e., $|x|$):
      \begin{gather*}
      |f(x) -0 | < \epsilon \\
      |e^{2x}-1 |<\epsilon \\
      -\epsilon< e^{2x}-1 < \epsilon \\
      1-\epsilon< e^{2x} < 1+\epsilon \\
      \ln (1-\epsilon) < 2x < \ln (1+\epsilon) \\
      \frac{\ln (1-\epsilon)}{2} < x < \frac{\ln (1+\epsilon)}{2} \\
      \end{gather*}
      Seja $\delta = \min\left\{\left|\frac{\ln(1-\epsilon)}{2}\right|,\frac{\ln(1+\epsilon)}{2}\right\}=\frac{\ln(1+\epsilon)}{2}.$
      Portanto:
      \begin{gather*}
      |x| < \delta \\
      |x| <\frac{\ln(1+\epsilon)}{2}<\left|\frac{\ln(1-\epsilon)}{2}\right| \\
      \frac{\ln(1-\epsilon)}{2} < x < \frac{\ln(1+\epsilon)}{2}\\
      \ln(1-\epsilon)< 2x < \ln(1+\epsilon)\\
      1-\epsilon < e^{2x} < 1+\epsilon\\
      -\epsilon < e^{2x}-1 < \epsilon\\
      |e^{2x}-1-(0)| < \epsilon,
      \end{gather*}

      que é o que buscávamos provar.


      585   

      Encontre todos os números reais que satisfazem a cada uma das desigualdades abaixo. Dê o intervalo solução e ilustre a solução sobre a reta real.

      1. $|6+4x|<\left| 2-{\frac{x}{2}}\right| $

      2. $\left| {\frac{5}{3x-2}}\right| \geq \left| {\frac{2}{x-1}}\right| $


      113   

      Considere a função real de variável real definida por \begin{align*}
      f(x)=\frac{\sqrt{1-x^2}}{1-tg x} \end{align*}

      1. Determine o domínio de $f$.
      2. Estude $f$ quanto a continuidade.


      1736   

      Prove o seguinte resultado: Se $f$ tiver um mínimo absoluto em um intervalo aberto $(a,b)$, então ele precisa ocorrer em um ponto crítico de $f$.


      1686   

      Determine o comprimento da curva a seguir no intervalo especificado.

      $y=(1/3)\left(x^2+2\right)^{3/2},\quad 0 \leq x \leq 3$


      1250   

      Compute a derivada $f''(x)$ de $f(x)=\frac{x^2+1}{x}$.


      $f''(x)=\dfrac{2}{x^3}$.


      1551   

      Se $p$ denota o preço de venda de um artigo e $x$ é a procura correspondente (em número de artigos vendidos por di, então a relação entre $p$ e $x$ pode ser dada por $p(x)=p_0e^{-ax}$ para constantes positivas $p_0$ e $a$. Suponha $p(x)=300e^{-0,02x}$. Determine o preço de venda que maximize a receita diária.


      1788   

      Sejam $f$ e $g$  funções contínuas em $[a,b]$ e diferenciáveis em $(a,b)$. Prove:  se $f(a)=g(a)$ e $f(b)=g(b)$, então há um ponto $c$ em $(a,b)$ onde $f'(c)=g'(c)$.


      1750   

      Considere $y=f(x)$, para $x$ real, sendo $f$ derivável até a segunda ordem e tal que, para todo $x$, $f''(x)+f(x)=0$. Seja $g$ uma função tal que $g(x)=f'(x) \sin x - f(x) \cos x$. Mostre que $g$ é constante.


      532   

      O raio $r$ e a altura $h$ de um cilindro circular reto estão variando de modo a manter constante o volume $V$. Num determinado instante, $h=3cm$ e $r=1cm$ e, neste instante, a altura está variando a uma taxa de $0,2cm/s$. A que taxa está variando o volume neste instante?


      1556   

      Os impulsos nervosos no corpo humano caminham ao longo de fibras nervosas que consistem em um axônio, que transporta o impulso, envolvido por uma camada de mielina. A fibra nervosa é semelhante a um cabo cilíndrico isolado, para o qual a velocidade $v$ de um impulso é dada por $v=-k(r/R)^2 \ln(r/R)$, onde $r$ é o raio do cabo e $R$ é o raio de isolamento. Ache o valor de $r/R$ que maximize $v$. Na maioria das fibras nervosas, $r/R$ vale aproximadamente $0,6$.

      Construa o gráfico da função 1 e da função 2 usando um único plano cartesiano disponível a seguir


      44   

      Calcule o limite $\lim\limits_{x\to e} \ln x$, em que $e$ é o número de Euler.


      $1$.


      1280   

      Calcule a seguinte integral:
         $\int_{0}^{r}\sqrt{r^{2}-x^{2}}dx.$


      1319   

      A soma dos perímetros de um triângulo equilátero e de um quadrado é $10$. Ache as dimensões do triângulo e do quadrado que produzem a área total mínima.


      209   

      Utilizando o gráfico a seguir, avalie os seguintes limites

      Construa o gráfico da função 1 e da função 2 usando um único plano cartesiano disponível a seguir

      1. $ \lim\limits_{x\to 1^-} f(x)$
      2. $ \lim\limits_{x\to 1^+} f(x)$
      3. $ \lim\limits_{x\to 1} f(x)$
      4. $f(1)$
      5. $ \lim\limits_{x\to 2^-} f(x)$
      6. $ \lim\limits_{x\to 2^+} f(x)$


      1. $1$
      2. $2$
      3. Não existe.
      4. $2$
      5. $0$
      6. Como $f$ não é definida para $x>2$, esse limite é indefinido.


      524   

      Dois automóveis movem-se em direção a um cruzamento em ângulo reto, um dirigindo-se para o leste à razão de $72 km/h$ e o outro para o sul à razão de $54 km/h$. Com que velocidade os carros aproximam-se um do outro no instante em que o primeiro está a $400 m$ e o segundo a $300 m$ do cruzamento?


      1752   

      1. Utilizando somas superiores, mostre que a área sob o gráfico de $y=x$ no intervalo $[0,b]$ é $b^2/2$.

      2. Mostre o mesmo resultado utilizando somas inferiores.


      147   

      Use o teorema do valor intermediário para mostrar que $f(x)=4x^3-6x^2+3x-4$ possui um zero no intervalo $[1,2]$.


      Como $f(1) = -3 < 0$ e $f(2) = 10 > 0$, temos que a função $f$ muda de sinal no intervalo $[1,2]$, e portanto, pelo teorema do valor intermediário, $f$ possui um zero nesse intervalo.


      5   

      Esboce o gráfico da função $f\left(x\right) =x+\dfrac{1}{x}$.


      736   

      Considere a região no plano com limite inferior dado por  $y=1+x^2$ e limite superior $y=2$. Calcule os volumes quando rotacionamos essa região:

      1. Ao redor do eixo $x$.
      1. Ao redor do eixo $y$.


      1606   

      Um peso de massa $m$ é preso a uma bola suspensa a partir de um suporte. O peso é posto em movimento movendo-se o suporte para cima e para baixo de acordo com a fórmula $h=A \cos(\omega t)$, onde $A$ e $\omega$  são constantes positivas e $t$ é o tempo. Se as forças de atrito são desprezíveis, então o deslocamento $s$ do peso em relação à sua posição inicial no instante $t$ é dada por $s=\dfrac{A \omega^2}{\omega_0^2-\omega^2}(\cos(\omega t)-\cos(\omega_0 t))$ com $\omega_0=\sqrt{k/m}$ para uma constante $k$ e com $\omega \neq \omega_0$. Calcule $\lim\limits_{\omega \to \omega_0}s$ e mostre que as oscilações resultantes aumentam em magnitude.


      1602   

      O gráfico a seguir mostra o custo hipotético $c=f(x)$ para fabricar $x$ itens. O chamado custo marginal é a mudança no custo total advinda da produção de uma unidade a mais do produto, para um certo volume de produção. Em aproximadamente qual nível de produção o custo marginal muda de decrescente para crescente?

      Construa o gráfico da função 1 e da função 2 usando um único plano cartesiano disponível a seguir


      80   

      O que significa dizer, em termos de limites, que uma função é "bem comportada"?


      1902   

      Prove que o comprimento de um arco de ciclóide é igual a $8$ vezes o tamanho do raio do seu círculo gerador. A figura abaixo mostra dois arcos e meio de ciclóide.

      Construa o gráfico da função 1 e da função 2 usando um único plano cartesiano disponível a seguir


      535   

      Encontre $a$ e $b$ tais que a função $f(x)=x^3 +ax^2+b$ tenha um extremo relativo em $(2,4)$.


      538   

      Esboce o gráfico da função $f\left( x\right) =\frac{x^{2}+3}{x-1}$, indicando domínio de definição, limites no infinito, assíntotas verticais e inclinadas, intervalos de crescimento e decrescimento e estudo da concavidade.


      1672   

      Calcule a integral a seguir utilizando substituições trigonométricas:

      $\int{\frac{x^3 dx}{\sqrt{x^2+4}}}$


      $\dfrac{1}{3}(x^2-8)\sqrt{x^2+4}+C$.


      1329   

      Encontre os pontos sobre o gráfico de $p(x)=x^3-2x^2-8x+3$ nos quais a reta tangente é paralela à reta $y=4-9x.$


      839   

      Derive a função abaixo e avalie a derivada no ponto indicado:


      $f\left( x\right) =e^{2x^{3}}+\cos \left( \sin \left( 3x\right)\right) ;$ avaliar em $f\,^{\prime }\left( 0\right) $.


      $f'(x) = 6 e^{2 x^3} x^2 - 3 \sin(\sin(3 x)) \cos(3 x)$.

      $f'(0) = 0$.


      1616   

      Calcule o limite $\lim\limits_{x \to \infty}\dfrac{x-\cos(x)}{x}$.


      $1$.


      958   

      Calcule o limite a seguir, justificando as passagens.

      $\lim\limits_{x\rightarrow 0}\dfrac{1-\cos x}{x}$


      0



      Para todo $x\neq 0$ temos que
      \begin{equation*}
      \dfrac{1-\cos x}{x}=\dfrac{1-\cos x}{x}\dfrac{1+\cos x}{1+\cos x}=\dfrac{
      1-\cos ^{2}x}{x}\dfrac{1}{1+\cos x}\text{.}
      \end{equation*}
      Como $1-\cos ^{2}x=\sin ^{2}x$ obtemos
      \begin{eqnarray*}
      \dfrac{1-\cos x}{x} &=&\dfrac{\sin ^{2}x}{x}\dfrac{1}{1+\cos x} \\
      &=&\sin x\dfrac{\sin x}{x}\dfrac{1}{1+\cos x}.
      \end{eqnarray*}
      Mas
      \begin{eqnarray*}
      \lim\limits_{x\rightarrow 0}\sin x &=&0\;\text{(pois }\sin x\text{ é contínua)} \\
      \lim\limits_{x\rightarrow 0}\dfrac{\sin x}{x} &=&1\;\text{(limite trigonométrico fundamental)} \\
      \lim\limits_{x\rightarrow 0}\dfrac{1}{1+\cos x} &=&\dfrac{1}{2}\;\text{(}
      \cos x\text{ cont\'{i}nua e }1+\cos \left( 0\right) \neq 0\text{).}
      \end{eqnarray*}
      Logo,
      \begin{equation*}
      \lim\limits_{x\rightarrow 0}\dfrac{1-\cos x}{x}=\lim\limits_{x\rightarrow
      0}\sin x\lim\limits_{x\rightarrow 0}\dfrac{\sin x}{x}\lim\limits_{x
      \rightarrow 0}\dfrac{1}{1+\cos x}=0.
      \end{equation*}


      1773   

      Os gráficos das equações $y=e^x$, $y=0$, $x=0$ e $x=\ln 3$ formam uma região delimitada no plano. Calcule o centroide dessa região.


      127   

      A afirmação: $`` \lim\limits_{x\rightarrow p^+} f(x) = \lim\limits_{x\rightarrow p^-} f(x)\Rightarrow f \mbox{ contínua em } p. "$  é verdadeira ou falsa?  Justifique.



      É falsa. Só seria verdadeira se o valor dos limites laterais fosse igual a $f(p)$.


      925   

      A área superficial de uma caixa retangular fechada de base quadrada é igual a $20 m^2$. Determine o volume desta caixa em função do comprimento do lado de sua base.


      1510   

      Se $(ln\ x)/x = (ln\ 2)/2$, é necessário que $x=2$? Se $(ln\ x)/x=-2ln\ 2$, é necessário que $x=\frac{1}{2}$? Justifique suas respostas.


      1621   

      Demonstre que, se $h>0$, aplicando o Método de Newton para

      $f(x)=\begin{cases}
      \sqrt{x},\quad \ \ x \geq 0\\
      \sqrt{-x},\quad x <0
      \end{cases}$,
      a aproximação tende a $x_1=-h$ se $x_0=h$ e a $x_1=h$ se $x_0=-h$.
      Desenhe uma figura para mostrar o que ocorre.


      1607   

      O modelo logístico de crescimento populacional prevê o tamanho $y(t)$ de uma população no instante $t$ por meio da fórmula $y(t)=\dfrac{k}{1+ce^{-rt}}$, onde $r$ e $k$ são constantes positivas e $c=\dfrac{k-y(0)}{y(0)}$. Os ecologistas denominam $k$ a capacidade de suporte e o interpretam como o número máximo de indivíduos que o ambiente pode sustentar. Calcule $\lim\limits_{t \to \pm \infty}y(t)$ e discuta o significado gráfico desses limites.


      1692   

      Qual das integrais a seguir, se houver alguma, serve para calcular a área da região sombreada mostrada aqui? Justifique sua resposta.

      1. Construa o gráfico da função 1 e da função 2 usando um único plano cartesiano disponível a seguir
        $\int_{-1}^{1} {\left(x-\left(-x\right)\right)dx} = \int_{-1}^{1} {2x\ dx}$
      2. $\int_{-1}^{1} {\left(-x-x\right)dx} = \int_{-1}^{1} {-2x\ dx}$



      Uma análise do resultado de ambas as integrais nos mostra, de imediato, que nenhuma delas é a adequada para o cálculo da área da figura. A segunda integral nada mais é do que a primeira integral com o sinal invertido, e portanto ambas são iguais a zero.

      A questão é que no caso, se denotarmos $f_1(x)=x$ e $f_2(x)=-x$, é fácil observar que para $x>0$ $f_1(x)>f_2(x)$, e para $x<0$ $f_2(x)>f_1(x)$. Portanto, o cálculo correto da área se daria através de duas integrais, na forma 

      $A=\int_{-1}^{0} {\left(-x-x\right)dx}+\int_{0}^{1} {\left(x-\left(-x\right)\right)dx}$

      Ou ainda, fazendo uso da simetria, poderia também se fazer:

      $A=2\int_{0}^{1} {\left(x-\left(-x\right)\right)dx}=4\int_{0}^{1} {x\,dx}=2$


      632   

      Determine $f\left(x\right)$ sabendo que: \begin{equation*} f\,^{\prime \prime }\left( x\right)  = 9e^{3x}+\cos x+x^{6},\;f\,^{\prime}\left( 0\right) =1\text{ e }f\left( 0\right) =2\text{ .} \end{equation*}


      1534   

      Calcule $F'(x)$ sendo $F(x)$ igual a:

      1. $xe^x\cos{x}$
      2. $e^x \sin{x} \cos{x}$


      548   

      Esboce o gráfico de $f(x)=x^4-5x^2+4$, indicando campo de definição, intervalos de crescimento e de decrescimento, assíntotas horizontais, verticiais e inclinadas (se houver), limites no infinito, extremos relativos, estudo da concavidade, pontos de inflexão e reta tangente à curva nos pontos de inflexão.


      1618   

      Calcule o limite $\lim\limits_{x \to 0^+}\dfrac{\ln(tg{x}+\cos{x})}{\sqrt{\ln(x^2+1)}}$.


      $1$.


      1504   

      Utilizando as leis de exponenciação, simplifique a expressão a seguir:
      $9^{1/3}\cdot9^{1/6}$


      645   

      Nos exercícios abaixo determine o domínio máximo de definição de cada uma das funções dadas.

      1. $y=\sqrt{x^{2}-4x+3}$

      2. $y=\sqrt{x^{2}+3x-10}$


      1253   

      Calcule a derivada de ordem $n$ da função $f(x)=\sin{x}+\cos{x}$.


      1802   

      Sejam $f$ e $g$ funções contínuas tais que $0 \leq f(x) \leq g(x)$, para $x\geq a$. Utilizando conceitos de área, explique informalmente o porquê dos resultados abaixo serem verdadeiros.

      1. Se $\displaystyle \int_a^{+\infty} f(x) \, dx$ diverge, então $\displaystyle \int_a^{+\infty} g(x) \, dx$ diverge.

      2. Se $\displaystyle \int_a^{+\infty} g(x) \, dx$ converge, então $\displaystyle \int_a^{+\infty} f(x) \, dx$ converge e $\displaystyle \int_a^{+\infty} f(x) \, dx \leq \int_a^{+\infty} g(x) \, dx$.

      Obs: estes resultados são chamados de testes de comparação para integrais impróprias.


      1097   

       Avalie a seguinte integral indefinida:
        $\int \sin\theta\  d\theta$


      $-\cos \theta+C$


      660   

      Considere o gráfico da função $f$:


      $f\left( x\right) =\left\{\begin{array}{c}-2x-2,-4\leq x\leq -2 \\x+4,-2\leq x\leq 1 \\6-x,1\leq x\leq 4\end{array}\right.$

      Construa o gráfico da função 1 e da função 2 usando um único plano cartesiano disponível a seguir

      Esboce, a partir deste, os gráficos das seguintes funções:

      1. $y=f\left( x+4\right) $

      2. $y=f\left( x\right) +4$

      3. $y=2f\left( x\right) $

      4. $y=-\dfrac{1}{2}f\left( x\right) +3.$


      690   

      Calcule o seguinte limite


      $\lim\limits_{x\rightarrow \infty }\left( 1+\dfrac{1}{x}\right)^{x+2}$.


      1823   

      1.  Explique por que a regra de L'Hospital  não se aplica ao problema $$ \lim_{x\to 0}\dfrac{x^2\sin(1/x)}{\sin x}.  $$

      2.  Ache o limite acima.


      1785   

      Teorema de Rolle: Seja $f$ uma função diferenciável em $(a,b)$ e contínua em $[a,b]$; se $f(a)=f(b)=0$, então há pelo menos um ponto $c$ em $(a,b)$ tal que $f'(c)=0$. Verifique que as hipóteses do Teorema de Rolle estão satisfeitas em cada intervalo dado e ache todos os valores de $c$ naquele intervalo que satisfazem a conclusão do teorema.

      1.  $\displaystyle f(x)=\dfrac{x^2-1}{x-2},\quad [-1,1]$;

      2.  $\displaystyle f(x)=\dfrac{1}{x^2}-\dfrac{4}{3x}+\dfrac{1}{3},\quad [1,3].$


      60   

      Determine se as afirmações abaixo são verdadeiras ou falsas. Justifique suas respostas ou forneça um contra exemplo.

      1.    Se $\lim\limits_{x\rightarrow a}f\left( x\right) =\infty $ e $\lim\limits_{x\rightarrow a}g\left( x\right) =0$, então $  \lim\limits_{x\rightarrow a}\dfrac{f\left( x\right) }{g\left( x\right) }  =\infty $. 

      2.    Sejam $p\left( x\right) $ e $q\left( x\right) $ polinômios de grau $m$ e $n$ respectivamente. Se $\lim\limits_{x\rightarrow \infty  }\dfrac{p\left( x\right) }{q\left( x\right) }=0$, então $m\geq n$.

      3.    Se $\lim\limits_{x\rightarrow a}\left( f\left( x\right)  g\left( x\right) \right) $ existe, então $\lim\limits_{x\rightarrow   a}f\left( x\right) $ e $\lim\limits_{x\rightarrow a}g\left( x\right) $   existem e $\lim\limits_{x\rightarrow a}\left( f\left( x\right) g\left(   x\right) \right) =\left( \lim\limits_{x\rightarrow a}f\left( x\right)  \right) \left( \lim\limits_{x\rightarrow a}g\left( x\right) \right) .$

      4.   Se $f\left( x\right) $ e $g\left( x\right) $ são contínuas em $a$, então $\left( f+g\right) \left( x\right) $ também é contínua em $a$.


      129   

      Seja $f:\mathbb{R\rightarrow R}$ a função definida por \begin{equation*} f\left( x\right) =\left\{ \begin{array}{cc} x^{2}, & \text{se }x\leq 1 \\ 2x-1, & \text{se }x>1 \end{array} \right. , \end{equation*} e defina $g\left( x\right) =\lim\limits_{x \rightarrow h}\dfrac{f \left(x+h \right) -f \left( x\right) }{h}$. 
      Mostre que $g\left( x\right) $ é contínua.



      Observe que para $x<1$ temos que

      \begin{eqnarray*} g\left( x\right) &=&\lim\limits_{h\rightarrow 0}\dfrac{f\left( x+h\right) -f\left( x\right) }{h} \\ &=&\lim\limits_{h\rightarrow 0}\dfrac{\left( x+h\right) ^{2}-x^{2}}{h} \\ &=&\lim\limits_{h\rightarrow 0}\dfrac{x^{2}+2hx+h^{2}-x^{2}}{h} \\ &=&\lim\limits_{h\rightarrow 0}\dfrac{2hx+h^{2}}{h} \\ &=&\lim\limits_{h\rightarrow 0}\left( 2x+h\right) =2x. \end{eqnarray*} 

      Já para $x>1$ temos que 

      \begin{eqnarray*} g\left( x\right) &=&\lim\limits_{h\rightarrow 0}\dfrac{f\left( x+h\right) -f\left( x\right) }{h} \\ &=&\lim\limits_{h\rightarrow 0}\dfrac{\left[ 2\left( x+h\right) -1\right] - \left[ 2x-1\right] }{h} \\ &=&\lim\limits_{h\rightarrow 0}\dfrac{2h}{h}=2. \end{eqnarray*} 

      Para $x=1$ temos que 

      \begin{eqnarray*} \lim\limits_{h\rightarrow 0^{+}}\dfrac{f\left( 1+h\right) -f\left( 1\right) }{h} &=&\lim\limits_{h\rightarrow 0}\dfrac{\left[ 2\left( 1+h\right) -1 \right] -1}{h} \\ &=&\lim\limits_{h\rightarrow 0}\dfrac{2h}{h}=2 \\ \lim\limits_{h\rightarrow 0^{-}}\dfrac{f\left( 1+h\right) -f\left( 1\right) }{h} &=&\lim\limits_{h\rightarrow 0^{-}}\dfrac{\left( 1+h\right) ^{2}-1}{h} \\ &=&\lim\limits_{h\rightarrow 0^{-}}\dfrac{2h+h^{2}}{h} \\ &=&\lim\limits_{h\rightarrow 0^{-}}\left( 2+h\right) =2. \end{eqnarray*} 

       Temos então que $g$ é bem definida também no ponto $x=1$ e, de modo geral, $g$ pode ser expressa por \begin{equation*} g\left( x\right) =\left\{ \begin{array}{cc} 2x & \text{se }x\leq 1 \\ 2 & \text{se }x>1 \end{array} \right. \text{.} \end{equation*} 

      Como as funções $h\left( x\right) =2x$ e $p\left( x\right) \equiv 2$ são contínuas, temos que $g\left( x\right) $ é contínua para todo $x\neq 1$. 

      Além disto, como $\lim\limits_{x\rightarrow 1^{-}}g\left( x\right) =\lim\limits_{x\rightarrow 1}2x=2=\lim\limits_{x\rightarrow 1^{+}}2=\lim\limits_{x\rightarrow 1^{+}}g\left( x\right) $, segue que $\lim\limits_{x\rightarrow 1}g\left(x\right) =2$. Mas como $g\left( 1\right) =2$, segue que a função $ g\left( x\right) $ também é contínua no ponto $x=1$.


      1542   

      Seja $f(x)=cotg{x}$. Calcule $f'(x)$ e $f'\left(\dfrac{\pi}{4}\right)$.


      $f'(x)=-cossec^2(x)$ e $f'\left(\dfrac{\pi}{4}\right)=-2$.


      1192   

      Calcule a derivada da seguinte função:
       $f\left(  x\right)  =x^{3}\ln\left(  x^{2}\right)  .$


      928   

      Segundo dados de uma pesquisa, a população de certa região do país vem decrescendo em relação ao tempo t, contado em anos, aproximadamente, segundo a relação $P(t)=P(0) \cdot 2^{-0,25t}$. Sendo $P(0)$ uma constante que representa a população inicial dessa região e $P(t)$ a população $t$ anos após, determine quantos anos se passarão para que essa população fique reduzida à quarta parte da inicial.



      Para que essa população fique reduzida à quarta parte da inicial devemos ter:
      $P(t) = \dfrac{1}{4} P_0$.
      Substituindo a expressão de $P(t)$:
      $P_0 2^{-0,25 t} = 0,25 P_0$.
      Com essa expressão podemos encontrar o valor de $t$.
      $2^{-0,25 t} = 0,25$.
      Aplicando $log_2$ dos dois lados:
      $\log_2 (2^{-0,25 t}) = \log_2(0,25)$.
      Utilizando propriedade de $\log$:
      $-0,25 t \log_2 2 = \log_2(0,25)$.
      $t = \dfrac{\log_2(0,25)}{-0,25}$.
      $t = 8$ anos.


      1285   

      Quais das seguintes funções f têm descontinuidade removível em $a$? Se a descontinuidade for removível em $a$, encontre a função $g$ que é igual a $f$ para $x\neq a$ e contínua em $a$.


      $f(x)=\frac{x^{2}+2x-8}{x+2}$, $a=-2$.
      $f(x)=\frac{x-7}{\vert x-7 \vert}$, $a=7$.
      $f(x)=\frac{3- \sqrt{x}}{9-x}$, $a=9$.


      934   

      Calcule:

      1. $log_3 (36) +log_3 (6)$
      2. $8^{\frac {2} {3}}+\sqrt{100}+2^{2^3}+2^{(2^3)}$


      696   

      Calcule a integral $\int {\dfrac{x}{\left( x-1\right) \left( x+2\right) }}dx$.


      1502   

      Esboce as curvas exponenciais transladadas:
      $y=1-e^x$ e $y=1-e^{-x}$.


      155   

      Determine todas as funções contínuas $f:\mathbb{R} \to \mathbb{R}$ tais que $f(x+y)=f(x)f(y)$ para quaisquer x, y reais.


      556   

      Determine os intervalos de decrescimento e crescimento e esboce o gráfico da seguinte função  $f\left( x\right) =e^{2x}-e^{x}$.


      1910   

      Ache o volume do sólido cuja base é a região limitada pelas curvas $y=x$ e $y=x^2$ cujas secções transversais perpendiculares ao eixo $x$ são quadrados.


      728   

      Calcule o seguinte limite:

      $\lim\limits_{x\rightarrow \infty }\dfrac{1-2^{x}}{1-3^{x}}$.


      $0$.


      1714   

      1. Defina $\displaystyle \lim_{x \to a^+} f(x) = \infty$ e $\displaystyle \lim_{x \to a^-} f(x) = \infty$. Se estiver muito difícil, escreva em palavras.

      2. Mostre que $\displaystyle \lim_{x \to 0^+} \dfrac{1}{x} = \infty$.

      3. Mostre que $\displaystyle \lim_{x \to 0^+} f(x) = \infty$ se e somente se $\displaystyle \lim_{x \to \infty} f\left(\dfrac{1}{x}\right) = \infty$.


      689   

      Calcule a integral $\int \cos ^{3}xdx$.


      $\frac{3}{4} x \sin (x)+\frac{1}{12} x \sin (3 x)+\frac{3 \cos (x)}{4}+\frac{1}{36} \cos(3 x)$


      606   

      Encontre todos os números reais que satisfazem cada uma das desigualdades abaixo. Dê o intervalo solução e ilustre a solução sobre a reta real.

      1. $(2-5x)^{20}>0$

      2. ${\frac{x-3}{x-5}}>0$


      1110   

      Encontre $f(x)$ que satisfaça o seguinte problema de valor inicial:
        $f'(x) = 3x+2$ e $f(0)= 7$


       $\frac{3 x^2}{2}+7 x+7$


      1303   

      Determine os valores de $\lambda$ que tornam contínua a função $g: \left( 0,\pi\right)\mathbb{\rightarrow R},$ dada por
        \[
        g\left( x\right) =\left\{
        \begin{array}{c}
        \tan \left( x\right) \mbox{ se }x\neq \dfrac{\pi }{2} \\
        \lambda \mbox{ se }x =  \dfrac{\pi }{2}
        \end{array}
        \right.
        \]


      1346   

      Mostre que existe um número real que é igual a soma de seu cubo e de seu quadrado mais um. Justifique sua resposta.



      Dizer que um número é igual a soma de seu cubo e de seu quadrado mais um significa dizer que $x=x^{3}+x^{2}+1$ ou, equivalentemente, que $f\left(  x\right)  =x^{3}+x^{2}-x+1=0.$

      Mas $f\left(  -2\right)  =\left(  -2\right)^{3}+\left(  -2\right)  ^{2}-\left(  -2\right)  +1=-1$ e $f\left(  0\right)=1$.

      Como $f\left(  x\right)  $ é contínua, pelo Teorema do Valor Intermediário, existe $-2<x<0$ tal que $f\left( x\right)  =0$.

      Resolução Alternativa:

      Uma vez definida $f(x)$, pode-se ver que $\lim_{x\rightarrow+\infty}f\left(  x\right)=+\infty$ e $\lim_{x\rightarrow-\infty}f\left(  x\right)  =-\infty $. Como $f\left(  x\right)$ é contínua, pelo Teorema do Valor Intermediário, existe $x$ tal que $f\left(x\right)  =0$.


      846   

      Calcule a derivada da função:

      $y=\dfrac{e^{\sec \sqrt{x}}}{x}$.


      $y'=\dfrac{(\tan x) e^{\sec x} \sec x)}{\sqrt{x}} - \dfrac{e^{\sec x}}{(2 x^{3/2})}$.


      1660   

      Calcule a seguinte integral:

      $\int{x\sin{\frac{x}{2}}dx}$.


      $4sin(x/2)-2xcos(x/2)+C$


      581   

      Encontre todos os números reais que satisfazem a cada uma das desigualdades abaixo. Dê o intervalo solução e ilustre a solução sobre a reta real.

      1. $|x+5|\geq \sqrt{2}$

      2. $|x-1|\leq |x+1|$


      1095   

      Avalie a seguinte integral indefinida:
        $\int \frac{1}{3t^2}\  dt$


        $-1/(3t)+C$


      534   

      Dois corredores iniciaram uma corrida ao mesmo tempo e terminaram a corrida empatados. Prove que os dois corredores estiveram à mesma velocidade $v^*$, ainda que talvez em instantes diferentes da corrida.


      1531   

      Em um gerenciamento de estoques, o custo médio semanal de pedidos, pagamentos e armazenamento de mercadoria é dado por:
      $$A(q)=\dfrac{km}{q}+cm+\dfrac{hq}{2},$$
      onde $q$ é a quantidade de produtos pedida em períodos de baixa no estoque; $k$ é o custo (fixo) da colocação de um pedido; $c$ é o custo (também fixo) de cada item; $m$ é a quantidade de itens vendidos por mês; e $h$ é o custo mensal para manter cada item (custos de espaço, seguro, etc). Determine $dA/dq$ e $d^2A/dq^2$. Interprete os resultados.


      1101   

      Avalie a seguinte integral indefinida:
        $\int (2t+3)^2\  dt$


        $4/3t^3+6t^2+9t+C$


      927   

      A partir de um ponto, observa-se o topo de um prédio sob um ângulo de $30^0$. Caminhando $23$m em direção ao prédio, atingimos outro ponto, de onde se vê o topo do prédio segundo um ângulo de $60^0$. Desprezando a altura do observador, calcule, em metros, a altura do prédio.


      1764   

      Prove que $\sinh'(x)=\cosh(x)$.


      929   

      Calcule, apresentando todos os cálculos e/ou justificativas.

      1. $log_2 (1024)+sen^2(40)+cos^2(40)$
      2. $log_\pi [sen(30^0)+cos(60^0)]$


      906   

      Perguntei a idade de minha professora de Matemática. Ela me contou e falou também a idade da filha, mas disse isso de modo enigmático por meio da expressão: "A soma de minha idade com a da minha filha é 44 anos. Dez anos atrás, eu tinha o triplo da idade dela."

      1. "Traduza" a primeira frase da expressão da professora por uma equação, representando por $x$ a idade da professora e por $y$, a idade de sua filha.
      2. Faça o mesmo com a segunda frase.
      3. Resolva o sistema obtido e dê a idade da professora e a de sua filha.



      Seja $x$ a idade da professora e $y$ a idade da filha. Temos, portanto

      1. $x+y=44$
      2. $x-10=3(y-10)$, ou, reescrevendo com as incógnitas do lado esquerdo, $x-3y=-20$
      3. Resolver $\begin{pmatrix}  1 & 1 \\ 1 &-3 \end{pmatrix}  \begin{pmatrix}  x\\ y \end{pmatrix}= \begin{pmatrix}  44 \\ -20 \end{pmatrix}$ nos dá $\begin{pmatrix}  x\\ y \end{pmatrix}= \begin{pmatrix}  28 \\ 16 \end{pmatrix}$


      1107   

      Este problema busca analisar o porquê de
        \begin{equation*}
          \int{\frac{1}{x}\ dx} = ln\left|x\right| + C
        \end{equation*}

      1. Qual o domínio de $y = ln\ x$?
      2. Calcule $\frac{d}{dx}(ln\ x)$
      3. Qual o domínio de $y = ln(-x)$?
      4. Calcule $\frac{d}{dx}\left(ln(-x)\right)$
      5. Com base nos itens anteriores, explique o resultado apresentado no início deste problema.


      204   

      Utilizando o gráfico a seguir, avalie os seguintes limites

      Construa o gráfico da função 1 e da função 2 usando um único plano cartesiano disponível a seguir

      1. $ \lim\limits_{x\to -2^-} f(x)$
      2. $ \lim\limits_{x\to -2^+} f(x)$
      3. $ \lim\limits_{x\to -2} f(x)$
      4. $f(-2)$
      5. $ \lim\limits_{x\to 2^-} f(x)$
      6. $ \lim\limits_{x\to 2^+} f(x)$
      7. $ \lim\limits_{x\to 2} f(x)$
      8. $f(2)$


      1. $2$
      2. $2$
      3. $2$
      4. $0$
      5. $2$
      6. $2$
      7. $2$
      8. Indefinido


      53   

      Calcule os limites indicados dividindo o numerador e o denominador por uma potência conveniente de $x$. Como esses limites se relacionam com as mais altas potências do numerador e do denominador?

      1. $\lim\limits_{x\rightarrow\infty}\frac{x^4-2}{3x^4-x^3+1}$

      2. $\lim\limits_{x\rightarrow\infty}\frac{\sqrt{2x^6-2x+1}}{x^3-x^2+2}$

      3. $\lim\limits_{x\rightarrow -\infty}\frac{\sqrt{x^2-3}}{x+1}$


      601   

      Para cada uma das afirmações abaixo, demonstre-a, se verdadeira, ou dê um contra-exemplo, se for falsa.

      1. $x\neq y\Longrightarrow |x|\neq |y|$.

      2. $|x-y|\geq |x|-|y| \forall x,y\in \mathbb{R}$


      1904   

      Mostre que o comprimento de arco total da elipse $x=a \cos t$, $y=b \sin t$, $0 \leq t \leq 2\pi$, para $a>b>0$ é dado por $4\displaystyle\int_{0}^{\pi/2} \sqrt{1+3\sin^3 t}dt$.


      180   

      Seja $n$ um número natural dado por $n= 2000 \cdot x$. Determine um possível valor para $x$ que torna $n$ um quadrado perfeito.



      Sabemos que quando decompomos um quadrado perfeito em fatores primos, os expoentes dos números primos na decomposição são necessariamente múltiplos de $2$. Ora, decompondo $2000$ obtemos $2000=2^4 \cdot 5^3$. Se multiplicarmos $2000$ por $5$ obteremos $10000=2^4 \cdot 5^4$, que é um quadrado perfeito, a saber $100^2$. Neste caso tomamos $x=5$, mas há infinitas outras possibilidades!


      1491   

      Dê o domínio e esboce o gráfico das seguintes funções:

      1. $f(x)=2/x$
      2. $f(x)=\dfrac{2}{x-1}$


      219   

      Avalie os seguintes limites para a função definida por partes
      $ f(x) = \left\{\begin{array}{ccc}
      x+1, & &  \text{ se } x\leq 1 \\
      x^2-5, & & \text{ se }  x>1
      \end{array}
      \right.$

      1. $ \lim\limits_{x\to 1^-} f(x)$

      2. $ \lim\limits_{x\to 1^+} f(x)$

      3. $ \lim\limits_{x\to 1} f(x)$

      4. $f(1)$


      1. 2

      2. $-4$

      3. Não existe.

      4. 2


      336   

      Se $a$, $b$, $c$ são as raízes de $x^3-x-1=0$, calcule o valor de $\frac{1+a}{1-a}+\frac{1+b}{1-b}+\frac{1+c}{1-c}.$


      1747   

      Seja

      $$f(x)=\left\{\begin{array}{ll}\dfrac{\sin(x)}{x}, &\text{ se } x\neq0,\\1, &\text{ se } x=0\end{array}\right..$$

      Começando com o polinômio de Taylor de ordem $2n+1$ para $\sin x$, junto com a estimativa para o termo de resto $R_{n,1}(x)=\dfrac{f^{(n+1)}(t)}{(n+1)!}(x-a){n+1}$, mostre que:

      $$f(x) = \left( 1-\dfrac{x^2}{3!}+\dfrac{x^4}{5!}+\ldots+(-1)^n\dfrac{x^{2n}}{(2n+1)!} + R_{2n,0,f}(x) \right),$$

      onde:

      $$|R_{2n,0,f}(x)| \leq \dfrac{|x|^{2n+1}}{(2n+2)!}.$$


      1733   

      Suponha que as equações do movimento de um avião de papel, durante os $12$ segundos iniciais de vôo, são $$ x=t-2\sin t, \quad y=2-2\cos t\quad(0\leq t\leq 12). $$Quais são os pontos mais alto e mais baixo da trajetória  e em que instantes eles são atingidos?


      1687   

      Determine o comprimento da curva a seguir no intervalo especificado.

      $y=x^{3/2},\quad 0 \leq x \leq 4$


      798   

      Calcule $f'\left( x\right) $, pela definição:

      $f\left( x\right) =\dfrac{x}{x+1}$.


      $f'(x) = \dfrac{1}{(x+1)^2}$.


      1762   

      Prove que $\tanh^2(x)+\dfrac{1}{\cosh^2(x)}=1$.


      579   

      Esboce o gráfico da função $f\left(x\right) =\dfrac{x^{2}-x+1}{2x-2}$, determinando o domínio, pontos de máximo e de mínimo, pontos de inflexão e assíntotas. Explicite o valor que a função assume nos pontos em questão. Justifique o seu raciocínio.


      1330   

      Considere a seguinte função:
      \begin{equation*}
        f(x)= \begin{cases}
              (x-b)^2 -2, \quad x\geq 0 
              a\sin x,\quad x<0.
              \end{cases}
      \end{equation*}

      1.  Encontre os valores de $a$ e $b$ tais que $f(x)$ seja contínua e diferenciável para todo $x\in\mathbb{R}.$
      2.  Encontre o valor de $b$ tal que a reta tangente $t$ à curva $f(x)$ no ponto $x=1$ possui inclinação 2. Escreva a equação de $t.$
      3.  Encontre o valor de $a$ tal que a reta $s$ normal à reta tangente à $f(x)$ no ponto $x=-\pi$ possui inclinação $-\frac{1}{2}$. Escreva a equação de $s$.



       Observamos que para todo $x\geq 0$ a função $(x-b)^2 -2$ é contínua e que para todo $x<0$ também a função $a\sin x$ é contínua. Logo, temos que verificar a continuidade no ponto $x=0$, isto é, deve acontecer que

      $\lim_{x\rightarrow 0^-}f(x)= \lim_{x\rightarrow 0^+}f(x),$

      ou seja,

      $\lim_{x\rightarrow 0^-}a\sin x=\lim_{x\rightarrow 0^+} (x-b)^2 -2.$

      A relação anterior implica que $0= b^2-2$, ou seja $b=\pm\sqrt{2}.$ \\

      Afim de achar o valor de $a$, encontramos a derivada de $f(x)$. Observamos que, sendo $a\sin x$ e $(x-b)^2 -2$ funções diferenciáveis para todo $x\in \mathbb{R}$, a derivada de $f(x)$ é a seguinte:

      $f'(x)= \begin{cases}     
      2(x-b), \quad x> 0 
      a\cos x,\quad x<0.
      \end{cases}$

      Como queremos que $f(x)$ seja diferenciável no ponto $x=0$ também, temos que impor

      $\lim_{x\rightarrow 0^-}f'(x)= \lim_{x\rightarrow 0^+}f'(x),$

      ou seja,

      $\lim_{x\rightarrow 0^-}a\cos x= \lim_{x\rightarrow 0^+}2(x-b).$

      A relação anterior implica que $a= -2b$, então as duplas de valores para os quais $f(x)$ é contínua e diferenciável para todo $x\in \mathbb{R}$, são $(a,b)= (2\sqrt{2}, -\sqrt{2})$ ou $(a,b)=(-2\sqrt{2}, \sqrt{2}).$

      Usando a função derivada calculada no ponto anterior, temos que $f'(1)= 2(1-b),$ então, como a inclinação da reta tangente deve ser 2, obtemos $2(1-b)=2$ e logo $b= 1$. A equação de $t$ é $y= f(1)+ f'(1)(x-1)$, isto é $y= -2+1\cdot(x-1)=x-3.$

      Usando a função derivada calculada no ponto anterior, temos que $f'(-\pi)= a\cos (-\pi)= -a,$ então, como a inclinação da reta normal $s$ é $-\frac{1}{2}$, deve ser $-a=2$, ou seja $a=-2$. A equação de $s$ é $y= f(-\pi)-\frac{1}{2}(x+\pi)$, isto é $y= -\frac{1}{2}x -\frac{1}{2}\pi.$


      122   

      De acordo com o gráfico de $f(x)$, avalie a continuidade da função em $x=0$.

      Construa o gráfico da função 1 e da função 2 usando um único plano cartesiano disponível a seguir


      $f$ não é contínua em $x=0$.


      1830   

      Esboce o gráfico completo da função $\displaystyle f(x)=x\tan x,\ -\pi/2<x<\pi/2$, e localize todos os extremos relativos e pontos de inflexão. Utilize um recurso computacional gráfico a fim verificar seu resultado.


      1383   

      De acordo com s primeira lei de Kirchhoff para circuitos elétricos $V=RI+L(dI/dt)$, onde as constantes $V$, $R$ e $L$ denotam a força eletromotriz, a resistência e a indutância, respectivamente, e $I$ denota a corrente no instante $t$. Se a força eletromotriz é interrompida no instante $t=0$ e se a corrente é $I_0$ no instante da interrupção, prove que $I=I_0 e^{-Rt/L}$.


      1339   

      Usando os limites fundamentais, encontre o limite $\lim\limits_{x\rightarrow0}\frac{cotg (2x)}{cossec (x)}$.


      $1/2$.


      796   

      Calcule $f'\left( x\right) $, pela definição:

      $f\left( x\right) =x^{2}+x$.


      $f'(x)=2x + 1$.


      1776   

      1. Calcule a integral $\displaystyle \int (5x-1)^2 \, dx$ elevando ao quadrado e depois integrando as potências de $x$.

      2. Calcule agora a integral utilizando a substituição $u=5x-1$.

      3. As duas respostas obtidas são iguais? São equivalentes de alguma forma? Justifique.


      187   

      Prove que $\sqrt{2}+\sqrt{3}$ é irracional.


      A teoria necessária para resolver esta questão pode não ser abordada em alguns cursos de Cálculo 1. Sendo, também pertinente, às disciplinas Teoria dos Números e Análise Real I.
      Para aprofundar seus conhecimentos, dentro do escopo de Cálculo 1, recomendamos a leitura do Cap. 1 de Guidorizzi, vol. 1 e /ou o Prólogo de Spivak (vide Bibliografia de Cálculo 1).


      1713   

      O gráfico a seguir representa o número de indivíduos de uma população ao longo do tempo.

      1. Pode-se dizer que há uma assíntota horizontal para essa população? Justifique.

      2. O que essa assíntota representa em termos biológicos? (Isto é, qual a interpretação da assíntota em função da população?)

      Construa o gráfico da função 1 e da função 2 usando um único plano cartesiano disponível a seguir


      789   

      Demonstre que as retas tangentes às curvas $4y^3-x^2y-x+5y=0$ e $x^4-4y^3+5x+y=0$ na origem são perpendiculares.


      1140   

      Considere a função $f(x)=\sin x.$

      1.  Escreva o polinômio de Taylor de $f(x)$ até a terceira ordem.
      2.  Usando o polinômio de Taylor, encontre o valor do seguinte limite: $\lim_{x\rightarrow 0}\frac{\sin x-x+2x}{3x^5}.$


      1703   

      Uma fonte de imprecisão nos cálculos feitos por computadores é a {\it subtração catastrófica}. Tal erro ocorre quando dois números aproximadamente iguais são subtraídos, e o resultado é usado como parte de outro cálculo.
      Um exemplo: $(0,123456789012345-0,123456789012344)\times 10^{15}=1$.
      Mas, na calculadora, obteríamos zero como resposta a esse cálculo pois ela armazena apenas 14 dígitos e os 14 primeiros dígitos são idênticos. Por vezes, pode-se evitar a subtração catastrófica fazendo um rearranjo algébrico das fórmulas. De todo modo, o melhor é estar atento à sua ocorrência, portanto, tome cuidado para resolver este exercício.

      1. Seja $f(x)=\dfrac{x-\sin x}{x^3}$. Faça uma conjectura sobre o limite de $f$ quando $x \to 0^+$ calculando $f$ nos pontos $x=0,1$, $0,01$, $0,001$, $0,0001$.

      2. Calcule $f$ nos pontos $x=0,00001$, $0,000001$, $0,0000001$, $0,00000001$, $0,000000001$, $0,0000000001$, e faça outra conjectura.

      3. Que falha isso revela sobre o uso da evidência numérica para fazer conjecturas sobre limites?

      4. Se você dispuser de um sistema de computação algébrica, um programa que pode efetuar cálculos numéricos ou simbólicos, use-o para mostrar que o valor exato desse limite é $\dfrac{1}{6}$. (Aqui, eu não posso pedir para calcular o limite à mão, de fato?)


      1137   

      Escreva a taxa de crescimento de $y$ em termos das taxas de crescimento de $k$, $l$ e $m$ para os seguintes casos. Assuma $\beta$ como uma dada constante.

      1. $y=k^{\beta }$ 
      2. $y=k/m$


      584   

      Encontre todos os números reais que satisfazem a cada uma das  desigualdades abaixo. Dê o intervalo solução e ilustre a solução sobre a reta real.

      1. $|2x-3|<5$

      2. $|4-x|\geq 1$


      726   

      Calcule o seguinte limite:

      $\lim\limits_{x\rightarrow \infty }\left( 2^{x}-3^{x}\right) $.


      $-\infty$.


      1117   

      Dado que os números no gráfico representam o valor das áreas demarcadas, avalie as seguintes integrais:

      Construa o gráfico da função 1 e da função 2 usando um único plano cartesiano disponível a seguir

      1. $ \int_0^2 f(x)\ dx$
      2. $ \int_2^4 f(x)\ dx$
      3. $ \int_0^4 f(x)\ dx$
      4. $ \int_0^1 f(x)\ dx$


      1. $4/\pi$
      2. $-4/\pi$
      3. $0$
      4. $2/\pi$


      1824   

      Discuta as hipóteses necessárias para que se possa aplicar a Regra de L'Hospital.


      1529   

      O projetista de um balão esférico (um projetista excêntrico) de ar quente com $10m$ de diâmetro quer suspender uma gôndola a $2m$ abaixo da parte inferior do balão, presa por cabos tangentes à superfície deste. Dado que os cabos, saindo da lateral do balão, tangenciam a superfície do mesmo nos pontos $(4,-3)$ e $(-4,-3)$, qual deve ser a largura da gôndola?

      Construa o gráfico da função 1 e da função 2 usando um único plano cartesiano disponível a seguir


      1792   

      Utilize uma substituição trigonométrica para mostrar que $\displaystyle \int \dfrac{1}{u^2 \sqrt{a^2 - u^2}} \, du = -\dfrac{1}{a^2 u} \sqrt{a^2-u^2} + C $.


      1104   

      Avalie a seguinte integral indefinida:
        $\int \frac{5^t}{2}\  dt$


        $\frac{5^t}{2\ln 5}+C$


      559   

      Determine os intervalos de decrescimento e crescimento e esboce o gráfico da seguinte função  $f\left( x\right) =\dfrac{x}{x^{2}+1}$.


      672   

      Calcule a seguinte integral $\int \ln xdx$.


      $x(lnx-1)+C$


      1694   

      Uma empresa deseja lançar uma tigela esmaltada de branco por dentro e de vermelho por fora. A camada de esmalte terá $0,5mm$ de espessura antes de ir ao forno. O departamento de produção quer saber a quantidade de cada esmalte que precisará dispor para produzir $5000$ tigelas. Ignorando desperdício e matéria prima não utilizada, dê a sua resposta em litros. Lembre-se de que $1\ cm^3 = 1m\ell$, logo $1\ell=1000cm^3$.

      Construa o gráfico da função 1 e da função 2 usando um único plano cartesiano disponível a seguir


      84   

      Calcule os limites:

      1. $\lim\limits_{x\to\pi/4} \cos x\sin x$

      2. $\lim\limits_{x\to0} \ln x$

      3. $\lim\limits_{x\to3} 4^{x^3-8x}$


      58   

      Calcule o limite $\lim\limits_{x\rightarrow -3}\frac{1-x}{\sqrt{x^2+2}}$.



      Como a função está definida em $x=-3$, o limite pode ser calculado diretamente por substituição:
      $\lim\limits_{x\rightarrow -3}\dfrac{1-x}{\sqrt{x^2+2}} = \dfrac{1-(-3)}{\sqrt{(-3)^2+2}} = \dfrac{4}{\sqrt{11}}$.


      1897   

      Se $f_{med}[a,b]$ denota o valor médio de $f$ no intervalo $\left[a,b\right]$ e $a<c<b$, mostre que
      \begin{equation*}
      f_{med}[a,b]=\dfrac{c-a}{b-a}f_{med}[a,c]+\dfrac{b-c}{b-a}f_{med}[c,b]
      \end{equation*}


      699   

      Calcule a integral imprópria $\int_{0}^{\infty }x^{2}e^{-x}dx$


      $2$.


      138   

      Para a função a seguir, dê os intervalos nos quais ela é contínua:

         $ h(k) = \sqrt{1-k}+\sqrt{k+1}$.


      $[-1,1]$


      56   

      Verifique se os seguintes limites existem. Explique.

      1. $\lim\limits_{x\rightarrow\infty}2^{1/x}$.

      2. $\lim\limits_{t\rightarrow\infty}\sin x$.

      3. $\lim\limits_{x\rightarrow 2^-}\tan^{-1}\left(\frac{1}{2x-4}\right)$.


      911   

      Sabendo-se que $\frac{x-a}{x^2+1} > \frac{x+a}{x^2}$ para todo $x$ real, determine o intevalo a que pertence o número real $a$.


      1709   

      Encontre os seguintes limites em termos do número $\alpha = \displaystyle \lim_{n \to 0} \dfrac{\sin x}{x}$.

      1. $\displaystyle \lim_{x \to \infty} \dfrac{\sin x}{x}$.

      2. $\displaystyle \lim_{x \to \infty} x \sin \left(\dfrac{1}{x}\right)$.


      30   

      Calcule o limite $\lim\limits_{x\rightarrow \infty }\dfrac{5x^{4}-2x+1}{4x^{4}+2x+3}$.


      $5/4$


      1659   

      Demonstre que se $k$ é uma constante positiva, então a área entre o eixo $x$ e um arco da curva $y=\sin kx$ é $2/k$.


      1267   

      Calcule a seguinte integral:
        $ \int x^2\sin (\pi x)dx$.


      $\dfrac{(2-\pi^2x^2)cos(\pi x+2 \pi xsin(\pi x)}{\pi^3}+C$.


      1825   

      Demonstre a Regra de L'Hospital para a indeterminação da forma $\displaystyle\dfrac{0}{0}$.


      1722   

      A aproximação $(1+x)^k \approx 1+kx$ pode ser utilizada para cálculos rápidos.

      1. Mostre porque esta aproximação é boa e use-a para fazer uma estimativa simples de $(1,001)^{37}$.

      2. Compare sua estimativa com a obtida por meio de algum recurso computacional (pode ser uma calculdadora científica).

      3. Agora utilize esta aproximação para calcular $(1,1)^{37}$ e compare com o recurso computacional. O que acontece neste caso? Justifique.


      1860   

      Considere uma cápsula esférica de $1cm$ de espessura cujo volume é igual ao volume do espaço oco dentro dela. Use o método de Newton para calcular o raio externo da cápsula com duas casas decimais de precisão.


      1717   

      Dizemos que duas famílias de curvas são trajetórias ortogonais uma da outra se cada curva de uma família for ortogonal a cada curva da outra. Faça um esboço de gráfico da família de curvas $xy=c$ e da família $x^2-y^2=k$ no mesmo plano cartesiano, para alguns valores de $c$ e $k$ reais (se necessário, utilize algum recurso computacional). Mostre que estas famílias (de hipérboles) são ortogonais uma da outra. (Sugestão: retas tangentes são perpendiculares em um ponto de interseção se as suas inclinações são recíprocas negativas uma da outra.)


      94   

      Calcule o limite a seguir. Justifique as passagens.

      $\lim\limits_{x\rightarrow 3^{+}}\dfrac{5}{x-3}$



      Seja $u=x-3$. Temos que $u$ tende a $0$ por valores positivos se $x$ tende a $3$ por valores maiores do que $3$. Logo, \begin{equation*} \lim\limits_{x\rightarrow 3^{+}}\dfrac{5}{x-3}=\lim\limits_{u\rightarrow 0^{+}}\dfrac{5}{u}\text{.} \end{equation*} Mas dado $M>0$, temos que se $0<u<\dfrac{5}{M},$ então $M<\dfrac{5}{u}$ e temos que, por definição, $\lim\limits_{u\rightarrow 0^{+}}\dfrac{5}{u}=\infty $.


      324   

      Um encanador $A$ cobra por serviço feito um valor fixo de $R\$ 60,00$, mais $R\$ 10,00$ por hora de trabalho. Um outro encanador $B$ cobra um valor fixo de $R\$40,00$ mais $R\$15,00$ por hora de trabalho. Considerando o menor custo para a realização de um trabalho, avalie a decisão de se contratar um ou outro encanador.


      78   

      Explique, usando suas palavras, o que significa escrever $\lim\limits_{x\to c} b = b$.


      1879   

      Discuta a seguinte "demonstração":  Dada a integral $\displaystyle\int (1/x)dx$, seja $dv=dx$ e $u=1/x$, de modo que $v=x$ e $du=(-1/x^2)dx$. Então $\displaystyle\int (1/x)dx=(1/x)x-\displaystyle\int x (-1/x^2) dx \Rightarrow \displaystyle\int (1/x)dx=1+\displaystyle\int (1/x)dx \Rightarrow 0=1.$


      1381   

      Usa-se a técnica do carbono-14 para determinar a idade de espécimes arqueológicos ou geológicos. Este método baseia-se no fato de que o carbono-14, isótopo instável ($^{14}C$) está presente no $CO_2$ na atmosfera. As plantas assimilam carbono da atmosfera; quando morrem o $^{14}C$ acumulado começa a decair, com uma meia vida de aproximadamente 5700 anos. Medindo-se a quantidade de $^{14}C$ que resta em um espécime, é possível determinar quando o organismo morreu. Suponha que um osso fóssil acuse 20\% da quantidade de $^{14}C$ presente em um osso dos dias atuais. Dê uma aproximação da idade do osso fóssil.


      87   

      Calcule os limites:

      1. $\lim\limits_{x\to2} \frac{x^2-10 x+16}{x^2-x-2}$

      2. $\lim\limits_{x\to-2} \frac{x^2-5 x-14}{x^2+10 x+16}$

      3. $\lim\limits_{x\to-1} \frac{x^2+9 x+8}{x^2-6 x-7}$


      1. $-2$
      2. $-3/2$
      3. $-7/8$


      1516   

      Dê os domínios e esboce os gráficos de $f+g$ e $\dfrac{g}{f}$ no seguinte caso:

      $f(x)=\left\{\begin{array}{ll}
      1, & \text{se x é racional} \\
      -1, & \text{se x é irracional}  \end{array}\right.$ 

      e

      $g(x)=\left\{\begin{array}{ll}
      -1, & \text{se x é racional} \\
      1, & \text{se x é irracional}  \end{array}\right.$


      515   

      Considere as funções trigonométricas hiperbólicas:

      \begin{equation*}  \sinh x=\dfrac{e^{x}-e^{-x}}{2};\;\cosh x=\dfrac{e^{x}+e^{-x}}{2}\text{.} \end{equation*}

      1. Mostre que $\cosh ^{2}x-\sinh ^{2}x=1$.

      2. Mostre que $\left( \sinh ^{\prime }x\right) ^{2}-\left( \cosh^{\prime }x\right) ^{2}=1$.


      1186   

      Determine a derivada da seguinte função:
        $f\left( x\right) =\cos \left( x^{-2}\right)+x^{3}e^{-3x}.$


      $f'(x) =  -3 e^{-3 x} x^3 + 3 e^{-3 x} x^2 + (2 \sin(x^{-2}))/x^3$.


      1539   

      Suponha que um meteorito pesado está a $s$ quilômetros do centro da Terra, e que sua velocidade de entrada na atmosfera terrestre seja inversamente proporcional a $\sqrt{s}$. Mostre que a aceleração do meteorito é inversamente proporcional a $s^2$ e interprete o resultado.


      195   

      Prove que não existe inteiro entre $0$ e $1$.

      A teoria necessária para resolver esta questão pode não ser abordada em alguns cursos de Cálculo 1. Sendo, também pertinente, às disciplinas Teoria dos Números e Análise Real I.

      Para aprofundar seus conhecimentos, dentro do escopo de Cálculo 1, recomendamos a leitura do Cap. 1 de Guidorizzi, vol. 1 e /ou o Prólogo de Spivak (vide Bibliografia de Cálculo 1).


      Dica: Suponha que exista um número inteiro $n$ tal que $0<n<1$. Então...


      889   

      Dados dois números reais distintos $a$ e $b$, podemos definir uma função $f(x)$ que chamaremos "distância ao conjunto $\left\lbrace a,b \right\rbrace$" da seguinte forma: $f(x)$ é igual ao menor dos números $|x-a|$ ou $|x-b|$. Se $a=-b=1$, construa o gráfico de $f(x)$.


      1798   

      Calcule a integral $\displaystyle \int \dfrac{x^5-x^4-2x^3+4x^2-15x+5}{(x^2+1)^2(x^2+4)} \, dx$.


      738   

      Encontre o volume do sólido obtido pela rotação da região limitada pela curva dada em torno do eixo especificado. Esboce a região e o sólido.
        $y=e^{x},y=0,x=0,x=1$ ao redor do eixo $x.$


      79   

      Explique, usando suas palavras, o que significa escrever $\lim\limits_{x\to c} x = c$.


      199   

      Mostre que qualquer intervalo de $R$ contém algum número irracional.

      A teoria necessária para resolver esta questão pode não ser abordada em alguns cursos de Cálculo 1. Sendo, também pertinente, às disciplinas Teoria dos Números e Análise Real I.

      Para aprofundar seus conhecimentos, dentro do escopo de Cálculo 1, recomendamos a leitura do Cap. 1 de Guidorizzi, vol. 1 e /ou o Prólogo de Spivak (vide Bibliografia de Cálculo 1).


      162   

      Use o Teorema do Confronto para demonstrar que $\lim\limits_{x \to 0} \cos{x} = 1$.


      1589   

      Suponha que $y=f(x)$ seja derivável em $x=a$ e que $g(x)=m(x-a)+c$ seja uma função linear, em que $m$ e $c$ sejam constantes. Se o erro entre $f$ e $g$, $E(x) = f(x)-g(x)$ for suficientemente pequeno perto de $x=a$, poderemos pensar em utilizar $g$ como aproximação linear de $f$ ao invés da linearização $L(x) = f(a)+f'(a)(x-a)$.

      1. Interprete as expressões $E(a)=0$ e $lim_{x\to a} \dfrac{E(x)}{x-a}=0$.
      2. Mostre que impondo as condições $E=0$ e $lim_{x\to a} \dfrac{E(x)}{x-a}=0$, temos $g(x)=f(a)+f'(a)(x-a)$. Interprete o resultado, relacionando com o item anterior.


      1876   

      Prove que $\displaystyle\int x^m \sin(x)dx=-x^m \cos(x)+m \displaystyle\int x^{m-1} \cos(x)dx$.


      167   

      Uma importante aplicação do Teorema do Valor Intermediário é o Método da Bissecção.

      Suponha que estamos interessados em encontrar as raízes de uma função contínua $f(x)$. O Método da Bissecção é uma alternativa que pode resultar em boas aproximações para as raízes, após sucessivas aplicações do método.

      Para iniciar o método, precisamos encontrar dois valores $a$ e $b$ tais que $f(a) \cdot f(b) < 0$.

      Sem perda de generalidade, vamos assumir $f(a) < 0$, $f(b) > 0$ e $a<b$. O Teorema do Valor Intermediário afirma que existe um valor $c$ no intervalo $[a,b]$ tal que $f(c) = 0$. O teorema não afirma nada a respeito da localização de $c$ dentro do intervalo, apenas que ele existe.

      O Método da Bissecção é, portanto, uma maneira sistemática de obter este valor $c$. Seja $d=\frac{a+b}{2}$ o meio do intervalo. Existem três possibilidades:

      1. $f(d) = 0 $ - Por sorte, encontramos a raiz e não é necessário prosseguir com o método.
      2. $f(d) < 0$ - Como $f(b)>0$, sabemos que há uma raiz no intervalo $[d,b]$. Este intervalo tem metade do tamanho do intervalo original, então estamos mais próximos de obter uma boa aproximação para a raiz.
      3. $f(d) > 0$ - Como $f(a)<0$, sabemos que há uma raiz no intervalo $[a,d]$. Novamente, este intervalo tem metade do tamanho do intervalo original, então estamos mais próximos de obter uma boa aproximação para a raiz.

      O Método da Bissecção é a aplicação sucessiva dos passos descritos até que se esteja próximo o suficiente da raiz de $f(x)$ para a aplicação desejada. Nota-se que para o caso em que $f(a)>0$ e $f(b)<0$ o método ainda funciona, mas no caso 2 o intervalo escolhido seria $[a,d]$ e no caso e $[d,b]$ (por quê?).

      Utilize o Método da Bissecção para encontrar as raízes de $f(x) = \cos x -\sin x$ no intervalo $[0.7,0.8]$.


      A raiz aproximada é $x=0.78$.

        Os intervalos utilizados são:

        $[0.7,0.8] \quad [0.75,0.8] \quad [0.775,0.8]$

        $[0.775,0.7875]\quad [0.78125,0.7875]$

        (Alguns passos a mais mostrariam que $0.79$ é melhor, dado que a raiz é $\pi/4 \approx 0.78539$.)


      37   

      Calcule os seguintes limites:

      1. $\lim\limits_{x\rightarrow \infty }3^{x}$

      2. $\lim\limits_{x\rightarrow \infty }\left( 2^{x}-3^{x}\right)$

      3. $\lim\limits_{x\rightarrow \infty }\left( 0,27\right) ^{x}$


      1. $\infty$.

      2. $-1$.

      3. $0$.


      1711   

      É possível mostrar que, sob certas condições, a velocidade $v(t)$ de uma gota de chuva caindo no instante $t$ é:

      $$v(t) = v^\star \left(1-\exp\left(-\dfrac{gt}{v} \right)\right),$$

      onde $g$ é a aceleração da gravidade e $v^\star$ é a velocidade final da gota.

      1. Calcule a velocidade para um tempo muito grande, isto é, calcule $\displaystyle \lim_{t \to \infty} v(t)$.

      2. Considerando $v^\star = 1$m$/$s e $g=9,8$m$/$s$^2$, faça o gráfico de $v(t)$. Quanto tempo levará para a velocidade da gota atingir $99\%$ de sua velocidade final?


      1260   

      Resolva a equação $e^{ax}=Ce^{bx}$, onde $a\neq b$.



      Usando as propriedades básicas da função exponencial temos que:
        \begin{align*}
        e^{ax}  & =Ce^{bx}\\
        & \Leftrightarrow e^{-ax}e^{ax}=e^{-ax}Ce^{bx}\\
        & \Leftrightarrow1=Ce^{(b-a)x}\\
        & \Leftrightarrow\frac{1}{C}=e^{(b-a)x}\\
        & \Leftrightarrow\ln\left(  \frac{1}{C}\right)  =\ln\left(  e^{(b-a)x}\right)
        =\left(  b-a\right)  x\\
        & \Leftrightarrow\frac{-\ln C}{b-a}=\frac{\ln C}{a-b}=x
        \end{align*}


      1806   

      Dê exemplo de uma função definida em $\mathbb{R}$, que não seja contínua em $a$, mas que $\lim\limits_{x \to a^+}f(x)=\lim\limits_{x \to a^-}f(x)$.


      1726   

      Seja $x$ uma função de $t$, isto é, $x=f(t)$, tal que para $t=0$, $x=1$ e para $t=1$, $x=2$. Suponha que $\dfrac{dx}{dt}>0$ para $t\geq0$; $\dfrac{d^2x}{dt^2}<0$ para $0<t<1$ e $\dfrac{d^2x}{dt^2}>0$ para $t>1$. Como você acha que deve ser o gráfico de $f$? Por quê?


      151   

      Seja $f: \mathbb{R} \to \mathbb{R}$ uma função derivável cuja derivada é sempre positiva e tal que $f(0)=1$ e $f(4)=2$. Use o TVM para mostrar que $f(2) \neq 2$.


      746   

      Ache as assíntotas verticais e inclinadas; depois calcule os limites laterais nas assíntotas verticais da função $f\left( x\right) =\frac{x^{3}-3x-1}{x^{2}-x}.$


      1770   

      Calcule $\displaystyle \int x^2 \ln (x+1) \, dx$ utilizando integração por partes.


      $\dfrac{1}{18}(6(x^3+1)ln(x+1)-2x^3+3x^2-6x)+C$.


      1756   

      Avalie a integral $\displaystyle \int_{-1}^1 (x^5+3) \sqrt{1-x^2} \, dx$ sem fazer nenhuma conta.


      1688   

      Determine o comprimento da curva a seguir no intervalo especificado.

      $y=(3/4)x^{4/3}-(3/8)x^{2/3}+5,\quad 0 \leq x \leq 3$


      1320   

      Jane está em um barco a $2 km$ da costa e deseja chegar a uma cidade litorânea, localizada $6 km$ ao longo de uma linha costeira retilínea desde o ponto (na costa) mais próximo do barco. Ela rema a $2 km/h$ e caminha a $5 km/h$. Em que ponto da costa ela deve aportar para chegar à cidade no menor tempo possível?


      563   

      Estude a função $f\left( x\right) =\sqrt{x^{2}-4}$ com relação à concavidade, pontos de inflexão, máximos e mínimos, e esboce o seu gráfico.


      1546   

      Demonstre as seguintes regras de derivação:

      1. $(sec{x})'=sec{x} \cdot tg{x}$
      2. $(cotg{x})'=-cossec^2{x}$
      3. $(cossec{x})'=-cossec{x} \cdot cotg{x}$


      570   

      Estude a função  $f\left( x\right) =\dfrac{\ln x}{x}$ com relação à concavidade, pontos de inflexão, máximos e mínimos, e esboce o seu gráfico.


      597   

      Mostre que $|x|<x^{2}+1,\forall x\in \mathbb{R}$.


      693   

      Calcule o seguinte limite:


      $\lim\limits_{x\rightarrow 0}\dfrac{\sin \left(10x\right) }{\sin \left( 5x\right) }$.


      $2$.


      105   

      Determine os valores de $c$ que tornam contínua a função \[ f\left( x\right) =\left\{ \begin{array} [c]{c} x^{2}+cx,\text{ se }x\leq1\\ \left( cx\right) ^{2}-1=c^{2}x^{2}-1,\text{ se }x>1 \end{array} \right. \text{.} \]

      Justifique sua resposta.


      $c=-1$ ou $c=2$.


      1809   

      Resolva os itens:

      1. Prove que existe $r>0$ tal que $\cos{x}-1<\dfrac{\sin{x}}{x}-1<0$ para $0<|x|<r$.
      2. Calcule $\lim\limits_{x \to 0}\dfrac{x-\sin{x}}{x^2}$.


      691   

      Calcule o seguinte limite

      $\lim\limits_{x\rightarrow \infty }\left( 1+\dfrac{1}{2x}\right)^{x}$.


      $e^{1/2}$.


      809   

      Calcule $f^{\prime }\left( x\right)$:

      $f\left( x\right) =4\sec x+\cot x$.


      $f'(x) = 4 \sec x \tan x - \csc^2 x$.



      Como a derivada da soma de funções é a soma de suas derivadas, temos inicialmente que

      \[ (4\sec x+\cot x)^\prime = (4\sec x)^\prime + (\cot x)^\prime = 4 (\sec x)^\prime + (\cot x)^\prime .\]

      Como $\sec x = \dfrac{1}{\cos x}$, podemos usar a regra do quociente para calcular sua derivada:

      \[(\sec x)^\prime = \left(\dfrac{1}{\cos x}\right)^\prime = \dfrac{(1)^\prime\cdot \cos(x) - 1\cdot (\cos x)^\prime}{(\cos x)^2} =\dfrac{0 - (-\sin x)}{(\cos x)^2} = \tan(x)\sec(x).\]

      De forma análoga, usaremos a regra do quociente para calcular a derivada da função $\cot x$, que é igual a $\frac{\cos x}{\sin x}$:

      \[(\cot x)^\prime = \left(\dfrac{\cos x}{\sin x}\right)^\prime = \dfrac{(\cos x)^\prime\cdot \sin(x) - \cos(x)\cdot (\sin x)^\prime}{(\sin x)^2} =\dfrac{(-\sin x) \sin x - \cos(x)(\cos x)}{(\sin x)^2} = -(\csc x)^2,\]

      em que usamos a identidade trigonométrica fundamental

      \[(\sin x)^2 + (\cos x)^2 = 1\]

      e a identidade $\csc x = \frac{1}{\sin x}$ para obter a cossecante.

      Substituindo as expressões encontradas para as derivadas de $\sec x$ e de $\cot x$ na primeira igualdade, concluímos que

      $f'(x) = 4 \tan(x)\sec(x) - (\csc x)^2$.


      1297   

      Seja $S$ a região entre as curvas $y=x^n$ e $y=x^{n+1}$, onde $n$ é um inteiro, $n\geq 1$.
        Considere o sólido $A_r$ obtido pela rotação de $S$ ao redor do eixo $x=r, r>1$ e considere o sólido $B_r$ obtido pela rotação de $S$ ao redor do eixo $y=r, r>1$. \\
        Calcule o volume $V(A_r)$ de $A_r$, o volume $V(B_r)$ de $B_r$.  Determine, se existir, ${\lim_{r\rightarrow\infty}\frac{V(A_r)}{V(B_r)}}$.


      1707   

      Embora limites como $\displaystyle \lim_{n \to \infty} \sqrt[\leftroot{-2}\uproot{2}n]{n}$ e $\displaystyle \lim_{n \to \infty} a^n$ possam ser avaliados utilizando conhecimentos sobre as funções logaritmo e exponencial, estes não são necessários. Neste exercício vamos calcular esses tipos de limite por meio de argumentos ``elementares''. As ferramentas básicas são desigualdades provenientes do teorema binomial, principalmente:

      $$(1+h)^n \geq 1+nh, \text{ para } h > 0,$$

      e, para o item 3 a seguir:

      $$(1+h)^n \geq 1+nh+\dfrac{n(n-1)}{2}h^2 \geq \dfrac{n(n-1)}{2}h^2, \text{ para } h>0.$$

      1. Mostre que $\displaystyle \lim_{n \to \infty} \sqrt[\leftroot{-2}\uproot{2}n]{a}= 1$ se $a>1$, fazendo $\sqrt[\leftroot{-2}\uproot{2}n]{a}=1+h$ e estimando $h$.

      2. Mostre que $\displaystyle \lim_{n \to \infty} \sqrt[\leftroot{-2}\uproot{2}n]{a}=1 $ se $1<a<1$.

      3. Mostre que $\displaystyle \lim_{n \to \infty} \sqrt[\leftroot{-2}\uproot{2}n]{n}= 1$.


      551   

      Estude o sinal de $f^{\prime }\left( x\right) $, calcule os limites $\lim\limits_{x\rightarrow \infty }f\left( x\right) $ e $\lim\limits_{x\rightarrow -\infty }f\left( x\right) $ e, utilizando esses dados, esboce o gráfico de $f\left( x\right) =x^{3}+3x^{2}+1$.


      1766   

      Mostre, diretamente da definição, que $\log_a'(x)=\dfrac{1}{x} \cdot log_a\left(\lim\limits_{k \to 0}(1+k)^{1/k}\right)$.


      910   

      Quais os valores de $x$ que satisfazem a inequação $\frac{x-3}{x-2}\leq x-1$?


      201   

      Prove que $\log3$ é um número irracional.

      A teoria necessária para resolver esta questão pode não ser abordada em alguns cursos de Cálculo 1. Sendo, também pertinente, às disciplinas Teoria dos Números e Análise Real I.

      Para aprofundar seus conhecimentos, dentro do escopo de Cálculo 1, recomendamos a leitura do Cap. 1 de Guidorizzi, vol. 1 e /ou o Prólogo de Spivak (vide Bibliografia de Cálculo 1).


      Dica: Suponha que existam inteiros $p$ e $q$ tais que $log3=p/q$, com $p/q$ sendo fração irredutível. Use a definição de logaritmo e o teorema fundamental da aritmética para chegar a um absurdo.


      1310   

      Avalie o limite $\lim\limits_{x\rightarrow p}\dfrac{x^{8}-p^{8}}{x-p}$.


      97   

      Calcule os limites:

      1. $\lim\limits_{x\rightarrow 1}\dfrac{\sqrt{x}-1}{\sqrt{2x+3}-\sqrt{5}}$
      2. $\lim\limits_{x\rightarrow 3}\dfrac{\sqrt{x}-\sqrt{3}}{x-3}$
      3. $\lim\limits_{h\rightarrow 0}\dfrac{\sqrt{x+h}-\sqrt{x}}{h}$


      602   

      Encontre os intervalos da reta real nos quais vale a desigualdade $\left| \frac{2x-3}{x+1}\right| \leq \frac{1}{2}.$


      1121   

      Com base no gráfico, avalie as seguintes integrais:

      Construa o gráfico da função 1 e da função 2 usando um único plano cartesiano disponível a seguir

      1. $\int_0^2 f(x)\ dx$
      2. $\int_0^3 f(x)\ dx$
      3. $\int_0^5 f(x)\ dx$
      4. $\int_2^5 f(x)\ dx$
      5. $\int_5^3 f(x)\ dx$
      6. $\int_0^3 -2f(x)\ dx$


      1. $-4$
      2. $-5$
      3. $-3$
      4. 1
      5. $-2$
      6. 10


      1518   

      Seja $f$ uma função definida em $\mathbb{R}$ e suponha que exista $M>0$ tal que $|f(x)-f(p)|\leq M|x-p|$ para todo $x$. Prove que $f$ é contínua em $p$.


      1247   

      Seja $ f(x)=\frac{x^3}{|x^2-1|}.$

      1. Encontre o domínio de $f$, os pontos de intersecção do gráfico de $f$ com os eixos, o sinal de $f$ e analise a simetria de $f$.
      2. Caso existam, determine as assíntotas horizontais, verticais e oblíquas de $f$.
      3. Determine os intervalos de crescimento e decrescimento de $f$, seus pontos de máximo e mínimo locais.
      4. Determine os intervalos onde $f$ tem concavidade para cima e para baixo e os pontos de inflexão.
      5. Esboce o gráfico de $f$ usando as informações obtidas nos itens anteriores.


      40   

      Calcule o limite $\lim\limits_{x\rightarrow \infty }\log _{3}x$.


      $\infty$.


      1682   

      A trombeta de Torricelli, também conhecida como trombeta de Gabriel, em referência à passagem da bíblia na qual o arcanjo Gabriel anuncia o dia do julgamento com sua trombeta,  é uma figura geométrica bastante interessante. Ela é descrita a partir da rotação da função $1/x$ no domínio $x>1$ em relação ao eixo $x$. Calcule sua área e seu volume.



      A área de uma superfície de revolução é dada por:

      $A=\int_{a}^b 2 \pi f(x) \sqrt{1+f'(x)^2}\ dx$


      Assim, temos, para a trombeta de Torricelli:

      $A= \lim\limits_{a\rightarrow \infty}\left(2 \pi \int_{1}^{a}\ \frac{1}{x} \sqrt{1+\left(-\frac{1}{x^2}\right)^2}\ dx\right)$

      Portanto, como $\sqrt{1+\left(-\frac{1}{x^2}\right)^2}>1$:

      $A > \lim\limits_{a\rightarrow \infty}\left( 2 \pi \int_{1}^{a} \frac{1}{x} \ dx\right)= \lim\limits_{a\rightarrow \infty}\left( 2 \pi ln\ a\right)$


      para $a\rightarrow \infty$, vemos que a área $A$ tende ao infinito.

      Quanto ao volume, temos que:

      $V= \lim\limits_{a\rightarrow \infty}\left( \pi \int_{1}^{a} \frac{1}{x^2}\ dx\right)$

      Portanto, obtemos:

      $V= \lim\limits_{a\rightarrow \infty}\ \pi \left(1-\frac{1}{a}\right)$

      Que para $a\rightarrow \infty$ tende a $V=\pi$.


      1178   

      Mostre que:

      1. $x\neq y\Longrightarrow x^{2}+2xy<2x^{2}+2y^{2}$.
      2. $|x|<x^{2}+1,\forall x \in \mathbb{R}$.


      1729   

      Um vaso em formato hemisférico de raio $7,5$cm está sendo enchido de água a uma taxa de $16$cm$^3/$s. Quando a profundidade da água está em $2,5$cm, com que velocidade o nível da água sobe?


      924   

      Uma caixa retangular aberta com volume de $2 m^3$ tem a base quadrada. Expresse a área superficial da caixa como função de um dos lados da base.



      Sejam $x$ a medida do lado da base da caixa e $z$ sua altura. O volume $V$ dessa caixa é dado por $V=x^2z$. Como $V=2$, temos $z=\dfrac{2}{x^2}$. A área superficial $A$ da caixa (sem tampa!) é $A=x^2+4xz$. Substituindo $z$ por $\dfrac{2}{x^2}$ obtemos $A=x^2+\dfrac{8}{x}$. 


      1738   

      Usando as fórmulas pra $\sin(2x), \cos(2x), \sin(3x)$ e $\cos(3x)$, calcule $\sin\left(\dfrac{\pi}{4}\right)$, $tg\left(\dfrac{\pi}{4}\right)$, $\sin\left(\dfrac{\pi}{6}\right)$ e $\cos\left(\dfrac{\pi}{6}\right)$. 


      $\sin\left(\dfrac{\pi}{4}\right) = \dfrac{1}{\sqrt{2}}$.

      $tg\left(\dfrac{\pi}{4}\right) = 1$.

      $\sin\left(\dfrac{\pi}{6}\right)=\dfrac{1}{2}$.

      $\cos\left(\dfrac{\pi}{6}\right)=\dfrac{\sqrt{3}}{2}$. 


      757   

        Seja $f:\mathbb{R\rightarrow R}$ a função
        definida por
        \begin{equation*}
        f\left( x\right) =\left\{
        \begin{array}{cc}
        x^{2} & \text{se }x\leq 1 \\
        2x-1 & \text{se }x>1
        \end{array}
        \right. ,
        \end{equation*}
        e defina $g\left( x\right) =\lim\limits_{x\rightarrow h}\dfrac{f\left(
        x+h\right) -f\left( x\right) }{h}$. Mostre que $g\left( x\right) $ é contínua.


      790   

      Dê a definição de derivada de uma função $f$ no ponto $p\in \mathbb{R}.$ O que é a função derivada $f^{\prime }(x)$?


      642   

      Nos exercícios abaixo determine o domínio máximo de definição de cada uma das funções dadas.

      1. $y=\sqrt[3]{x}$

      2. $y=\sqrt[3]{-x}$


      1. $\mathbb{R}$.
      2. $\mathbb{R}$.


      329   

      Qual o número de raízes distintas da equação $(x^2 – 14x + 38)^2 = 11^2$?


      $3$


      33   

      Calcule os seguintes limites:

      1. $\lim\limits_{x\rightarrow \infty }\left( 5-3x+4x^{2}-x^{3}\right)$

      2. $\lim\limits_{x\rightarrow \infty }\dfrac{5x^{3}-6x-3}{6x^{3}+2}$


      1. $-\infty$
      2. $5/6$


      218   

      Avalie os seguintes limites para a função definida por partes
      $ f(x) = \left\{\begin{array}{ccc}
      2x^2+5x-1, & & \text{ se }  x<0 \\
      \sin x, & & \text{ se }  x\geq 0
      \end{array}
      \right.$

      1. $ \lim\limits_{x\to 0^-} f(x)$

      2. $ \lim\limits_{x\to 0^+} f(x)$

      3. $ \lim\limits_{x\to 0} f(x)$

      4. $f(0)$


      1. $-1$
      2. 0
      3. Não existe.
      4. 0


      1191   

      Calcule a derivada da seguinte função:
       $f\left(  x\right)  =\log_{2}\left(  \cos^{3}\left(  x\right)  \right).$


      $f'(x) = -\dfrac{3 \tan x}{\log 2}$.


      890   

      Dadas $a$ e $b$ constantes reais não nulas, esboce um gráfico da família de funções $f(x)=min\{|x-a|,|x-b|\}$.


      108   

      Mostre que a função $f\left( x\right) =\sqrt[n]{x}$ é contínua em seu domínio.


      72   

       Identifique as assíntotas verticais e horizontais, caso existam, da função

        $f(x)=\frac{2 x^2-2 x-4}{x^2+x-20}$.


      Assíntota horizontal em $y=2$; assíntotas verticais em $x=-5$ e $x=4$.


      1462   

      Nos primórdios da geração comercial de eletricidade, havia uma disputa bastante acirrada entre duas formas de se distribuir energia elétrica: A disputa entre corrente alternada e corrente contínua. A corrente alternada provou-se mais eficiente para transmissão a longas distâncias, principalmente pela facilidade com que é possível elevar os níveis de tensão (e, portanto, para uma mesma potência transmitida, diminuir a corrente e consequentemente os diâmetros dos fios utilizados na transmissão, implicando em significativa economia).

        Com o advento da eletrônica, na segunda metade do século XX, a corrente contínua reconquistou um papel fundamental no dia a dia da sociedade contemporânea, dado que circuitos eletrônicos são alimentados com corrente contínua. A conversão de corrente alternada é feita a partir de dispositivos chamados retificadores. Infelizmente, o funcionamento destes dispositivos foge do escopo desta disciplina.

      As figuras abaixo representam uma corrente $i(t)$ antes e depois de um circuito:

      Construa o gráfico da função 1 e da função 2 usando um único plano cartesiano disponível a seguir

      Construa o gráfico da função 1 e da função 2 usando um único plano cartesiano disponível a seguir

      Responda:

      1. Dado que a função original seja $i_0(t)= \sin(2\pi\ 60\ t)$, qual a relação entre o seu período $T_0$ e o período da corrente retificada $i_1(t)$?
      2. Quais operações sobre a função $i_0(t)$ você realizaria para obter $i_1(t)$?
      3. Qual o valor médio, em um período, de $i_0(t)$? Qual seria sua estimativa para o valor médio de $i_1(t)$?


      1270   

      Calcule a seguinte integral:
         $\int \cos ^{3}xdx.$


      1579   

      Dados $f(x) = x^2+2x$ e $x_0 = 0,1$, escolha um valor inteiro próximo a $x_0$ tal que $f(x_0)$ e $f'(x_0)$ sejam fáceis de calcular, e calcule uma linearização da função neste ponto.


      1181   

      Para cada uma das afirmações abaixo, demonstre se verdadeiro
      ou dê contra-exemplo se for falso.

      1. $\sqrt{x^{2}}=x,\forall x \in \mathbb{R}$.
      2. $x\neq y\Longrightarrow |x|\neq |y|$.
      3. $|x-y|\geq |x|-|y| \forall x,y\in \mathbb{R}$.


      337   

      Enuncie e prove o Algoritmo de Briot-Ruffini. Dê exemplos.


      161   

      Seja $f:\mathbb{R} \to \mathbb{R}$ uma função contínua que satisfaz as seguintes propriedades:

      1. $f(n)=0$, para todo inteiro $n$;
      2. Se $f(a)=0$ e $f(b)=0$ então $f \left(\frac{a+b}{2} \right)$.

        Mostre que $f(x)=0$, para todo real $x$.


      131   

      Dê um exemplo de uma função definida em $\mathbb{R}$ que não seja contínua em $2$ mas que $\lim\limits_{x\rightarrow 2^{+}}f\left( x\right) =\lim\limits_{x\rightarrow 2^{-}}f\left( x\right) .$


      1284   

      Admitindo-se que $\lim\limits_{x\rightarrow a}f(x)$ existe, prove que
        $\lim\limits_{x\rightarrow a}f(x)=\lim\limits_{h\rightarrow0}f(a+h).$


      1775   

      Suponha que uma população de piolhos parasitas de aves, representada por $p$, é estimada no começo do ano de $2015$ em $100 000$ em uma certa região. Um modelo matemático de crescimento da população assume que a taxa de crescimento (em milhares) após $t$ anos é dada por

      $$p'(t)=(4+0,15t)^{3/2}.$$

      Faça uma estimativa para o número de piolhos para o início do ano de 2025.


      1532   

      Sejam $f_1,f_2,\ldots,f_n$, $n \geq 2$, funções deriváveis em $p$. Prove, por indução finita, que $f_1+f_2+\ldots+f_n$ é derivável em $p$. 



      Veja Guidorizzi, volume $1$, página $158$.


      1590   

      Uma criança empina uma pipa a uma altura de $50$m. O vento age sobre a pipa horizontalmente a uma velocidade de $7$m$/$s em relação à criança. Com que velocidade a criança deve soltar a linha quando a pipa estiver a $100$m de distância?


      339   

      Encontre as raízes do polinômio $x^4-7x^3+35x^2-50x+24.$


      697   

      Calcule a integral $\int \dfrac{x^{3}+x}{x-1}dx$.


      921   

      Esboce os gráficos de $f(x) =x^2-1$ e $ g(x) = x^2 +1.$


      549   

      Esboce o gráfico de $f(x)=x^2\sqrt{4-x}$, indicando campo de definição, intervalos de crescimento e de decrescimento, assíntotas horizontais, verticiais e inclinadas (se houver), limites no infinito, extremos relativos, estudo da concavidade, pontos de inflexão e reta tangente à curva nos pontos de inflexão.


      590   

      Encontre os valores de $x$ para os quais cada o número $\sqrt{{\frac{5x-2}{x^{2}-4}}}$ é real.


      1259   

      Resolva a equação $\ln\left(  x^{2}+2x+1\right)  =3$.
        



      Como a função exponencial é estritamente monótona, temos que $\ln\left(  x^{2}+2x+1\right)  =3$ se, e somente se, $e^{\ln\left(x^{2}+2x+1\right)  }=x^{2}+2x+1=e^{3}$. Mas $ x^{2}+2x+1=\left(  x+1\right)  ^{2}$. Logo $\ln\left(  x^{2}+2x+1\right)  =3\Leftrightarrow\left(  x+1\right)^{2}=e^{3}\Leftrightarrow x+1=\pm e^{3/2}\Leftrightarrow x=\pm e^{3/2}-1$.


      745   

      Encontre o volume de um tronco de cone circular reto de altura $h$, raio da base inferior $R$ e raio da base superior $r.$


      569   

      Estude a função  $f\left( x\right) =1-e^{-x}$ com relação à concavidade, pontos de inflexão, máximos e mínimos, e esboce o seu gráfico.


      1671   

      Calcule a integral a seguir utilizando substituições trigonométricas:

      $\int{3\frac{dx}{\sqrt{1+9x^2}}}$


      $sinh^{-1}(3x)+C$.


      1338   

      Usando os limites fundamentais, encontre o limite $\lim\limits_{x\rightarrow0}\frac{\tan x}{x}$.


      $1$.


      1405   

      O terremoto de 1952 em Assam teve uma magnitude de 8,7 na escala Richter - a maior registrada até então. Se o maior terremoto em dado ano tem tem magnitude $R$, então a energia $E$ (em Joules) liberada por todos os terremotos naquele ano é estimada pela fórmula 

      $E=9,13 \times 10^{12} \int_{0}^{R}e^{1,25x}dx$.

      Determine $E$ se $R=8$.


      69   

      Dê um exemplo de uma função tal que $\lim\limits_{x \rightarrow p}\left| f\left( x\right) \right| $ exista mas $ \lim\limits_{x\rightarrow p}f\left( x\right) $ não exista.


      143   

      Para a função a seguir, dê os intervalos nos quais ela é contínua:

       $ g(s) = \ln s$.


        $(0,\infty)$


      1098   

      Avalie a seguinte integral indefinida:
        $\int \frac{1}{\sqrt{x}}\  dx$


      $2\sqrt{x}+C$


      1318   

      Você está projetando uma lata (um cilindro de revolução) de $1000 cm^3$ cuja manufatura levará o desperdício em conta. Não há desperdício ao se cortar a lateral de alumínio, mas tanto a base como o topo, ambos de raio $r$, serão recortados de quadrados que medem $2r$ de lado.

      1. Escreva uma fórmula que forneça a quantidade total de alumínio usada para fazer uma lata.

      2. Qual a razão $h/r$ para a lata mais econômica?


      55   

      Sabemos que limites que tomam a forma indeterminada ``$\infty-\infty$" exigem um pouco mais de trabalho para serem calculados. Calcule, de forma adequada, o limite $\lim\limits_{x\rightarrow\infty}\left(\sqrt{2x^2-7}-x\right)$.


      1465   

      Uma empresa de motores solicitou a fabricação de cilindros com área de seção transversal $A=9cm^2$ (Ou seja, com diâmetro $D=3,385cm^2$). Entretanto, o funcionário que respondeu à solicitação perguntou qual era a margem de erro permitida no diâmetro do cilindro.
      Dado que para o correto funcionamento dos motores o cilindro deve ter uma área $A$ tal que $|A-9|<0,01 cm^2$, e que a área da seção transversal do cilindro é dada por $A=\pi \left(\frac{D}{2}\right)^2$, em qual intervalo deve estar o valor do diâmetro do cilindro para atender tal especificação?


      847   

      Calcule a derivada da função:

      $y=e^{x^{x}}$.


      $y'=e^{x^x} x^x (\log x + 1)$.


      1648   

      Mostre que $\pi^e < e^\pi$. Sugestão: Analise a função $ln(x)/x$.



      Pelas propriedades do logaritmo, podemos escrever:

      $
      ln(e^\pi)=\pi
      $

      e

      $
      ln(\pi^e) = e\ ln(\pi)
      $

      Como $\pi > e$, pode-se escrever $\pi = ae,\ a > 1$. Assim, a primeira equação pode ser escrita como:

      $
      ln(e^\pi)=ae
      $

      E a segunda equação como:

      $
      ln(\pi^e) = e\ ln(a\ e) = e\ ln(a)ln(e)=e\ ln(a)
      $

      Assim, podemos escrever a razão entre as equações como:

      $
      \frac{ln(\pi^e)}{ln(e^\pi)} = \frac{ln(a)}{a}
      $

      Analisando a equação $ln(x)/x$, vemos que para $x>1$ ela é estritamente decrescente, dado que em $x=1$ o denominador é igual a um e o numerador igual a zero e como $\frac{d(ln(x))}{dx}=\frac{1}{x}$ e $\frac{d(x)}{dx}=1$, o denominador cresce mais rapidamente para $x>1$. Assim, como $a>1$, sabemos que:

      $
      \frac{ln(\pi^e)}{ln(e^\pi)} = \frac{ln(a)}{a} < 1
      $

      Portanto:

      $
      ln(\pi^e) < ln(e^\pi)
      $

      Como $\frac{d(ln(x))}{dx}=\frac{1}{x}>0$ para $x>0$, a função logaritmo é monotônica no intervalo desejado, e portanto podemos concluir que:

      $\pi^e < e^\pi$


      76   

      Dê exemplo de duas funções, $f$ e $g$, para ilustrar que se $g(x)\le f(x)$ para todo $x$ suficientemente próximo de $a$, então $\lim\limits_{x\rightarrow a}g(x)\le\lim\limits_{x \rightarrow a}f(x)$.


      1245   

      Calcule a derivada da seguinte função:
           $f\left(  x\right)  =\arcsin\left(  \cos\left(  x\right)  \right)  .$


      -\frac{\sin (x)}{\sqrt{1-\cos ^2(x)}}


      1283   

      Sejam $f$ e $g$ funções contínuas. Demonstre que $h(x)=\max(f(x),g(x))$ é contínua.


      99   

      Calcule e justifique os seguintes limites, quando existirem, ou justifique a inexistência:

      1. $\lim\limits_{x\rightarrow 4}\sqrt{x}$
      2. $\lim\limits_{x\rightarrow 0}\dfrac{x^{2}+3x-1}{x^{2}+2}$
      3. $\lim\limits_{x\rightarrow 1^{+}}\dfrac{\left| x-1\right| }{x-1}$
      4. $\lim\limits_{x\rightarrow 1^{-}}\dfrac{\left| x-1\right| }{x-1}$


      1505   

      Utilizando as leis de exponenciação, simplifique a expressão a seguir:
      $6^{1/3}\cdot18^{1/6}$


      1689   

      Determine o comprimento da curva a seguir no intervalo especificado.

      $y=\int_{-2}^{x}{\sqrt{3t^4-1}dt},\quad -2 \leq x \leq -1$


      1614   

      Calcule o limite $\lim\limits_{x \to \infty}\dfrac{x \ln{x}}{x+\ln{x}}$.


      $\infty$.


      1585   

      Dados $f(x) = e^{-x}$ e $x_0 = -0,1$, escolha um valor inteiro próximo a $x_0$ tal que $f(x_0)$ e $f'(x_0)$ sejam fáceis de calcular, e calcule uma linearização da função neste ponto.


      729   

      Calcule o seguinte limite:

      $\lim\limits_{x\rightarrow -\infty }\left( 2^{x}+2^{-x}\right) $.


      $-\infty$.


      1808   

      Em matemática e estatística, a função de Heaviside (ou função degrau) é uma função singular e descontínua, com valor zero quando o seu argumento é negativo e valor unitário quando o argumento é positivo. Seja $H$ a função de Heaviside. Prove, usando a definição de limite, que $\lim\limits_{x \to 0}H(x)$ não existe.

      Construa o gráfico da função 1 e da função 2 usando um único plano cartesiano disponível a seguir


      537   

      Seja $g\left( x\right) =\int_{0}^{x}f\left( t\right) dt$, onde $f\left( t\right) $ é a função cujo gráfico encontra-se abaixo.

      Construa o gráfico da função 1 e da função 2 usando um único plano cartesiano disponível a seguir

      \begin{equation*} f(t) = \sqrt{|t|}\cos\left(\frac{\pi}{2}t\right) \end{equation*}

      Determine os pontos de máximo e mínimo local de $g\left( x\right) $. Justifique a sua resposta


      886   

      Resolva a equação $|2x+1|=3$.



      Se $2x+1\geq0$: $|2x+1| = 2x+1$, logo $2x+1=3 \Rightarrow x = 1$.
      Se $2x+1<0$: $|2x+1| = -(2x+1)$, logo $-2x-1=3 \Rightarrow x = -2$.
      Portanto $x=1$ ou $x=-2$.


      141   

      Para a função a seguir, dê os intervalos nos quais ela é contínua:

        $ g(x) = \frac{1}{1+x^2}$.


      $(-\infty,\infty)$


      517   

      Calcule, pela definição, a derivada das seguntes funções:

      1. $f\left( x\right) =ax+b$
      2. $g\left( x\right) =ax^{2}+bx+c$.


      1. $f'(x)=a$.

      2.$f'(x)=2ax+b$.


      1108   

       Encontre $f(x)$ que satisfaça o seguinte problema de valor inicial:

        $f'(x) = 5e^x$ e $f(0)= 10$


      $5e^x+5$


      656   

      Esboce o gráfico de cada uma das funções abaixo.

      1. $y=|x|+x$

      2. $y=1-x$ se $x\leq 0$ e $y=\sqrt{1-x^{2}}$ se $0\leq x\leq 1$.


      38   

      Calcule o limite $\lim\limits_{x\rightarrow \infty }\dfrac{1-2^{x}}{1-3^{x}}$.


      $0$.


      541   

      Determine $a$ para que a equação $x^{3}+3x^{2}-9x+a=0$ admita uma única raiz real.



      Primeiramente, calculamos $f'(x)$ e $f''(x)$. Temos então

      $f'(x)=3x^2+6x-9=3(x+3)(x-1)$

      $f''(x)=6x+6$

      Pela análise de sinal da segunda derivada, vemos que $f(x)$ é uma concavidade para baixo para $x<-1$ e uma concavidade para cima para $x>-1$, e os zeros da primeira derivada nos dizem que há um máximo local em $x=-3$ e um mínimo local em $x=1$. Assim, avaliando a função em $x=-3$ tem-se $f(-3) = 27+ a$. Qualquer valor de $a$ que torne $f(-3)<0$ garante que $f(x)$ terá apenas uma única raiz real. Finalmente, portanto, tem-se:

      $a<-27$


      1290   

      Calcule a área do conjunto $A$ dos pontos $\left( x,y\right)$ tais que $x^{2}-1\leq y\leq x+1.$


      891   

      Resolva a equação modular $|x-2|-|x-1| =2$.


      808   

      Calcule $f^{\prime }\left( x\right)$:

      $f\left( x\right) =\dfrac{\sec x}{3x+2}$.


      $f'(x) = \dfrac{\tan x \sec x}{3x+2}-\dfrac{3 \sec x}{(3x+2)^2}$.



      Queremos calcular a derivada da divisão da função $\sec x$ pela função $3x+2$. Usando a regra da derivada do quociente, obtemos:

      \[\left( \dfrac{\sec x}{3x+2} \right)^\prime = \dfrac{(\sec x)^\prime \cdot (3x+2) - (\sec x)\cdot (3x+2)^\prime}{(3x+2)^2}.\]

      Como $\sec x = \dfrac{1}{\cos x}$, podemos usar a regra do quociente para calcular sua derivada:

      \[(\sec x)^\prime = \left(\dfrac{1}{\cos x}\right)^\prime = \dfrac{(1)^\prime\cdot \cos(x) - 1\cdot (\cos x)^\prime}{(\cos x)^2} =\dfrac{0 - (-\sin x)}{(\cos x)^2} = \tan(x)\sec(x).\]

      Por outro lado, sabemos que $(3x+2)^\prime = 3$.

      Dessa forma, voltando à primeira igualdade e substituindo $(\sec x)^\prime$ e $(3x+2)^\prime$ pelas expressões encontradas, obtemos:

      \[\dfrac{(\sec x)^\prime \cdot (3x+2) - (\sec x)\cdot (3x+2)^\prime}{(3x+2)^2} = \dfrac{\tan(x) \sec(x) (3x+2) - (\sec x)(3)}{(3x+2)^2} .\]

      Ou seja,

      \[ \left( \dfrac{\sec x}{3x+2} \right)^\prime = \dfrac{\tan(x) \sec(x)}{3x+2} - \dfrac{3\sec(x)}{(3x+2)^2}. \]


      1735   

      Seja $f(x)=ax^2+bx+c$, onde $a>0$. Prove que $f(x)\geq 0$ para todo $x$ se, e somente se, $b^2-ac\leq 0$. [Sugestão: ache o mínimo de $f(x)$.]


      193   

      O princípio da Boa Ordenação diz que todo subconjunto não-vazio de $N$ possui elemento mínimo. Demonstre que $N$, com a relação $\leq$, verifica o Princípio da Boa Ordenação.

      A teoria necessária para resolver esta questão pode não ser abordada em alguns cursos de Cálculo 1. Sendo, também pertinente, às disciplinas Teoria dos Números e Análise Real I.

      Para aprofundar seus conhecimentos, dentro do escopo de Cálculo 1, recomendamos a leitura do Cap. 1 de Guidorizzi, vol. 1 e /ou o Prólogo de Spivak (vide Bibliografia de Cálculo 1).


      1737   

      Usando as fórmulas do seno da soma e do cosseno da soma de dois ângulos, obtenha fórmulas para:

       $\sin(2x), \cos(2x), \sin(3x)$ e $\cos(3x)$.


      $\sin(2x) = 2 \sin x \cos x$.

      $\cos(2x) = \cos^2 x - \sin^2 x$.

      $\sin(3x) = \sin x (2 (\cos^2 x - \sin^2 x) + 1)$.

      $\cos(3x) = \cos^3 x - 3 \sin^2 x \cos x$.


      1497   

      Sejam $a$ e $b$ reais quaisquer. Verifique que:

      1. $\sin{a}\cos{b}=\dfrac{1}{2}(\sin(a+b)+\sin(a-b))$
      2. $\cos{a}\cos{b}=\dfrac{1}{2}(\cos(a+b)+\cos(a-b))$



      1. $\begin{array}{rcl} \frac{1}{2} ( \sin(a+b) + \sin(a-b) ) &=& \frac{1}{2} ( \sin a \cos b + \sin b \cos a + \sin a \cos b - \sin b \cos a )  \\ &=& \frac{1}{2} ( 2 \sin a \cos b) \\ &=& \sin a \cos b .\end{array}$
      1. $\begin{array}{rcl} \frac{1}{2} ( \cos(a+b) + \cos(a-b) ) &=& \frac{1}{2} ( \cos a \cos b + \sin a \sin b + \cos a \cos b - \sin a \sin b )  \\ &=& \frac{1}{2} ( 2 \cos a \cos b) \\ &=& \cos a \cos b .\end{array}$


      667   

      Use o Teorema Fundamental do Cálculo e a Regra de l'Hospital para calcular o limite abaixo, justificando claramente sua resolução.
        \begin{equation*}
        \lim\limits_{x\rightarrow 0}\left( \dfrac{\displaystyle\int_{0}^{x}e^{t^{2}}dt}{x}\right)
        \end{equation*}


      758   

      Calcule o limite $\lim\limits_{x\rightarrow 7}\frac{\sqrt{x}-\sqrt{7}}{\sqrt{x+7}-\sqrt{14}}$ usando uma estratégia algébrica simples e, em seguida, usando a regra de L'Hospital. Compare os resultados.


      1464   

      A Lei de Ohm para circuitos elétricos, afirma  que a queda de tensão em um resistor $R$ sob corrente $I$ é $V=RI$. Uma empresa recebeu pedidos de fornecimento de resistores para um circuito como o da figura a seguir. Neste circuito, $V=120V$ e, para atender as especificações de segurança e de funcionamento desejado do circuito, a corrente deve ser $I=5\pm0,1A$. Em que intervalo $R$ deve ficar para que $I$ esteja dentro da margem de segurança?

      Construa o gráfico da função 1 e da função 2 usando um único plano cartesiano disponível a seguir



      Pela Lei de Ohm, conseguimos escrever que $I=\frac{V}{R}$. Para $V=120V$ fixo, a corrente depende portanto apenas do valor da resistência, sendo inversamente proporcional a esta. 

      A corrente deve estar no intervalo $4,9 \leq I \ leq 5,1$. Temos que $R_{max}=\frac{120}{I_{min}}\approx 24,49$ e $R_{min}=\frac{120}{I_{max}}\approx 23,53 \Omega$.

      Portanto, $23,53 \leq R \leq 24,49$.


      1653   

      Uma superfície é criada a partir de segmentos de reta perpendiculares traçados sobre um círculo de raio $a$, perpendiculares ao plano do círculo. O comprimento de um segmento correspondente a um ponto $p$ sobre o círculo é $ks$, sendo $k$ uma constante positiva e $s$ o comprimento de arco do círculo no sentido anti-horário de $(a,0)$ até o ponto $p$.

      Determine a área desta superfície, conforme a figura a seguir, em função de $k$.

      Construa o gráfico da função 1 e da função 2 usando um único plano cartesiano disponível a seguir


      202   

      Prove que $\log2+\log3$ é um número irracional.

      A teoria necessária para resolver esta questão pode não ser abordada em alguns cursos de Cálculo 1. Sendo, também pertinente, às disciplinas Teoria dos Números e Análise Real I.

      Para aprofundar seus conhecimentos, dentro do escopo de Cálculo 1, recomendamos a leitura do Cap. 1 de Guidorizzi, vol. 1 e /ou o Prólogo de Spivak (vide Bibliografia de Cálculo 1).


      Dica: Note que $\log2+\log3=\log6$. Suponha que existam inteiros $p$ e $q$ tais que $log6=p/q$, com $p/q$ sendo fração irredutível. Use a definição de logaritmo e o teorema fundamental da aritmética para chegar a um absurdo.


      1673   

      Calcule a integral a seguir:

      $\int_{0}^{ln\ 4}{\frac{e^t dt}{\sqrt{e^{2t}+9}}}$


      Aproximadamente $0,77116$


      1309   

      Avalie o limite $\lim\limits_{x\rightarrow p}\dfrac{x^{4}-p^{4}}{x-p}$.


      1490   

      Reescreva a função $f(x)=|x-1|+|x+2|$ usando desigualdades e representação por partes. Esboce o gráfico de $f$


      521   

      A que taxa o nível do líquido diminui dentro de um tanque cilíndrico vertical de raio $2$ metros se bombearmos o líquido para fora a uma taxa de $3000$ litros por minuto?


      1670   

      Calcule a integral a seguir utilizando substituições trigonométricas:

      $\int{\frac{dx}{\sqrt{9+x^2}}}$


      $sinh^{-1}(x/3)+C$.


      628   

      Encontre a área do maior retângulo que pode ser inscrito em um triângulo retângulo com catetos de comprimento $3$ e $4cm$, se dois lados do retângulo estiverem sobre o cateto.


      32   

      Calcule os seguintes limites:

      1. $\lim\limits_{x\rightarrow \infty }\left( x-\sqrt{x^{2}+3}\right)$

      2. $\lim\limits_{x\rightarrow -\infty }\left( x-\sqrt{x^{2}+3}\right)$

      3. $\lim\limits_{x\rightarrow \infty } \left( \sqrt{x+1}-\sqrt{x+3}\right)$


      1. $0$
      2. $-\infty$
      3. $0$


      1831   

      Uma grandeza física desconhecida é medida $n$ vezes, obtendo-se valores $x_1,x_2,\ldots,x_n$, cuja variação  depende de fatores imprevisíveis, tais como temperatura, pressão atmosférica etc. Desta forma, o cientista  enfrenta o problema de obter uma estimativa $\bar{x}$ de uma grandeza desconhecida $x$. Um método de se  obter estimativas está baseado no princípio dos mínimos quadrados, o qual estabelece que a estimativa $\bar{x}$  deve ser escolhida de forma a minimizar a função  $$ s= (x_1-\bar{x})^2+(x_2-\bar{x})^2+\ldots +(x_n-\bar{x})^2, $$que é a soma dos quadrados dos desvios entre a estimativa $\bar{x}$ e os valores medidos.  Mostre que a estimativa resultante do princípio dos mínimos quadrados é dada por  $$ \bar{x}= \dfrac{1}{n}(x_1+x_2+\cdots+x_n), $$  ou seja, $\bar{x}$ é a média aritmética dos valores observados.


      1511   

      O quociente $(log_4\ x)/(log_2\ x)$ possui um valor constante. Qual valor é este?


      936   

      Se $ f(x) = \sqrt{x} $ e $ g(x) =\sqrt{2-x},$ encontre e determine o domínio das funções:

      1. $f \circ g (x).$
      2. $g \circ f(x).$
      3. $f \circ f (x).$
      4. $g \circ g(x).$


      1908   

      Ache uma reta vertical $x=k$ que divida a área entre as curvas $y=x^2$ e $y=9$ em duas partes iguais.


      $x=k=0$


      1796   

      Uma alternativa ao método das frações parciais é calcular integrais da forma

      $$\displaystyle \int \dfrac{1}{ax^2+bx} \, dx$$

      utilizando a substituição $u=a+\dfrac{b}{x}$. Mostre que com essa substituição a integral se torna:

      $$\displaystyle \int \dfrac{1/x^2}{a+b/x} \, dx.$$


      1308   

      Mostre que existem funções $f(x)$, $g(x)$ com  $\lim_{x\rightarrow p} f(x) = \lim_{x\rightarrow p} g(x) =0,$ tais que $\lim_{x\rightarrow p} (f(x)/g(x)) =\lambda$, onde $\lambda$ assume qualquer valor em $\mathbb{R} \cup \{+\infty, -\infty\}$. Escolha o ponto $p$ como achar mais conveniente.


      1587   

      Determine a linearização de $f(x) = \sqrt{x+1} + \sin x$ em $x=0$. Como ela se relaciona com as linearizações individuais de $\sqrt{x+1}$ e $\sin x$ em $x=0$?


      1786   

      1.  Use o Teorema do Valor Médio para mostrar que, $$\sqrt{y}-\sqrt{x}<\dfrac{y-x}{2\sqrt{x}},$$ quando $0<x<y$.

      2.  Use o resultado anterior para mostrar que se $x$ e $y$ forem positivos, então $$ \sqrt{xy} < \dfrac{1}{2}(x+y).$$ (A média aritmética é maior que a média geométrica). 

      3.  Tente generalizar o resultado anterior para um conjunto amostral discreto de tamanho $n>2$.


      1265   

      Calcule a seguinte integral:
         $\int{x^2e^{2x}dx}.$


      $\dfrac{1}{4}e^{2x}(2x^2-2x+1)+C$.


      203   

      Utilizando o gráfico a seguir, avalie os seguintes limites

      Construa o gráfico da função 1 e da função 2 usando um único plano cartesiano disponível a seguir

      Seja $-3\leq a\leq 3$ um número inteiro

      1. $ \lim\limits_{x\to a^-} f(x)$
      2. $ \lim\limits_{x\to a^+} f(x)$
      3. $ \lim\limits_{x\to a} f(x)$
      4. $f(a)$


      1. $a-1$
      2. $a$
      3. Não existe.
      4. $a$


      1906   

      A figura mostra as curvas aceleração versus tempo para dois carros movendo-se em uma pista reta, começando alinhados e acelerando a partir do repouso. O que representa a área $A$ entre as curvas no intervalo $0 \leq t \leq T$? Justifique.

      Construa o gráfico da função 1 e da função 2 usando um único plano cartesiano disponível a seguir



      Para uma aceleração $a(t)$ qualquer, $\int_0^Ta(t)\,dt$ representa a velocidade adquirida desde o instante $t=0$, ou seja, $v(T)= v_0+\int_0^Ta(t)\,dt$. Sendo assim, a área entre as curvas representa $v_1(T)-v_2(T)$, a diferença de velocidade entre os carros no instante $t=T$.


      109   

      Mostre que a função $f\left( x\right) =\left\{ \begin{array}{cc} \dfrac{x^{3}-8}{x-2}, & \text{se }x\neq 2 \\ 12, & \text{se }x=2 \end{array}\right. $ é contínua em seu domínio.


      946   

      Mostre que

      1. o limite de $f(x)=\dfrac{x-2}{|\,x-2|}$, quando $x\to 2$, não existe.
      2. o limite de $f(x)=\left\{\begin{array}{ll} x^2+2, & x\geq -1 \\ 2x+1, & x<-1 \\ \end{array}\right.$, quando $x\to -1$, não existe.


      591   

      Encontre os valores de $x$ para os quais cada o número $\sqrt[4]{{\frac{x^{2}-x-2}{x^{2}-4x-3}}}$ é real.


      66   

      Sendo $f(x) = \left\{\begin{array}{cl} \cos x & x\leq 0 \\ x^2+3x+1 & x>0 \end{array}\right.$, calcule $\lim\limits_{x\to 0} f(x)$.


      1


      1317   

      Será construído um campo de atletismo retangular, com $x$ unidades de comprimento, tendo nas extremidades duas áreas semicirculares com raio $r$. O campo terá em volta uma pista para corrida com $400 m$ de extensão.

      1. Expresse a área da porção retangular do campo só em função de $x$ ou só em função de $r$ (a escolha é sua).

      2. Quais valores de $x$ e de $r$ dão à porção retangular maior área possível?


      1100   

      Avalie a seguinte integral indefinida:
        $\int \sec^2\theta\  d\theta$


       $\tan \theta+C$


      1680   

      Calcule a seguinte integral:

      $\int_{0}^{\infty}{\frac{dx}{x^{1,001}}}$


      Não converge.


      626   

      Um fabricante de óleo deseja confeccionar latas cilindricas de volume igual a $1$ litro. Quais são as dimensões da lata para que o consumo de material seja o mínimo possível? Se a lata fosse esférica, o gasto de material seria maior ou menor que o gasto de material da lata cilíndrica que voce encontrou? E se a lata fosse cúbica?


      124   

      Classifique a veracidade das afirmações a seguir

      1. Se $f$ é contínua em $c$, então $\lim_{x\to c^+}f(x) = f(c)$.
      2. Se $f$ é contínua em $c$, então $\lim_{x\to c}f(x)$ existe.
      3. Se $f$ é definida em um intervalo aberto contendo $c$, e $ \lim_{x\to c}f(x)$ existe, então $f$ é contínua em $c$.


      1. Verdadeiro
      2. Verdadeiro
      3. Falso


      1331   

      Determine uma primitiva para cada uma das funções:

      1. $f(x)=x^n$

      2. $f(x)=sen(x)$


      1111   

      Encontre $f(x)$ que satisfaça o seguinte problema de valor inicial:
        $f'(x) = 4x^3-3x^2$ e $f(-1)= 9$


        $x^4-x^3+7$


      85   

      Calcule os limites:

      1. $\lim\limits_{x\to\pi/6} cos(sec x)$

      2. $\lim\limits_{x\to0} \ln(1+x)$


      144   

      Para a função a seguir, dê os intervalos nos quais ela é contínua:

       $ h(t) = \cos t$.


        $(-\infty,\infty)$


      920   

      Um fabricante de refrigerante quer produzir latas cilíndricas para seu produto. A lata dever ter um volume de $360 ml$. Expresse a área superficial total da lata em função do seu raio e dê o domínio da função.



      Sejam $r$ o raio da base do cilindro e $h$ a sua altura. O volume $V$ do cilindro é dado por $V=\pi r^2 h$. Como $V=360$, obtemos $\pi r^2 h=360$, isto é, $h=\dfrac{360}{\pi r^2}$. A área superficial $A$ do cilindro é $A=2 \pi r^2+2 \pi r h$. Substituindo $h$ por $\dfrac{360}{\pi r^2}$ chegamos a $A=2 \pi r^2+2 \pi r \dfrac{360}{\pi r^2}$, ou seja, $A=2 \pi r^2+ \dfrac{360}{r}$. O domínio da função $A(r)$ é $\mathbb{R}^+$.


      1724   

      Obtenha a fórmula da distância entre dois pontos quaisquer no plano cartesiano. Use o teorema de Pitágoras. Veja o livro: Simmons, página $11$.


      1708   

      Defina ``$\displaystyle \lim_{x \to -\infty} f(x) = l$''.

      1. Ache $\displaystyle \lim_{x \to -\infty} \dfrac{a_n x^n + \ldots + a_0}{b_m x^m + \ldots + b_0}$.

      2. Mostre que $\displaystyle \lim_{x \to \infty} f(x) = \displaystyle \lim_{x \to -\infty} -f(x)$.

      3. Mostre que $\displaystyle \lim_{x \to 0^-} \dfrac{1}{f(x)} = \displaystyle \lim_{x \to -\infty} f(x)$.


      1661   

      Calcule a seguinte integral:

      $\int{e^x(x^2-2x+1)dx}$.


      $e^x(x^2-4x+5)+C$


      837   

      Determine a derivada da função:

      $f\left( x\right) =\left( sen x+\cos x\right) ^{3}.$


      $3 (\cos (x)-\sin (x)) (\sin (x)+\cos (x))^2$


      673   

      Se $f$ for contínua e $\int_{0}^{9}f\left( x\right) dx=9$, calcule $\int_{0}^{3}xf\left( x^{2}\right) dx.$


      47   

      O gráfico da função $f(x)=\frac{x^3+2x^2+1}{5-x^2}$ possui alguma assíntota horizontal?


      Não possui.


      659   

      Mostre que a área $A$ de um círculo de raio $r$ é $A=\pi r^{2}$.


      578   

      Esboce o gráfico da função $f\left(x\right) =e^{-x^{2}}$ explicitando domínio, intervalos de crescimento e decrescimento, concavidade, pontos de inflexdão, assíntotas, máximos e mínimos locais e globais.


      1622   

      Para calcular as coordenadas espaciais de um planeta, temos de resolver equações do tipo $x=1+0,5\sin(x)$. O traçado da função $f(x)=x-1-0,5\sin(x)$ sugere que a função possui uma raiz próxima de $x=1,5$. Utilize uma iteração do Método de Newton para melhorar essa estimativa, com $x_0=1,5$.


      43   

      Considere a função $f(x) = 2^x+10$. Calcule os seguintes limites e, depois, discuta se a função $f(x)$ tem assíntotas horizontais.

      1. $\lim\limits_{x\to -\infty} f(x)$.

      2. $\lim\limits_{x\to \infty} f(x)$.


      1. $10$.

      Possui assíntota horizontal de equação $y=10$,


      1535   

      Calcule $F'(x)$ sendo $F(x)$ igual a:

      1. $x^2e^x\cos{x}$
      2. $e^x \sinh{x} \cos^2{x}$


      918   

      Para tranformar graus Fahrenheit em graus centígrados usa-se a fórmula $C=\dfrac{5(F-32)}{9}$, em que $F$ é o número de graus Fahrenheit e $C$ é o número de graus centígrados.

      1. Transforme $35$ graus centígrados em graus Fahrenheit.
      2. Qual a temperatura (em graus centígrados) em que o número de graus Fahrenheit é o dobro do número de graus centígrados?


      760   

      Mostre que a equação
        \begin{equation*}
        x^{26}+x^{2}-320=0
        \end{equation*}
        possui ao menos uma raiz real positiva e também uma raiz real negativa.


      121   

      De acordo com o gráfico de $f(x)$, avalie a continuidade da função em $x=0$

      Construa o gráfico da função 1 e da função 2 usando um único plano cartesiano disponível a seguir


      $f$ é contínua em $x=0$.


      1683   

      Calcule o valor de $p$ para a integral a seguir convergir:

      $\int_{1}^{2}{\frac{dx}{x\left(ln\ x\right)^p}}$


      1896   

      Prove o teorema do valor médio para integrais aplicando o teorema do valor médio para derivadas(consulte Stewart, seção 4.2, para obter mais informações sobre a função $F(x)=\displaystyle\int_{a}^{x} f(t)dt$.


      346   

      Seja $P(x)$ um polinômio de grau $n$ tal que $P(k)=k/(k+1)$ para $k=0,1,\ldots n$. Encontre $P(n+1)$.


      963   

      Calcule e justifique os seguintes limites, quando existirem, ou justifique a inexistência:

      1. $\lim\limits_{x\rightarrow p}\dfrac{\tan \left( x-p\right) }{x^{2}-p^{2}}$
      2. $\lim\limits_{x\rightarrow p}\dfrac{\sin x-\sin p}{x-p}$
      3. $\lim\limits_{x\rightarrow p}\dfrac{\cos x-\cos p}{x-p}$

      Como fazer uma função no plano cartesiano?

      Construa uma tabela com duas colunas, na primeira coloque valores de x (domínio) e na segunda os valores de f(x) (imagem da função). Marque no plano cartesiano os pares ordenados (x,y), depois trace a reta da função.

      Qual é o gráfico da função y 2x 1?

      a) y = 2x + 1 é uma função afim, pois a = 2 e b = 1. b) y = 2x é uma função afim, pois a = 2 e b = 0. Como a função afim possui a mesma lei de formação das funções do primeiro grau vistas no Ensino Fundamental, o seu gráfico é igual ao gráfico da função do primeiro grau: uma reta.

      Como fazer um gráfico de uma função do 1 grau?

      Na construção de um gráfico de uma função do 1º grau basta indicar apenas dois valores pra x, pois o gráfico é uma reta e uma reta é formada por, no mínimo, 2 pontos. Apenas um ponto corta o eixo x, e esse ponto é a raiz da função. Apenas um ponto corta o eixo y, esse ponto é o valor de b.

      Como construir o gráfico de uma função do 2 grau?

      Cinco passos para construir o gráfico de uma função do 2º grau.
      → Primeiro passo: Calcular o valor de ∆.
      → Terceiro passo: Encontrar as raízes (quando possível).
      → Quarto passo: Calcular pontos (quase) aleatórios..
      → Quinto passo: Desenhar o gráfico..